Wednesday, September 26, 2012

Endeavor's Passing

Shuttle Endeavor above Golden Gate Bridge

I never got to see any of the Shuttles during their active service as spacecraft. Planned West Coast military launches from Vandenberg were scrubbed after the Challenger loss. I did hear the double sonic boom a couple of times  when Shuttles crossed the coast en route to landings at Edwards AFB.

And only by sheerest luck did I end up seeing Endeavor on its final piggyback flight last week: Paula happened to turn on local TV news half an hour or so before it overflew San Francisco en route to retirement in Southern California.

It was an impressive sight. But like the sight of USS Iowa passing under the same bridge, it was a somewhat melancholy occasion. What in my early adulthood was the spaceship of the future has become a spaceship of the past.

Will we see its like again?


The Shuttle program was star-crossed in multiple ways. Thus its experience does not provide a 'fair' test of reusable spacecraft. This is the good news.

First of all, it conflated the roles of experimental prototype and operational vehicle: a beta pushed into production. Its development costs were squeezed, compromising the design, and further compromised by demanding of it an enormous payload capacity.

Given all these fundamental shortcomings it is remarkable that it succeeded at all. It surely cost far more to operate over its service career than either a capsule atop a conventional rocket or a smaller, fully reusable spacecraft, refined from a prototype, would have cost - not to mention the greatest and most needless cost, two large crews. Spaceflight is dangerous, but the points of failure for both Challenger and Columbia were direct results of the flawed development process.

We cannot say how much a more robust Shuttle would have cost to operate, or how safe it would have been. All we can say is that it would have been cheaper and safer than the ones we actually flew.


Unfortunately, we can also say that a more robust Shuttle would have been - would still be - horrendously expensive to develop. The projected development cost of the original Shuttle design, circa 1969 - before the compromises mentioned above - was on order of $10 billion. This is equivalent to $60 billion in present-day dollars. Hear the deafening sound of wallets snapping shut.

And it gets worse. Assume a 30-year service lifetime, with monthly launches - less than hoped for, but a lot more human spaceflights than we have actually flown. The apportioned development cost - ignoring interest and such - thus comes to about $170 million per mission. Remember, this does not include any of the costs of actually flying the missions, or training the astronauts, or anything else: It is just the development cost leading up to the first operational flight.


There is also a line of argument, all too credible, that a truly viable reusable spacecraft - one that is cheaper in the long run than expendable rockets - is just not attainable at our techlevel. We know a lot about building large, lightweight structures, along with powerful engines able to drive them into the upper atmosphere or even into space. We can do significantly better at these things now than we could fifty years ago, but not dramatically better - an indication that our technology in these areas is pretty mature.

But getting into space is so intrinsically difficult that our normal technique involves large, expendable boosters or lower stages. Payloads are, at most, a few percent of launch weight. And the problem for reusable spacecraft is that they must carry heavy fittings, such as heat shields, along with wings and landing gear, that their expendable counterparts can do without.

A simple design burdened with these heavy fittings probably couldn't reach orbit at all. But a design refined to the point that it can reach orbit is liable to be so extreme in its specifications that it requires extensive  tear-down and inspection, and perhaps refurbishment, after every flight. Which defeats the whole point of being 'reusable.'


In all of this there is a glimmer of hope: We are not dealing here with 'cold equations' but with devils in the details, and the lines between not-quite-feasible and just-feasible are pretty fine. And even as the Shuttle rode off into the west, the US has begun operating another spaceplane, the X-37B.

This is in no sense a 'Shuttle replacement.' It is very much smaller, launched atop a conventional expendable rocket, and it is unmanned. It is also a classified DARPA project - even thought it began as a NASA project - meaning that not much is being said either about its performance or its missions. But it may well be more operationally robust than the Shuttle - in particular, safer during re-entry.

Incremental progress in mature technologies is glacially slow compared to the Moore's Law-style progress seen in tech revolutions. But in the course of this century we might (or might not!) gradually develop our launch capabilities to a level approaching what the Shuttle once hoped to achieve.

After which, things could get interesting.


Discuss.




Note: A recent, truly awesome XKCD comic has a relevant comment on space rocketry. You will have to look ... carefully ... to find it.




Another Note: Unrelated to this post, but a blog reader has done the service of converting my Planetary Climate Sim into Linux and Win32 object code.

The sim itself is designed primarily to test the effects on an Earthlike planet's climate of greater orbital eccentricity or different axial tilt. It also has some settings for different average locations within the habitable zone, greenhouse gas composition, differing proportions of ocean and land surface, and so on. But these things are far more complex, and pretty much above my pay grade.

 I don't warranty the results! And I haven't tested the Linux and Win32 versions at all - let me know how they work! I'll pass any bug reports along to the contributor.




The image of Endeavor and its 747 carrier above the Golden Gate Bridge comes, a bit paradoxically, via the Baltimore Sun.

249 comments:

1 – 200 of 249   Newer›   Newest»
Brett said...

I wonder if some kind of orbital re-fueling capability would make renewal launched spacecraft much more viable. They would actually be able to slow down before re-entering into the atmosphere, reducing the impact of re-entry on the spaceship.

Until then, I'm okay with just using an expendable ship with the appropriate balance of cheap and safe. It's not like we're in imminent danger of running out of the components of rocketry.

 Ashley said...

As you imply, the end of an era. The hopes and dreams of the shuttle promised became rather grimdark with the Challenger disaster.

The drive to go where no one has gone before comes at a cost, be it dollars, or lives. I suspect that at this point in time that the costs are too high.

If alternative technologies, and hybrid drives could be effective then one might see a change in the cost. Until then it will be low risk, and cheap'ish disposable booster launches.

longbeast said...

The problem of trying to cram all your reusability components into the last few percent-by-mass of a giant flying fuel tank doesn't necessarily have to be true. It very much depends on how many stages you're trying to achieve.

SSTO is obviously going to be very difficult. Two stages is a quite a bit easier if you're willing to build a large and fast carrier plane. There's no particular reason why you have to stop there.

Back in ye olden days, Wernher von Braun imagined four stage rockets for which three stages were fully reusable: http://s2.hubimg.com/u/154249_f260.jpg

The trick of getting that to work is not so much fighting against fundamental limits of the technologies involved, but just finding a sensible way to fit all the parts together.

Anonymous said...

Reaction Engines Limited in the UK is developing an engine that starts off as an air breather and then converts to a hydrogen and oxygen rocket. That means their proposed Skylon single stage to orbit spaceplane doesn't have to carry all the oxygen it needs to reach orbit.

Since fuel is most of the mass of a chemical rocket, cutting down on how much you have to carry will be a big advance.

BTW, Skylon looks very sci-fi.

Ron

jollyreaper said...

I'll bring up SpaceX again. They're not so much inventing brand new technologies but using streamlined production methods and thinking differently about how the parts go together. So it's along the lines of "we didn't invent a better mousetrap so much as inventing a cheaper, faster, more reliable way of building one."

Video of reusable concepts.

I'd initially put Musk in the big talker camp. With his successes in Tesla and space flight, I'm moving him to the big talker, big delivery category. I won't tell you 100% for sure he's going to make it happen but it's a lot more likely than most other big-talking promoters.

jollyreaper said...

One other thought that I think we've hit upon on this site before: there's a minimal level of societal complexity before certain projects can be attempted, simply based on the difficulty of the task and the number of specialized workers required for each element.

Technology increases the amount of work each person can perform, both physically and mentally. It's often pointed out that over 90% of society was directly involved with food production and anything else aside from farming was the result of that surplus, the priests and kings and coopers and blacksmiths and merchants and other cityfolk coming from the less than 10% not on the farm.

Every form of human organization eventually moves from being about the primary cause to being about self-perpetuation. Iron law of bureaucracy as discussed before.

So these are the general facts and truisms as I see them and what I consider then navel-gazing at the future:

1. All human organizations (religions, governments, hobby associations, everything) grow corrupt in time with the potential for a firebrand making a reformation from time to time.

2. Larger than life people who can turn the course of organizations usually have to be the people who built them in the first place. Once an organization has grown large enough, petty politics will prevent any potential new leader from accumulating such power because it is a threat to entrenched interests. And dynamic leaders are often assholes who will gore a lot of oxen in pursuit of their dream.

3. We hear about success, not so much about failure. So the brilliant innovators who flamed out early pass beneath our notice and we pay attention to the few who won big. Small organizations are psychologically prepared to take far bigger risks than larger organizations who feel they have more to lose.

4. The sands can shift beneath the feet of even the most formidable giant. The conditions that allowed the giant to become great and mighty may come to no longer favor him but a rival, only this will likely only be blindingly obvious in hindsight.

5. There's "the way things work, general case" and "the way things work in our particular case." Greed and vice, love and loyalty, dreams and ambition, all of these factors are part of the human condition and shared across time. But the special case is particular to time and place, the shifting sands alluded to. People who have invested a considerable portion of their lives learning those rules are seldom happy to see that hard-won wisdom become obsolete. Samurai in an era of peace, the windjammer captain in the age of steam, the bootlegger at the end of Prohibition, the Mustache Pete mafiosos with their old world ways clashing with the young turks who grew up in America and had new ideas, etc. An old criminal who grew up in the era of mom and pop stores was at a loss looking at a big box store. "I don't even know how to rob the place," he lamented. (American Gangster, good book.)

6. "It can't be done." Consider the source. For them, it very likely could be. Cheaper ELV's likely cannot be provided by the traditional aerospace community. But consider that they are not in the business of building rockets: they are in the business of keeping executives in bonuses and it just so happens they do that through selling expensive rockets to customers. They have a good thing going, a large enough that it is impossible for dominant personalities to set course. Large organizations are run by committees, not larger-than-life visionaries. And the concern is understandable. It's hard to tell the difference between a visionary who is smarter than the rest of the room and a lunatic. And when the purpose of the business at this point is comfortable persistence rather than revolutionary nonsense, why rock the boat? So saith Kodak. So saith the RIAA and MPAA. So saith Microsoft.

jollyreaper said...

7. Technology is lowering the amount of organizational complexity required to accomplish a given task. Wikipedia would not be possible without the internet. Just consider the difficulty of running Rocketpunk as a scifi newsletter. It would take a lot more time and effort in the photocopier days, let alone if Our Good Host needed his own printing press. Look at how Kickstarter isn't a new idea (float an idea before a pool of investors, beg for money, that's capitalism 101) but it's a brand new way of implementing that idea. It bypasses the old gatekeepers of industry, the gatekeepers whose business models are based on the old way of doing things, the old truisms. Old advantages can become irrelevant, having a large and pyramidal bureaucracy and a hefty bulk could become as irrelevant to the significance of a business as the number of battleships for a WWII navy.

8. Of course, prognostication is a delicate science. The atom bomb was predicted to be the end of armies and navies. It wasn't. The guided missile would be the end of the aircraft gun. That was a premature prediction for Vietnam. The missiles are better now, we haven't had an aerial gun kill since an A-10 vs. a chopper in Gulf War 1, but you'll note the F-22 and F-35 still carry internal cannons. And despite all of our other advancements, the basic means of fighting a war is with a grunt and a rifle. Air supremacy helps your grunt win the war but doesn't win the war without the grunt.

9. We usually get it wrong. We know we are susceptible to myopia, we look for what we're overlooking, and we can foolishly discount things that are timeless and put our stock in things that won't last. In terms of point 5, we can't tell the difference between general case and special case. In trying to avoid blunders, we can often blunder into them.

jollyreaper said...

forgot to hit track when posting.

johnny abacus said...

You might be interested in the Google Maps version of the XKCD comic

here

Tony said...

Re: Shuttle

The original sin was a philosophical belief in resuability without a corresponding engineering solution. Putting 120 tons in orbit, only to bring 100 of them back, was simply engineering nonsense then, and it still is now. It just costs too much to put stuff in space to bring anything back, except for the minimum RV needed to return humans and scientific samples. It doesn't seem like costs are coming down any time soon.

Re: SpaceX and expendable launch vehicle futures

First of all, launch providers do not sell the rocket to the customer. They sell a launch service to the customer. The rocket is a consumable item of inventory. Though not a fungible item -- the customer is generally assigned a specific airframe, to which significant mission-specific modifications are usually made; but the customer never "owns" the rocket in an economic sense. If the customer fails to deliver on his end of the bargain, he doesn't sell the rocket back to the launch services provider. The provider just reassigns it to another customer.

So there's really no selling expensive rockets to rich customers. It's about selling the launch service at cost plus profit to a customer who has the money to pay. SpaceX thinks it can lower costs in the long run. I'm on record here and other places that I don't think they can -- not substantiall, and not in the long run. They still live in the same physical and economic universe as everyone else, and their current startup operation methods aren't going to fly there ten or twenty years from now.

Anonymous said...

Tony said...

Re: Shuttle

The original sin was a philosophical belief in resuability without a corresponding engineering solution. Putting 120 tons in orbit, only to bring 100 of them back, was simply engineering nonsense then, and it still is now. It just costs too much to put stuff in space to bring anything back, except for the minimum RV needed to return humans and scientific samples. It doesn't seem like costs are coming down any time soon.



Anonymous said...

Oops, the anonymous above was me. I was filling out the captcha thing after quoting the part I wanted to respond to. I leave my reply here; please forgive my newness to commenting.

The problem with Tony analysis is that the 100 tons he mentions is only deadweight in the sense it isn't cargo. Those 100 ton existed to get stuff up and carry it safely down. Your not going to toss out the airframe or the engines and still be able to accomplish whatever mission the shuttle is supposed to do. I do agree in one sense though: the shuttle could have been good for ~110 tons of heavy lift it you engineered the orbiter out.

OrbitingPluto

Tony said...

OrbitingPluto:

"The problem with Tony analysis is that the 100 tons he mentions is only deadweight in the sense it isn't cargo. Those 100 ton existed to get stuff up and carry it safely down. Your not going to toss out the airframe or the engines and still be able to accomplish whatever mission the shuttle is supposed to do. I do agree in one sense though: the shuttle could have been good for ~110 tons of heavy lift it you engineered the orbiter out."

The only validated engineering justification for the Shuttle's reusability was to get the main engines back down to the ground for reuse. That's right -- the Shuttle existed because the Shuttle existed. Everything else that it's done could have been done with expendable launch vehicles, even if some of the payloads would have required capsules to return physical research results. Even the Hubble repair missions could have been accomplished with mission-specific, dockable jigs and fixtures.

Anonymous said...

Tony said...
"The only validated engineering justification for the Shuttle's reusability was to get the main engines back down to the ground for reuse. That's right -- the Shuttle existed because the Shuttle existed. Everything else that it's done could have been done with expendable launch vehicles, even if some of the payloads would have required capsules to return physical research results. Even the Hubble repair missions could have been accomplished with mission-specific, dockable jigs and fixtures."


I said "whatever mission the shuttle is supposed to do". Whether or not some other system could do the things Shuttle did ignores the fact Shuttle was supposed to be all things to all people. It was heavy lift(sorta). It was reusable(kinda). It created jobs in California(at least for a while). It was supposed to be cheap(big fail there). It kept NASA in business. That there isn't an engineering reason for Shuttle ignores the fact the Shuttle was tailored to political reasons. The 1000 mile crossrange and the 15x60 foot payload bay are examples of politically driven requirements that was built in. Did it make engineering sense? No. If you wish to deride the Shuttles, start with how politicians, bureaucrats and generals wrote up the specifications. Shuttle might not have existed in either the form we had or in any form at all without those three groups.

OrbitingPluto

Tony said...

OrbitingPluto:

"I said 'whatever mission the shuttle is supposed to do'..."

I would definitely agree with all of that. But I think it's also important to remember that the Shuttle, even without political enlargement, was a no-win proposition, from the engineering standpoint. You want to size it down to the original 10 ton payload capacity and eliminate the silly cross range requirements? Fine. The question would still be why place 50 tons in orbit, just to bring back 40 tons every time? In order to be competitive, a shuttle mission would have to cost no more than a fair-to-middlin Titan launch (about $75M in 1972).

Now I know they thought they could have had a launch rate to amortize development costs over a lot of flights, and keep costs down generally. But in reality? Not so much.

Anonymous said...

I'm really sorry to see the end of the Shuttle era; even though it wasn't what was promised. The original concept (before the politians got a hold of it), was for three different vehicles; one for scientific research, one for military missions, and one for hauling cargo. The shuttle as built was a combination of all three; a horse as designed by commitee. Some commenters here don't believe in reusable spacecraft, on principle, but I do think that vehicles for hauling people to and from orbit might be better as reusable. However, having said that, I also think that more developemnet in light-weight construction material and compact, powerful engines is needed. Others may dissagree with me, but I'm confedent (from the trends I see happening now), that over the next few decades, we will see significent advances in surface-to-orbit launch technology. I'm sure that the several companies that are pursuing this technology will have more losers than winners, and that it will take longer and more money then expected, but sooner than the birth of Captain Kirk, there should be modest vehicles that actually live up to the hype that the Shuttle didn't.

Ferrell

Byron said...

The shuttle should be a case study for engineering failures. The various participants lost sight of the true objective, which was not reusability. It was to lower the cost of space access on a regular basis. If they'd been on the ball, the final product would probably resemble the Falcon-9.

That said, the shuttle itself could have been done better. Assuming that we can toss the cross-range requirement, and that we still have to bring the engine back, the best thing to do is to split the shuttle. Put the crew in the front and the engines in the back in separate capsules, which reenter and land via paraglider. The stuff in the middle is mostly structure, which is quite cheap.

Of course, the really scary thing is that the shuttle program compares favorably with the various programs intended to replace it. Why not just replace NASA's manned spaceflight program? The last time they had real success was 1975.

The only part of a spacecraft that should be reusable is the part that must come back: the crew capsule. By definition it has to come back, and you might as well design it so you get more than one mission out of it.

Brett:
I wonder if some kind of orbital re-fueling capability would make renewal launched spacecraft much more viable. They would actually be able to slow down before re-entering into the atmosphere, reducing the impact of re-entry on the spaceship.
That's not the answer. Where is the fuel coming from? If the answer is Earth, then the cost to get it up far outweighs the saving from any potential reusability. If it's coming from elsewhere, we're a little far into the future for this.

Thucydides said...

6. "It can't be done." Consider the source. For them, it very likely could be. Cheaper ELV's likely cannot be provided by the traditional aerospace community.

Actually TRW did an engineering stury to consider how large the LEM engine could be scaled. Since there was no obvious upper limit (and a really huge engine was built by an industreal contractor to shipyard tolerences) there was a surge of interest in building low cost Big Dumb Boosters in the early 1970's.

Sadly, the customer (US Government) wasn't interested in the concept, so time and resources were shifted to what the customer was demanding (the Space Shuttle). Even if the true motivations were to employ a standing army fo NASA employees left over from the Apollo program, the customer is always right...

Byron said...

Thucydides:
Actually TRW did an engineering stury to consider how large the LEM engine could be scaled. Since there was no obvious upper limit (and a really huge engine was built by an industreal contractor to shipyard tolerences) there was a surge of interest in building low cost Big Dumb Boosters in the early 1970's.
The LEM engine was vacuum-optimized. It's not going to work well for a booster. The whole big, dumb booster thing seems like a good idea, but I'm skeptical. Given the relative costs of steel in space and electronics in space (virtually the same), the cost saving might not be that great.

Tony said...

Byron:

"The shuttle should be a case study for engineering failures. The various participants lost sight of the true objective, which was not reusability. It was to lower the cost of space access on a regular basis. If they'd been on the ball, the final product would probably resemble the Falcon-9."

Thucydides is actually right here. The customer requirement was not an efficient space transportation system from eveolved expendables. The customer requirement was reusability. Everything followed from that. The actual engineers didn't fail so much as they simply couldn't deliver better than what they did, with the technology available. As a budding aerospace engineer, you should really watch this entire series of MIT lectures, delivered by the people that were actually there:

http://ocw.mit.edu/courses/aeronautics-and-astronautics/16-885j-aircraft-systems-engineering-fall-2005/video-lectures/

Ferrell:

"Others may dissagree with me, but I'm confedent (from the trends I see happening now), that over the next few decades, we will see significent advances in surface-to-orbit launch technology. I'm sure that the several companies that are pursuing this technology will have more losers than winners, and that it will take longer and more money then expected, but sooner than the birth of Captain Kirk, there should be modest vehicles that actually live up to the hype that the Shuttle didn't."

Where are these advances going to come from? The Space Shuttle Main Engine already approached the maximum practical specific impulse and thrust-to-weight ratio for liquid fueled rockets. Airframe structures just aren't going to get any lighter. What trends do you see happening that's going to change any of that?

Byron said...

Tony:
Thucydides is actually right here. The customer requirement was not an efficient space transportation system from eveolved expendables. The customer requirement was reusability. Everything followed from that. The actual engineers didn't fail so much as they simply couldn't deliver better than what they did, with the technology available. As a budding aerospace engineer, you should really watch this entire series of MIT lectures, delivered by the people that were actually there:
True. I suppose I should have framed it better. The point is that it failed at its meta-objective, which was to bring down launch costs, by focusing on reusability. I'll have to go to the library and see if I can find if they did any studies on expendables vs the shuttle. And thanks for the link. Maybe I'll have time this weekend.

Where are these advances going to come from? The Space Shuttle Main Engine already approached the maximum practical specific impulse and thrust-to-weight ratio for liquid fueled rockets. Airframe structures just aren't going to get any lighter. What trends do you see happening that's going to change any of that?
Actually, the structures might improve somewhat. Materials is an often-overlooked field that has surprising potential. My understanding is that we're at the cusp of a real SSTO, but it's going to take a lot of money to make it work. And NASA would rather spend it on more paper rockets. But the actual rockets aren't going anywhere.

Tony said...

Byron:

"Actually, the structures might improve somewhat. Materials is an often-overlooked field that has surprising potential. My understanding is that we're at the cusp of a real SSTO, but it's going to take a lot of money to make it work. And NASA would rather spend it on more paper rockets. But the actual rockets aren't going anywhere."

We've been on that cusp for fifty years. The Atlas missile could actually put itself and a small payload in orbit, only having to discard a couple of engines to do it. But the structure had to be so light that it was a metal balloon, literally inflated -- with gaseous nitogent for storage, propellants for operations -- to maintain structural integrity. When it had to be depressurized for maintenance or repair, it had to be mechnically stretched on a special jig. (My dad, among his many other sins, edited the manual for the ground support equipment that did this particular task.) Of course, such a flimsy structure could never be recovered from orbit for reuse. No matter how small the payload, you could never reinforce it enough, much less provide it with a thermal management system for reentry.

So you would have to develop a material as light as a stainless steel balloon, yet as structurally rigid and thermally efficient as inconel. Barring miraclematerials science on the order of unobtainium, I'm just not going to hold out hope.

I will say this: SpaceX is on the right track in trying to recover the booster stage. That's where extra weight is least costly to the final payload mass, and where most of the structures and engines dollar cost is. But all that will be is a marginal improvement on cost -- and only if refurbishment costs can be kept down. You're still throwing away the upper stage(s) and payload fairing. But, if it costs as much to recover and refurbish a stage as it does to fly a new one, what's the point? And the cost of a refurbished booster stage would have to be real low, because most customers would readily pay a premium to get an entirely new rocket for their launch, if the cost delta between that and running a refurbished booster is less than (at a guess) 15-20%.

Byron said...

Tony:
We've been on that cusp for fifty years. The Atlas missile could actually put itself and a small payload in orbit, only having to discard a couple of engines to do it. But the structure had to be so light that it was a metal balloon, literally inflated -- with gaseous nitogent for storage, propellants for operations -- to maintain structural integrity.
I'm familiar with balloon tanks, and honestly wonder why we don't use them more. I was speaking specifically about a true spaceplane.

So you would have to develop a material as light as a stainless steel balloon, yet as structurally rigid and thermally efficient as inconel. Barring miraclematerials science on the order of unobtainium, I'm just not going to hold out hope.
Or, you know, engines more efficient than the 1950s-vintage LOX/kerosene ones on the Atlas. Say a LOX-LH2 aerospike?

I will say this: SpaceX is on the right track in trying to recover the booster stage. That's where extra weight is least costly to the final payload mass, and where most of the structures and engines dollar cost is. But all that will be is a marginal improvement on cost -- and only if refurbishment costs can be kept down. You're still throwing away the upper stage(s) and payload fairing. But, if it costs as much to recover and refurbish a stage as it does to fly a new one, what's the point? And the cost of a refurbished booster stage would have to be real low, because most customers would readily pay a premium to get an entirely new rocket for their launch, if the cost delta between that and running a refurbished booster is less than (at a guess) 15-20%.
Complete agreement here. The best part about tinkering with the first stage is that the stuff above it doesn't care, so long as it ends at the right altitude and velocity.

What we really need is for NASA (or someone who can give NASA orders) to get a bunch of engineers and tell them "We need a new space launch system. Design one that does the following as cheaply as possible over the next fifty years. Nothing is too crazy. Go." And then actually act on the conclusions reached. And I expect that Hell will freeze over well before that happens.
So, just for fun, if we were that panel, what would we look at? (Not what would be the best option, as we can't know that without a full study. What sorts of things might we look at?)

One thing that springs to mind is recovering only the engines. This was actually proposed for the Shuttle-C (unmanned cargo version).

Tony said...

Byron:

"I'm familiar with balloon tanks, and honestly wonder why we don't use them more."

I suspect there are practical limitations above a certain size. I'm pretty sure they abandoned the balloon tank for an isogrid tank in the Atlas V because the upper stages and payloads they wanted to stack were too heavy. Also, ground support for them is more complex.

"I was speaking specifically about a true spaceplane."

I was too, in a more oblique fasion. I was illustrating that we know how light the structures have to be, in order to put something in space on top of a single stage. We also know that those structures have long been within our grasp, at least for small payloads. When you start adding mass for recovery equipment, reusability goes out the window almost immediately, because you start requiring the structure to be more than it can be.

"Or, you know, engines more efficient than the 1950s-vintage LOX/kerosene ones on the Atlas. Say a LOX-LH2 aerospike?"

How massive do you suppose a LOX/LH2 engine has to be, in comparison to the rest of the launch vehicle, in order to provide enough thrust to get the vehicle off of the ground? Even with aerospike technology eliminating some nozzle mass and optimizing Isp? The problem is that you have to take the higher efficiency => lower thrust-to-weight tradeoff into account. That's why you wind up with LOX/hydrocarbon lower stages, or solid rocket strap-ons if you do put LOX/LH2 engines on the first stage. If you have a hydrogen engine powerful enough to get the vehicle off of the ground, the vehicle can't carry enough propellant to get anywhere. And none of that is susceptible to clever engineering.

In that respect, please note that the engines on the Atlas were conservative, not particularly inefficient. Yes, they developed considerably less specific impulse than modern hydrocarbon engines on the Atlas V, but they were also much lighter. The Atlas booster engines had only 248 sec sea-level Isp, compared to the modern engine's 313, but they also had a thrust-to-weight ratio of 120, whereas the modern hydrocarbon engine has only 77. And the lighter the structure, the more that engine weight eats up total vehicle weight.

"What we really need is for NASA (or someone who can give NASA orders) to get a bunch of engineers and tell them 'We need a new space launch system. Design one that does the following as cheaply as possible over the next fifty years. Nothing is too crazy. Go.' And then actually act on the conclusions reached. And I expect that Hell will freeze over well before that happens."

I suspect they would, after much study, come up with marginal improvements on current expendable designs. See, maybe NASA hasn't conducted a formal fifty year study of what works, but the space launch industry certainly has conducted an informal one, over the last fifty years. The results are what we see today.

"So, just for fun, if we were that panel, what would we look at? (Not what would be the best option, as we can't know that without a full study. What sorts of things might we look at?)

One thing that springs to mind is recovering only the engines. This was actually proposed for the Shuttle-C (unmanned cargo version)."


I'd recommend recovering the entire booster stage. And, to make it actually reusable without ridiculous amounts of refurbishment, I'd make that booster stage as large and inefficient as possible, consistent with recoverability. That would make both the structures and engines reconditioning for the next mission as inexpensive as possible. after all, the extra fuel that you would burn is the cheapest aprt of the whole enterprise.

Byron said...

Tony:
I suspect there are practical limitations above a certain size. I'm pretty sure they abandoned the balloon tank for an isogrid tank in the Atlas V because the upper stages and payloads they wanted to stack were too heavy. Also, ground support for them is more complex.
All true. I should see if there are any sources on balloon tanks. I love academic database access.

I was too, in a more oblique fasion. I was illustrating that we know how light the structures have to be, in order to put something in space on top of a single stage. We also know that those structures have long been within our grasp, at least for small payloads. When you start adding mass for recovery equipment, reusability goes out the window almost immediately, because you start requiring the structure to be more than it can be.
On the other hand, recovery system mass goes down for a system that has tanks in orbit. My understanding is that bigger tanks with more drag slow it down higher up where the air is thinner and the heating rate lower.

I suspect they would, after much study, come up with marginal improvements on current expendable designs. See, maybe NASA hasn't conducted a formal fifty year study of what works, but the space launch industry certainly has conducted an informal one, over the last fifty years. The results are what we see today.
No, because you miss the point of the study. The exact timeframe is irrelevant, so long as it's enough to have serious amortization of the research costs. A contractor in the space launch industry is not able to amortize development costs over fifty years.

I'd recommend recovering the entire booster stage. And, to make it actually reusable without ridiculous amounts of refurbishment, I'd make that booster stage as large and inefficient as possible, consistent with recoverability. That would make both the structures and engines reconditioning for the next mission as inexpensive as possible. after all, the extra fuel that you would burn is the cheapest aprt of the whole enterprise.
Something like the flyback Saturn stages that were proposed? Go with that, the cheapest possible second stage, and we're in business. Of course, if we make the second stage recoverable, we have something that works almost as well as an SSTO, and probably cheaper.

The S-IC on the Saturn V, for example, staged at 67km and 2.3 km/s. On the other hand, the winged S-IC was proposed as the first stage for the shuttle, but then the SRBs got the nod.

Anonymous said...

@Tony
I slept on it, and I've figured that you are right about hauling up stuff into orbit that is just going to have be brought back down. It was a no win.

Time to re-read what has been posted in the mean time.

OrbitingPluto

Anonymous said...

There are several new carbon materials that are light and strong; they do need development as far as lowering cost and efficency of manufacture, but that is where the time and money can come into play. As far as engines go, new high tempauture/high pressure engines made from lighter, stronger materials able to better resist higher tempuratures are being developed...in the aircraft industry; technology in the one field does have a way of moving into another. Composites have come a long way; we don't need as much heavy (or even light) metal in our spacecraft and rockets as we used to; those are the trends I'm seeing and those are the technologies I'm expecting to see continue to develop in the near future. I'm not saying that its going to happen overnight, and may well take decades to come into fruation, but these technological trends are real and people only need to look for them to find them; they are not hard to find, several articles have appeared on AOL news, after all. I'm very surprised that some people here haven't noticed that technology, especially materials technology, has moved forward a great deal since the 1950's...

Ferrell

jollyreaper said...

Looks like spam filter is binning comments across multiple posts.

Locki said...

Interestingly, I was randomly browsing a 2nd hand book shop last week and came across an old, old Arthur C Clarke novel where the need for thousands and thousands of data transfer satellites made mass production of rockets a necessity and space flight consequently relatively affordable. Clarke is, of course, famous for having totally missed predicting the internet.

Possibly one of the really big problems is there is no longer any pressing economic need to boost enormous numbers of satellites into orbit, therefore there is no economy of scale as envisaged by Space X.

Excluding military hardware how many satellites do we really need nowadays? Other than GPS satellites, the occassional weather satellite and a few satellite phones most of everything else is handled by the internet.

I have to admit I always presumed there was little more efficiency to squeeze out of chemical rockets because we have made practically no progress in 50 years. I've always noted the world has had 5 completely separate programs (Soviet/USA/European/Japanese/Chinese) operating for half a century and the costs and reliability are roughly comparable between all of them. I never bought into the Space X program.
I've always presumed to get space fight cheaper would take a technological leap. Where is the most likely place this is going to come from?

1. Are advances in material sciences going to be enough?

2. Air breathing scramjets?
From what I could understand of scramjets they are not the holy grail for SSTO. The problem is their efficiency is greatest at low altitudes whilst a traditional rocket gains in efficiency at high altitudes. The point at which these efficiencies cross over means you are probably just better off using a chemical rocket the whole way.

3. Is laser boosting really the next most likely technological leap that may open up space travel? Or is it equally as implausible as space elevators?

On a slight tangential issue. Is the gravity of Mars low enough to allow the astronauts to get back into orbit in a SSTO rocket? I was just doing some mind exercises and after reading about the intricate landing manouvres Curiosity required to get a mere 1000kg safely onto the ground it occurred to me this landing stage would also be the riskiest and most complex part of any manned mission. They would have to somehow safely land 50+ tonnes worth of volatile rocket fuel.

jollyreaper said...

Looks like spam filter is binning comments across multiple posts.


=========

Yeah its eaten a few of my posts. I'm now saving my posts in a word document in case my post is deleted 5-10 minutes later. You seem to be safe if its not taken down after about 5 minutes.

Rick said...

Re the spam filter: Nothing is in spam jail, so if posts are not showing up it must be some other problem. They may just be delayed in transit.


True in practice, given current operating constraints. Not so true in principle - which after all is the only thing they had to go by in the late 1960s when the 'pre-compromised' Shuttle concept was taking form.

It is true, for fundamental reasons, that reusable hardware to put a given payload in orbit will cost much more than expendable hardware. But, after all, the whole point of reusable hardware is that you get to reuse it, and amortize its cost over a fairly large number of launches.

The deal breaker has been launch operation cost: Checking out the vehicle, putting it on the pad, and sending it up. All of which is labor-intensive. And it cannot be amortized across multiple launches, since you need to do it each time.

Also, the thing about launch operation cost is that it is much harder to estimate in advance based on general principles. High school physics will give you a pretty good estimate of what basic physical characteristics a vehicle needs to get a payload into orbit. How much it will cost to launch, not so much.


And the toughest unknown to have predicted in advance, in the 1960s, was the public willingness to pay for space travel. The NASA budget, even in the Apollo era, was not much more than a rounding error relative to overall federal spending.

Whether your budget whine of choice is Medicare or the Pentagon, the amounts involved are at least an order of magnitude larger. In a slightly alternate history, the pre-compromised Shuttle might simply not have been controversial.

Something that has not gotten enough attention, IMHO, is that Apollo happened at just the same time as a particularly contentious era in American politics and culture, one that continues to reverberate.

jollyreaper said...

Oh so you have seen posts make it to the site and yet still disappear? I thought once I saw it on the site I was safe.

Anonymous said...

Locki:

Clarke is, of course, famous for having totally missed predicting the internet.

Just about everyone missed predicting the Internet

Possibly one of the really big problems is there is no longer any pressing economic need to boost enormous numbers of satellites into orbit, therefore there is no economy of scale as envisaged by Space X.

There never really was a need. There are about fifty launches per year worldwide. ULA production of Atlas and Delta rockets could easily be ramped up, but there is not enough demand.

Excluding military hardware how many satellites do we really need nowadays? Other than GPS satellites, the occassional weather satellite and a few satellite phones most of everything else is handled by the internet.

The bulk of satellites are stationary orbit communication satellites (as envisioned by Clarke). They are the backbone of worldwide communications. However, the planet still only has about fifty launches per year to support all space activity.

I never bought into the Space X program.

All SpaceX has to do to lower their launch cost is to make the first stage reusable and toss the rest of the stack. The Falcon 9 first stage has nine rocket engines while the second stage only has one. The first stage is the vast majority of the cost. You don’t have to make the whole thing reusable.

I've always presumed to get space fight cheaper would take a technological leap.

1. Are advances in material sciences going to be enough?


No, most of the mass of a chemical rocket is fuel.

2. Air breathing scramjets?


Scramjets are probably not going to work for SSTO. The Skylon has an innovative design that starts out as a jet and switches to a rocket. It could work, but it is very complicated and I wouldn’t be surprised if it doesn’t work.

3. Is laser boosting really the next most likely technological leap that may open up space travel?

I don’t think laser boosting will work for anything other than very small rockets. It will take extremely powerful lasers. It might work in space, but for getting to orbit.

On a slight tangential issue. Is the gravity of Mars low enough to allow the astronauts to get back into orbit in a SSTO rocket? I was just doing some mind exercises and after reading about the intricate landing manouvres Curiosity required to get a mere 1000kg safely onto the ground it occurred to me this landing stage would also be the riskiest and most complex part of any manned mission. They would have to somehow safely land 50+ tonnes worth of volatile rocket fuel.

Maybe, but you have found the big problem. How do you actually land something that large in the first place? There is some research into building landing pads out of local materials. You would send robots down first to build a large landing pad for your landing craft. If you don’t have a landing pad then the exhaust from the landing craft’s massive rockets kicks up dangerous debris while excavating a crater for you to crash in.

Ron

Jim Baerg said...

Reply to Locki:

On a slight tangential issue. Is the gravity of Mars low enough to allow the astronauts to get back into orbit in a SSTO rocket?

Yes. Though it's the speed needed to stay in orbit rather than the gravity which is the important factor 8 km/s for earth vs. 3.5 km/s for Mars.

I was just doing some mind exercises and after reading about the intricate landing manouvres Curiosity required to get a mere 1000kg safely onto the ground it occurred to me this landing stage would also be the riskiest and most complex part of any manned mission. They would have to somehow safely land 50+ tonnes worth of volatile rocket fuel.

That's a major reason Zubrin proposed making the fuel for the return trip on Mars. Save both the cost of sending the fuel there & the risk of landing it with the crew.

Tony said...

Ferrell:

"There are several new carbon materials that are light and strong; they do need development as far as lowering cost and efficency of manufacture, but that is where the time and money can come into play. As far as engines go, new high tempauture/high pressure engines made from lighter, stronger materials able to better resist higher tempuratures are being developed...in the aircraft industry; technology in the one field does have a way of moving into another. Composites have come a long way; we don't need as much heavy (or even light) metal in our spacecraft and rockets as we used to; those are the trends I'm seeing and those are the technologies I'm expecting to see continue to develop in the near future. I'm not saying that its going to happen overnight, and may well take decades to come into fruation, but these technological trends are real and people only need to look for them to find them; they are not hard to find, several articles have appeared on AOL news, after all. I'm very surprised that some people here haven't noticed that technology, especially materials technology, has moved forward a great deal since the 1950's..."

The problem, as always, isn't getting into orbit, but getting back, with the same vehicle. No material can manage to work in every environment that entails. Also, if you make the tankage light, you aerodynamically unbalance the vehicle. That's what ultimately killed VentureStar, which did have composite tankage -- they actually had to put tons of ballast in the nose so that it wouldn't be tail heavy in flight (because all of the wieght would toherwise be in the engines, at the extreme rear).

See, it's not the material technology so much as it is building a real-world vehicle that can do a real world task. Turns out that trumps pretty much every expected advantage of everything that's been developed in the last several decades.

Tony said...

Ron:

"All SpaceX has to do to lower their launch cost is to make the first stage reusable and toss the rest of the stack. The Falcon 9 first stage has nine rocket engines while the second stage only has one. The first stage is the vast majority of the cost. You don’t have to make the whole thing reusable."

The difficulty is making a reusable stage that is also cheaper to recover and recondition than it is to just fly a new one. And it has to be enough cheaper to attract customers who would otherwise pay a premium to fly a new stage. As Shuttle demonstrated, that's a non-trivial task against which to rank all non-trivial tasks. Good luck to 'em, but I ain't holdin' my breath.

Tony said...

Byron:

"No, because you miss the point of the study. The exact timeframe is irrelevant, so long as it's enough to have serious amortization of the research costs. A contractor in the space launch industry is not able to amortize development costs over fifty years."

Where in real world engineering does that ever happen? You get set of requirements, you look at the available technology, and away you go. On the other hand, it's not like research in materials, structures and power has been sitting still. That's why the Delta IV and Atlas V are essentially new vehicles. That's also why Space X, with a world of new materials and processes, and a clean design sheet, wound up with a very conventional rocket. Mature technology is mature technology, no matter how you cut it.

Rick said...

Forgot to say - Welcome to a new commenter!

Rick said...

By the way, 'True in practice,' in my post a bit upthread, was regarding the problems of reusable spacecraft.

Anonymous said...

Tony said:"The problem, as always, isn't getting into orbit, but getting back, with the same vehicle. No material can manage to work in every environment that entails. Also, if you make the tankage light, you aerodynamically unbalance the vehicle. That's what ultimately killed VentureStar, which did have composite tankage -- they actually had to put tons of ballast in the nose so that it wouldn't be tail heavy in flight (because all of the wieght would toherwise be in the engines, at the extreme rear).

See, it's not the material technology so much as it is building a real-world vehicle that can do a real world task. Turns out that trumps pretty much every expected advantage of everything that's been developed in the last several decades"

Congratulations Tony, you've just proved that it is logically impossible to bring anything down from orbit. Maybe you should stop using the term "real world" until you actually know what it means. Oh, and stop looking at the past during nearly every discussion thread and try to look forward sometimes, it makes for a better overall discussion, in my opinion.

Ferrell

Rob Lopez said...

I don't have anything to add to this subject, but I'd just like to say: Awesome site!!
And stunningly informed commenters.

I'd especially like to applaud jollyreaper's initial (and follow-up) post. I think I've just about managed to reach a level where some of that actually hit my brain rather than pass straight over my head.
It blew my mind.
And I learned something new.

I shall lie down now. Kudos to everyone and I look forward to reading more.

Geoffrey S H said...

Barring a miracle in new technology, there is no space future.

Unless you somehow get propellant cheap and powerful enough to lift a million tons into orbit every day, in incredibly light and strong rockets, there will be no progress IMHO. My own future history has no development in rocketry until the 2400's (first manned landing on mars- mid 25th century) when a "miracle" occurs with new technological developments.

A population capable of sending up far more astronauts compared to now is needed (a greater awareness of astro-dynamics, etc).A capability to live on and terraform MArs (so no living in cans on the surface with the horrendous psychological problems and sociological difficulties that come with that). That requires a lot of payload capacity for space-bound tail-sitters, with any asteroid mining that is more profitable than earth mining, and therefore viable needing a similar capacity. What else would get people into space? To get all that you need a miracle. You could say I've taken taken Zubrin's "we'll go into space on the backs of space conestogas" and agreed with that... with the caveat that his "we'll cobble together what we have now to achieve it" mantra is worse than nonsense.


If no techno-miracle then no space future beyond what we have now. Nothing.

Geoffrey S H said...

Oh, and I don't believe that this miracle will happen. I'm just really playing around with ideas in my little future history.

Tony said...

Ferrell:

"Congratulations Tony, you've just proved that it is logically impossible to bring anything down from orbit. Maybe you should stop using the term 'real world' until you actually know what it means."

Pure sophistry. I'm perfectly familiar with the real world. I live in it every day.

Of course you can bring something down from orbit, if it got there as a part of a bigger overall vehicle. But you can't, with any technology we know of, send a single, whole vehicle from the ground into orbit, then bring that same single, whole vehicle back.

"Oh, and stop looking at the past during nearly every discussion thread and try to look forward sometimes, it makes for a better overall discussion, in my opinion."

It's just my opinion, but I think you need to stop thinking like an enthusiast and start thinking like an engineer. Engineers need to know prior art to get anything done at all, because everything they have to work with today has a history. And everything new is not always the best choice. That's certainly SpaceX's opinion on some things. And it's also been proven with the Soyuz in comparison to the Shuttle. Sometimes slow and steady really does win the race.

Also, in evaluating progress, you actually have to have a history to compare to the present. The history of spaceflight suggests that the technology matured very quickly and has been susceptible to only margnial progress since. Going back to your original complaint, that is the real world.

jollyreaper said...

@rob

glad you enjoyed. Check out the older posts, plenty of gems there!

Anonymous said...

Tony, it really is easy to get you rilled; you're so grumpy and so enarmored of your own opinion that you can't bear to hear someone'else's that disagrees with your own; you go ballistic almost every time.

Anyway, as I said, there are trends in materials science that in several years, or a few decades, should see significantly lighter and stronger air/spaceframes; of all the present-day experiments in advanced propulsion, I'm sure that at least one will pan out (before the end of the century); the major unknown is interest and finance of spaceflight during the course of this century. Some of you seem to be cynical and pessimistic to an extreme level; I work with technology all the time and know its limits and the dependability of the people that create that technology. I think that 80+ years to develop a modest-capability reusable, commercial surface-to-orbit people-hauller isn't beyond the pale, no matter how fearful some of you seem to find that idea. :p (I'm still amazed that I use these things, it only due to my youngest daughter teaching me)

Ferrell

Locki said...

Maybe, but you have found the big problem. How do you actually land something that large in the first place? There is some research into building landing pads out of local materials. You would send robots down first to build a large landing pad for your landing craft. If you don’t have a landing pad then the exhaust from the landing craft’s massive rockets kicks up dangerous debris while excavating a crater for you to crash in.

Ron


==========================

Do people on this blog really think its possible for us to send astronauts to mars, land them, then launch them back into orbit and come all the way back to earth with our current level of technology?

Could we guarantee 90% survival, like the apollo missions, given the current state of rocketry?

There are just so many things that could go wrong.

Like I mentioned in my earlier post the whole curiosity mission just shows how hard it is to land a small object (1 lousy tonne) in the first place reliably.


I'm starting to form the opinion we can't really get people safely to Mars and back until there's a big jump in technology similar to the huge leap forward we saw going from propeller planes to jet/rocket planes.

Byron said...

Locki:
Do people on this blog really think its possible for us to send astronauts to mars, land them, then launch them back into orbit and come all the way back to earth with our current level of technology?

Could we guarantee 90% survival, like the apollo missions, given the current state of rocketry?

How much are you willing to spend? I believe the official NASA plan and estimate is on the order of $50 billion. This does not strike me as wildly unreasonable, either on the high end (bloated) or on the low end (impossible to stay on budget). This is with some tricks like Mars Direct, where most of the propellant for the return trip is produced on the Martian surface.

Locki said...

Byron said...

How much are you willing to spend? I believe the official NASA plan and estimate is on the order of $50 billion. This does not strike me as wildly unreasonable, either on the high end (bloated) or on the low end (impossible to stay on budget). This is with some tricks like Mars Direct, where most of the propellant for the return trip is produced on the Martian surface.


==============

Yeah I read about the Mars direct mission with interest.

I wonder if the huge increase in complexity won't introduce more points of failure than just boosting 50-ish tonnes of rocket fuel direct to mars.

For Mars direct you will need to produce something that can produce rocket fuel from basic elements/compounds.

Something we've never done before. Anywhere. Even with direct human supervision let alone being fully automated.

You will still have to land I guess 5-ish tonnes of rocket refinary and a light weight storage container that can handle the high pressue hydrogen.

Then you need to be able to transfer the highly volatile rocket fuel to your launcher safely. Presumably the astronauts will be doing this whilst wearing bulky spacesuits and after a year of microgravity to play havoc with their dexterity.

Build the launch pad.

If at any stage you have a failue in any of the above steps you have dead astronauts.

In comparison carrying all the fuel you need has only a single (large) point of failure,

Honsetly, $50 billion sounds cheap to go to Mars. When you consider the curiosity mission cost $2.5 billion to send a 1 tonne robot on a one way trip to Mars I'm pretty sure send live humans on a return trip is going to be astronomically higher rather than just 25 times the cost.

Anonymous said...

Locki:

Do people on this blog really think its possible for us to send astronauts to mars, land them, then launch them back into orbit and come all the way back to earth with our current level of technology?

No. NASA is talking about sending missions to Mars orbit in the 2030s. From orbit you can control rovers in real time and you can explore the Martian moons. That's worth the trip.

Ron

Tony said...

Ferrell:

"Tony, it really is easy to get you rilled; you're so grumpy and so enarmored of your own opinion that you can't bear to hear someone'else's that disagrees with your own; you go ballistic almost every time."

You have a strange idea of "ballistic". I don't cuss people out or even breathe heavily.

Speaking of what I think of my own opinion, I've said it before and I'll say it again: I'm far from the smartest person I know. But I'm a good student of what I endeavor to learn. If I keep returning to a line of opinion that fails to affirm what somebody else wants to believe, maybe it's because I've studied the subject thoroughly and know what I'm talking about.

"Anyway, as I said, there are trends in materials science that in several years, or a few decades, should see significantly lighter and stronger air/spaceframes;"

Could composites make lighter airframes? Yes. We already use them for that. Could they make lighter launch vehicle airframes that can launch and land in one piece? It's doubtful, simply because you couldn't make the airframe monotlithic. Light, strong composites aren't likely to absorb reentry heat while at the same time be able to adequately manage the cold of cryogenic fuels. Not in the same material. Even metal rockets have insulation applied to their tankage for safely and reliably handling cold fluids, while materials good at thermal management are either heavy and dense, like inconel, or ultra-light and structurally unsould, like Space Shuttle thermal tiles.

The difficulty is that you're running up against basic physics. No one material is competent in every environment, and launch vehicle reusability requires the management of three distinct environments -- launch, space, and atmospheric reentry. Composites may make it possible to lighten launch vehicle tankage somewhat. (But to what degree is questionable, given that they are likely to become brittle at very low temperatures.) But enable single-piece reusable spacecraft? It doesn't seem likely.

"of all the present-day experiments in advanced propulsion, I'm sure that at least one will pan out (before the end of the century);"

Which advanced propulsion would that be? The SSME was already running up against the theoretical limits of liquid propellants, thirty years ago. What else are you going to use? Nuclear? Not politically palatable (for good technical reasons, not just hysteria) and harder to get high thrust out of than was once thought. Electric rockets? We're talking about getting off of the Earth, not going from orbit to orbit.

Are you perhaps talking about scramjets or hybrids? Maybe, but just because people are spending money on research and development doesn't mean it will ever pan out. In my educated estimation, relying on those to work out effectively and economically is seriously betting the come. I'll eat my words if it happens, but my knife and fork are still sitting in the drawer, so to speak.

"the major unknown is interest and finance of spaceflight during the course of this century. Some of you seem to be cynical and pessimistic to an extreme level; I work with technology all the time and know its limits and the dependability of the people that create that technology. I think that 80+ years to develop a modest-capability reusable, commercial surface-to-orbit people-hauller isn't beyond the pale, no matter how fearful some of you seem to find that idea. :p (I'm still amazed that I use these things, it only due to my youngest daughter teaching me)"

See, this is what you're so not getting. I would love to be wrong about the limits of technology. But the more I study space technology, the less sure I am that reusability was ever a practical idea, or ever will be. In my studied and carefully considered opinion, I'm just facing facts.

Byron said...

Locki:
Honsetly, $50 billion sounds cheap to go to Mars. When you consider the curiosity mission cost $2.5 billion to send a 1 tonne robot on a one way trip to Mars I'm pretty sure send live humans on a return trip is going to be astronomically higher rather than just 25 times the cost.
Admittedly, I'm not sure how long ago that number was from. At a guess, between inflation and cost growth, it would be more like $100 billion today.

I wonder if the huge increase in complexity won't introduce more points of failure than just boosting 50-ish tonnes of rocket fuel direct to mars.
People probably said the same thing about lunar orbit rendezvous. The problem is that to send that 50 tons to Mars, you have to first get it off Earth, which means that you need about 200 tons of rockets. Oh, and the actual estimate for propellant is more like 112 tons, of which 7% comes from Earth.

For Mars direct you will need to produce something that can produce rocket fuel from basic elements/compounds.

Something we've never done before. Anywhere. Even with direct human supervision let alone being fully automated.

Actually, the reaction is quite old and well-characterized. And IIRC they've already built prototypes.

Build the launch pad.
The LM didn't use one.

If at any stage you have a failue in any of the above steps you have dead astronauts.
The same is true of any spaceflight. The fuel is generated before the astronauts, and the generator is directly attached to the Mars Ascent Vehicle. Oh, and they generate gravity via a tether and the spent booster.

No. NASA is talking about sending missions to Mars orbit in the 2030s. From orbit you can control rovers in real time and you can explore the Martian moons. That's worth the trip.
No, it's not. Running the robot from close up would be a lot better, sure, but not enough better. We might get twice the science out of a given rover, but we could cancel the humans and send 10 more rovers instead.

jollyreaper said...

We didn't have a moon rocket when we set the goal of going to the moon. When doing what has never been done before, you have to invent whole new fields of technology. Part of the guessing game is determining whether we are yet capable of such research in the first place.

If we decided we needed to get to mars ASAP, I don't doubt we could. It would be expensive and difficult and cost loads of money but if we wanted to we could. The question is whether we could do so in an economically reasonable way. I doubt we would have been on a manned trip to the moon abscent the Cold War.

I think one of the real kickers for a mars mission is getting a better interplanetary rocket so there's not do much time spent in space. Just dropping the consumables from 2 years to a month there and back and a month on the ground would change everything. Of course, then the question is if we even have an inkling on how such an engine could be built. The Germans had a good idea on how a moon rocket could be built and were correct.

There's a whole black art in looking at what we can currently do and guessing at where we could go next given money and drive. Sometimes all the pieces are laying there waiting to be picked up, sometimes only half are there but only one guy has the vision of what the missing pieces should look like.

Brett said...

I think Zubrin tends to be a little optimistic about how easy some things would be on Mars Direct, but the idea of producing fuel on site is sound . . . if you can get the right equipment there. It's not easy landing a 40-ton spacecraft on Mars, and IIRC the nuclear reactor he needs for the mission has a power density that doesn't exist yet.

Byron said...

Brett:
I think Zubrin tends to be a little optimistic about how easy some things would be on Mars Direct, but the idea of producing fuel on site is sound . . . if you can get the right equipment there. It's not easy landing a 40-ton spacecraft on Mars, and IIRC the nuclear reactor he needs for the mission has a power density that doesn't exist yet.
Zubrin is insanely optimistic. A year or so ago, I saw something he wrote in the Wall Street Journal. It was a Mars mission for about a billion. At the same time, the figures I was quoting for Mars Direct were from a full technical study, and NASA has adopted a variant of it for their mission. That removes most of the doubt from my mind that it can be done.

Tony said...

Byron:

"between inflation and cost growth, it would be more like $100 billion today."

At least.

"People probably said the same thing about lunar orbit rendezvous. The problem is that to send that 50 tons to Mars, you have to first get it off Earth, which means that you need about 200 tons of rockets. Oh, and the actual estimate for propellant is more like 112 tons, of which 7% comes from Earth."

Lunar Orbit Rendezvous (LOR) in fact wound up shaping the entire space program through Apollo. For example, they wouldn't have screwed around with rendezvous and docking experiments on Gemini if the Direct Ascent (DA) option had been chosen for Apollo. And LOR required a much smaller launch vehicle than the Nova LV that would have been used with DA.

Now, with 20/20 hindsight DA seems absolutely ludicrous in terms of both launch and lunar landing vehicle size. But it probably could have been done, had the technical risks of LOR proved too great. I think for Mars the same point can be made -- whatever would have to be done would be done. Heck, if we could get the Shuttle to work, we can probably do anything that requires no more than brute force.

"Actually, the reaction is quite old and well-characterized. And IIRC they've already built prototypes."

Zubrin likes to refer to it as "Gaslamp Era" technology. And yes, benchtop prototypes that demonstrate the reaction have been made and operated. The trick is to take it off the bench and turn it into flight hardware.

"The LM didn't use one."

To be pedantic, the LM brought it's own launch pad, in the form of the descent stage. Also, there was serious concern about getting a level landing of the descent stage. Even if the ascent stage could have achieved a tilted launch as an exercise in sheer ability to operate, it's guidance program could handle only so much of a departure from local veritcal. Too much of an angle and it just wouldn't be able to put the LM in a stable and predictable orbit.

"The same is true of any spaceflight."

Indeed. Even in everyday commercial air travel, there are single point failure modes. If the wing breaks, death, no saving throw. Complaining about that is pointless.

"The fuel is generated before the astronauts, and the generator is directly attached to the Mars Ascent Vehicle."

Clearing that up a little bit, the ascent and return vehcles are sent ahead on a previous opportunity. They have a year and a half on the ground to make all the necessary propellant before the manned element ever leaves the Earth. IOW, the program knows it has a good, fully fuelled return vehicle before it sends people.

Of course, achieving a ground rendezvous on Mars is still going to be a neat trick, but only in the snese that it's never been done on Mars. Apollo 12 ground rendezvoused with Surveyor 3on the Moon way back in 1969.

Finally -- and this is admittedly a semantic quibble -- I think the propellant generator would be bolted on to the Mars descent stage of the ascent and return vehicles. No point lugging it off the surface of Mars after it's job is done.

"Oh, and they generate gravity via a tether and the spent booster."

Or they just live with the free fall aspect of the flight and hope that Mars gravity is enough to keep a human healthy, at least for a couple of years. Health risks are part of the cost of the ticket for pioneers. They always have been.

Tony said...

Byron:

"No, it's not. Running the robot from close up would be a lot better, sure, but not enough better. We might get twice the science out of a given rover, but we could cancel the humans and send 10 more rovers instead."

I think it would be more than ten times the science, because driving and the positioning of sampling equipment could be real-time controlled from Mars orbit. That's a big time saver. The limiting factors would then become the rover's power supply and the ability of the science team on Earth to quickly turn analyze results and make decisions for further investigation.

But the real attraction of sending humans is actually getting them on the surface. As Steve Squyres, the principal investigator for MER has pointed out, a good geologist on the secene can get as much work done in five minutes as a rover can do in a whole day.

Byron said...

Tony:
Zubrin likes to refer to it as "Gaslamp Era" technology. And yes, benchtop prototypes that demonstrate the reaction have been made and operated. The trick is to take it off the bench and turn it into flight hardware.
I really doubt that will be a significant challenge (not that it will be easy, but it's unlikely to put the whole program at risk).

To be pedantic, the LM brought it's own launch pad, in the form of the descent stage. Also, there was serious concern about getting a level landing of the descent stage. Even if the ascent stage could have achieved a tilted launch as an exercise in sheer ability to operate, it's guidance program could handle only so much of a departure from local veritcal. Too much of an angle and it just wouldn't be able to put the LM in a stable and predictable orbit.
I was replying to Locki, who said that the crew would have to build the launch pad. As for slope, that's relatively easy. Mount a lidar on the lander, and tie it into the landing autopilot. If something goes wrong, the crew swap some science gear for stuff to restore it to a proper angle.
The more I look at it, the more the advantage of having the return hardware already at Mars is a great idea.

Of course, achieving a ground rendezvous on Mars is still going to be a neat trick, but only in the snese that it's never been done on Mars. Apollo 12 ground rendezvoused with Surveyor 3on the Moon way back in 1969.
Not worried in the slightest. Curiosity was 2.4 km off-target. With a little bit of work, and a much better target, we should see significant improvements.

Finally -- and this is admittedly a semantic quibble -- I think the propellant generator would be bolted on to the Mars descent stage of the ascent and return vehicles. No point lugging it off the surface of Mars after it's job is done.
Absolutely. It might even come in handy if we were for some reason to return to the same site.

Or they just live with the free fall aspect of the flight and hope that Mars gravity is enough to keep a human healthy, at least for a couple of years. Health risks are part of the cost of the ticket for pioneers. They always have been.
On the trip out and back? There's no reason not to tether oneself to the booster and spin the thing. Under most plans, the entire vehicle will land on Mars, which means the structural mass penalty will be negligible.

Byron said...

I think it would be more than ten times the science, because driving and the positioning of sampling equipment could be real-time controlled from Mars orbit. That's a big time saver. The limiting factors would then become the rover's power supply and the ability of the science team on Earth to quickly turn analyze results and make decisions for further investigation.
I am going to maintain that putting people in Mars orbit solely to run the rovers is a bad idea from a cost standpoint, to say nothing of operational issues. Given the costs to develop the equipment to do so, it's cheaper to send rovers. If we take $100 billion as our cost, we could send 40 Curiosities for one manned mission. In all reality, we could probably do quite a bit better if we mass-produced the rovers. On the other hand, how much of that 100 billion would be saved by not landing? Call it a wash. Later manned missions would be significantly cheaper (maybe $10 billion).
Operationally, it will probably take at least four people to operate one rover efficiently. Given how many rovers we could send for the cost of not sending those people (and how many places we could go), there's no reason to do so. That's not to say that NASA shouldn't set it up so that the Mars team could control any rovers if they have time, but it's certainly not a valid mission driver. And as you said, the science team will still be back on Earth.
If for some reason, they do fly astronauts to Mars orbit rather than landing them (as our illustrious President proposes) then rover-driving would salvage some value from it. IMO, the differing circumstances suggest that check-out missions are not feasible.

But the real attraction of sending humans is actually getting them on the surface. As Steve Squyres, the principal investigator for MER has pointed out, a good geologist on the secene can get as much work done in five minutes as a rover can do in a whole day.
Exactly. Plus, the humans will be bringing back samples, which do far more than any rover. I know that sample-return has been proposed, but the Lunar sample-return missions were not nearly as useful as the Apollo samples.

While we're talking about Mars, here's a thought. Why don't we build the Mars hab as a giant, slow-moving rover. If they've got six months, and move a few miles a day, then they cover a lot of ground in reasonable depth. There would obviously be a smaller rover in case of problems, but this avoids a lot of the issues of transit to interesting sites.

Byron said...

Sorry. Above I should have said that check-out missions are not a good idea. Unlike on the Moon, you can't check out everything to 90% of the way to the surface. And in that case, why not just orbit the Earth for a year or so instead? Most of the hardware will get checked on the unmanned missions to land the return vehicle and hab/rover.

Tony said...

Byron:

Please understand, I'm not trying to be pessimistic, I'm just pointing out that there are technical risks to every proposed initiative. Think of it as for-the-sake-of-completeness-izing.

That should about cover most of the points under discussion, WRT the exceptions:

"On the trip out and back? There's no reason not to tether oneself to the booster and spin the thing. Under most plans, the entire vehicle will land on Mars, which means the structural mass penalty will be negligible."

The booster isn't going to land on Mars. It happens to be going the same way after trans-Mars orbit insertion. Also, if current practice is maintained, the intitial orbit will not be designed to intersect Mars, because we don't want random rocket parts raining all over the planet.

But my real problem with the tethered booster artificial gravity is the mission risk involved if there's a tether failure. It would impart an unplanned-for vector in some random direction that might be mission or even crew-safety compromising. That has to be balanced against the health effects of several months of microgravity living on the way out and on the way back. It may just be an extra risk that isn't worth the reward.

"I am going to maintain that putting people in Mars orbit solely to run the rovers is a bad idea from a cost standpoint..."

I would agree, as far as that goes. but it also depends on what the actual manned mission modes are. If one of the modes calls for a manned orbital insertion without landing, plus a significantly long "winter-over", then controling rovers from Mars orbit is a valid mission objective, and even a desirbale capability.

"If for some reason, they do fly astronauts to Mars orbit rather than landing them (as our illustrious President proposes) then rover-driving would salvage some value from it. IMO, the differing circumstances suggest that check-out missions are not a good idea. Unlike on the Moon, you can't check out everything to 90% of the way to the surface. And in that case, why not just orbit the Earth for a year or so instead? Most of the hardware will get checked on the unmanned missions to land the return vehicle and hab/rover."

I'm not as down on Flexible Path as a lot of people. The incrementalism may upset a lot of people who have grown up with certain expectations about what interplanetary exploration means, but to me it's just a prudent, learn-as-you-go approach, with minimal additional technological risk at each stage. Mars orbit, for example, is both technologically less challenging and requires less delta-v than going all the way to the surface and coming back. And it would certainly work out interplanetary space flight technologies at least as well as a landing mission. Amd a Mars flyby, on a free return trajectory, should probably be tried before that. Even Apollo, working under time pressure that simply doesn't exist for a manned Mars exploration program, went the orbit first route. (Yes, I know that Apollo 8 used a previously unplanned alternate mission mode, but Apollo 10 did fly and orbital mission without a landing.)

"While we're talking about Mars, here's a thought. Why don't we build the Mars hab as a giant, slow-moving rover. If they've got six months, and move a few miles a day, then they cover a lot of ground in reasonable depth. There would obviously be a smaller rover in case of problems, but this avoids a lot of the issues of transit to interesting sites."

Once again there's a tradeoff between technoligcal risk and reward. It sounds like a good advanced mission idea, but not for the first couple of landings, at least.

Byron said...

Tony:

The booster isn't going to land on Mars. It happens to be going the same way after trans-Mars orbit insertion. Also, if current practice is maintained, the intitial orbit will not be designed to intersect Mars, because we don't want random rocket parts raining all over the planet.

But my real problem with the tethered booster artificial gravity is the mission risk involved if there's a tether failure. It would impart an unplanned-for vector in some random direction that might be mission or even crew-safety compromising. That has to be balanced against the health effects of several months of microgravity living on the way out and on the way back. It may just be an extra risk that isn't worth the reward.

The booster is by definition going to be on the same trajectory as the capsule is when it burns out. There is no reason not to slam it into mars (if we have seismographs on the ground, it might even help) and the advantages gained by not having the crew be weightless for several months are substantial.
I don't see worries about tether failure as a major issue. Obviously, whoever designs it will do so in a manner that ensures it is unlikely to break. There are quite a few things I'd be more worried about.

I would agree, as far as that goes. but it also depends on what the actual manned mission modes are. If one of the modes calls for a manned orbital insertion without landing, plus a significantly long "winter-over", then controling rovers from Mars orbit is a valid mission objective, and even a desirbale capability.
Sure, but I don't see any particular mission mode that would have that result. If there is one, by all means, do it.

I'm not as down on Flexible Path as a lot of people. The incrementalism may upset a lot of people who have grown up with certain expectations about what interplanetary exploration means, but to me it's just a prudent, learn-as-you-go approach, with minimal additional technological risk at each stage. Mars orbit, for example, is both technologically less challenging and requires less delta-v than going all the way to the surface and coming back. And it would certainly work out interplanetary space flight technologies at least as well as a landing mission. Amd a Mars flyby, on a free return trajectory, should probably be tried before that. Even Apollo, working under time pressure that simply doesn't exist for a manned Mars exploration program, went the orbit first route. (Yes, I know that Apollo 8 used a previously unplanned alternate mission mode, but Apollo 10 did fly and orbital mission without a landing.)
The problem is that we gain nothing beyond "deep space operational experience" from anything but the Mars landing in Flexible Path. Nothing. And analogies to Apollo break down when you look at it. They had never gone very far out before, and were reasonably safe if anything went wrong. It was entirely possible for Apollo 8 to go, spend 20 hours in Lunar orbit, and come back. You can't do that with Mars. Sending half a dozen people out for 18 months, a third of which is spent sitting around Mars driving rovers, doesn't seem like a good plan. The hardware can be checked in Earth orbit (or to some extent on the Lunar surface) just as well, and if something should go wrong, the crew is far safer. There is nothing particularly special about the deep space environment.
The missions to the Lagrange points make even less sense. Most people don't understand what those are, and there is nothing to see there. The only Lagrange points worth visiting are Jupiter's.

Once again there's a tradeoff between technoligcal risk and reward. It sounds like a good advanced mission idea, but not for the first couple of landings, at least.
Just something I came up with.

Tony said...

Byron said...
Tony:

Byron:

"The booster is by definition going to be on the same trajectory as the capsule is when it burns out. There is no reason not to slam it into mars (if we have seismographs on the ground, it might even help) and the advantages gained by not having the crew be weightless for several months are substantial."

I'm just reporting the facts. Current robotic missions, with much smaller trans-Mars injection stages, are intentionally launched on non-intercept trajectories, in order to avoid contaminating Mars with rocket junk. The trajectory is later modified to affect a Mars atmosphere intercept. I don't see that changing, even with humans in the mix. Several manned Moon landing missions were initiated without a free return trajectory, in order to achive other mission objectives besides absolute safety.

"I don't see worries about tether failure as a major issue. Obviously, whoever designs it will do so in a manner that ensures it is unlikely to break. There are quite a few things I'd be more worried about."

Like I said, it's a balance of risk vs reward. I'm just not a sanguine as you appear to be to dismiss that particular risk.

"Sure, but I don't see any particular mission mode that would have that result. If there is one, by all means, do it."

Mars orbit insertion without a landing is a mission mode that would have that result. It happens to be one of the mission modes currently under serious consideration as a Flexible Path option.

"The problem is that we gain nothing beyond 'deep space operational experience' from anything but the Mars landing in Flexible Path. Nothing."

I know it doesn't seem like much, from a certain point of view, but just the experience of sending people somewhere we haven't sent them before and getting them back is valuable. Otherwise, why bother with Mercury and Gemini? They didn't go all the way out to the Moon, so by your definition, they were worthless. But of course they weren't, because they tested technologies and procedures for later, more ambitious missions.

"And analogies to Apollo break down when you look at it. They had never gone very far out before, and were reasonably safe if anything went wrong. It was entirely possible for Apollo 8 to go, spend 20 hours in Lunar orbit, and come back."

And if the Service Propulsion System had failed in boosting the spacecraft out of Lunar orbit? Or if the comms system had failed at some point between the Earth and the Moon? The Apollo astronauts were dependent (as were the Shuttle astronauts, and every other astronaut, ever, and probably for some time to come) upon ground tracking for navigation and guidance. The tracking observations made use of the spacecraft communications system signals.

"You can't do that with Mars. Sending half a dozen people out for 18 months, a third of which is spent sitting around Mars driving rovers, doesn't seem like a good plan. The hardware can be checked in Earth orbit (or to some extent on the Lunar surface) just as well, and if something should go wrong, the crew is far safer. There is nothing particularly special about the deep space environment."

Except actually having done it. I really don't understand why people deprecate accomplishment, unless it's the maximum, ultimate accomplishment that they wish to see. Can you explain that to me?

"The missions to the Lagrange points make even less sense. Most people don't understand what those are, and there is nothing to see there. The only Lagrange points worth visiting are Jupiter's."

People don't really understand the point of the ISS all that well either -- and not just the disinterested general public; plenty of interested, well-informed spectators wonder as well. It's still an accomplishment. (Albeit one that doesn't take us anywhere, and is growin long in the tooth, even for it's admittedly limited objectives.)

Brett said...

That Zubrin essay reminded me of an introductory speech I saw him give for a space conference, where he ran through his whole slide set for Mars Direct for the millionth time. It really seemed sad and dated, which is bad since it's an interesting idea.

Brett said...

@Tony
Zubrin likes to refer to it as "Gaslamp Era" technology. And yes, benchtop prototypes that demonstrate the reaction have been made and operated. The trick is to take it off the bench and turn it into flight hardware.

Good point. Aside from the stresses it will take during the flight, you'll also got the problem in that we just don't know if there will be any issues we haven't foreseen due to a combination of the Martian environment and the significantly lower gravity. You can create a simulated Martian environment in a closed space here on Earth, but simulating the gravity of Mars is much harder. You'd probably have to have a bunch of them sent to Mars first for testing, before you send out the ones that the astronauts would ultimately use.


@Byron

I wonder if you could "harden" an AI or advanced computer, park it in orbit around Mars, and then just use that to control a bunch of rovers on the surface simultaneously using data from the rovers and from other satellites put in Mars orbit to observe the surface. Assuming you can make that happen with the orbiting computer, it would probably be cheaper than trying to have humans control stuff from orbit.

The problem is that we gain nothing beyond "deep space operational experience" from anything but the Mars landing in Flexible Path. Nothing.

What about the ability to visit Near-Earth Asteroids? That would actually be more useful in a commercial sense than going to Mars.

Byron said...

Tony:
I'm just reporting the facts. Current robotic missions, with much smaller trans-Mars injection stages, are intentionally launched on non-intercept trajectories, in order to avoid contaminating Mars with rocket junk. The trajectory is later modified to affect a Mars atmosphere intercept. I don't see that changing, even with humans in the mix. Several manned Moon landing missions were initiated without a free return trajectory, in order to achive other mission objectives besides absolute safety.
So what if not hitting Mars with the stage is an objective? Again, it really doesn't matter. The advantages to being able to work in gravity are likely to be substantial, and the penalties are likely to be minor. It costs a couple dozen m/s, and the tether, plus a little bit of structure.

Like I said, it's a balance of risk vs reward. I'm just not a sanguine as you appear to be to dismiss that particular risk.
The technology dates back to the 60s. Gemini 11 attached a tether to their Agena. Why is it such a risk? For that matter, what about the risks to crew health from six months of zero-G before landing? If somebody breaks a bone, it's a year before they can get to a hospital.

Mars orbit insertion without a landing is a mission mode that would have that result. It happens to be one of the mission modes currently under serious consideration as a Flexible Path option.
Let me rephrase that. I see no pressing need to use that particular mission mode. What is so important about Mars orbit?

And if the Service Propulsion System had failed in boosting the spacecraft out of Lunar orbit? Or if the comms system had failed at some point between the Earth and the Moon? The Apollo astronauts were dependent (as were the Shuttle astronauts, and every other astronaut, ever, and probably for some time to come) upon ground tracking for navigation and guidance. The tracking observations made use of the spacecraft communications system signals.
The question I have is this: What scientific objectives do we have that can be served only by men in Mars orbit? Why should we go, except to test hardware? If that's all we're going to do, what can we not do just as well closer to home?

I know it doesn't seem like much, from a certain point of view, but just the experience of sending people somewhere we haven't sent them before and getting them back is valuable. Otherwise, why bother with Mercury and Gemini? They didn't go all the way out to the Moon, so by your definition, they were worthless. But of course they weren't, because they tested technologies and procedures for later, more ambitious missions.
The problem is that you're confusing place and environment. Mercury and Gemini went into environments we'd never been in before. So did Apollo. But nobody went to sea to sail to a spot in the open ocean that nobody had been to before. (Except oceanographers, but the analogy breaks down there. We have no need to survey deep space.) To put it another way, what will going to a Lagrange point show? Nobody doubts our capability to do so, and we have no immediate need to.

Byron said...

Except actually having done it. I really don't understand why people deprecate accomplishment, unless it's the maximum, ultimate accomplishment that they wish to see. Can you explain that to me?
The problem is that the drawbacks outweigh the benefits. First off, any Mars orbit test is going to take at least six months off the development schedule. The launch windows are about every two years, and an earth orbital test could launch six months later. Second, the relative risks for a Mars test are much higher than for a Lunar test. Something like Apollo 13 would probably not be survivable on a Mars mission. The key point is that you can come home from Luna any time you like. If something goes wrong, you're three days out. There are lots of problems that can be dealt with for that long, but would be fatal over a year. So you're increasing the risk to the crew, not gaining any new data, and putting more pressure on the schedule. Also, new hardware would have to be built for this mission, which drives up the cost.
A useful mission for hardware checkout is as follows. The capsule goes up and flies out past Luna. It stays in orbit for six months. A box between the radio antenna and mission control records everything that comes in, and plays it out to the other side a few minutes later, depending on where they are in the sim. After the first six months, they land on the Lunar farside, and a satellite relays for them. Six months later, they lift off and repeat the first six months again.
This checks out surface operations procedures and produces useful science, as well as having better abort options in an emergency. And it does just as well in human-factor tests, which is the only other thing that a Mars mission would be useful for. It would require some hardware modifications, but probably no more than Mars orbit (because, IIRC, you'd be doing direct descent, so you need a booster to stop in Mars orbit.)

People don't really understand the point of the ISS all that well either -- and not just the disinterested general public; plenty of interested, well-informed spectators wonder as well. It's still an accomplishment. (Albeit one that doesn't take us anywhere, and is growin long in the tooth, even for it's admittedly limited objectives.)
I understand that ISS has a point. I can grant that a Mars orbital mission has a point as a testbed for landings (though I remain unconvinced that it's a good idea). But why would we go to a Lagrange point? I don't know. Except that we can, and NASA hopes that "Lagrange Point" sounds cool enough to buy it political support. Or that it will get squashed so they can do more studies.

Byron said...

Brett:
Good point. Aside from the stresses it will take during the flight, you'll also got the problem in that we just don't know if there will be any issues we haven't foreseen due to a combination of the Martian environment and the significantly lower gravity. You can create a simulated Martian environment in a closed space here on Earth, but simulating the gravity of Mars is much harder. You'd probably have to have a bunch of them sent to Mars first for testing, before you send out the ones that the astronauts would ultimately use.
The one they're going to use will have been operating for about a year before they launch. I don't see a big problem.

I wonder if you could "harden" an AI or advanced computer, park it in orbit around Mars, and then just use that to control a bunch of rovers on the surface simultaneously using data from the rovers and from other satellites put in Mars orbit to observe the surface. Assuming you can make that happen with the orbiting computer, it would probably be cheaper than trying to have humans control stuff from orbit.
It would absolutely be cheaper. In fact, why not just install the AI on the rover. (Raw processing power does not seem to be the key to AI, so power and weight are not the issue. And the money saved on operations would be available to add the hardware.) However, it requires AI we don't have yet.

Byron said...

Brett:
What about the ability to visit Near-Earth Asteroids? That would actually be more useful in a commercial sense than going to Mars.
That's not such a bad idea. (And I know that it's in the plan. I sort of forgot earlier.) On the other hand, we would actually be landing. My biggest problem is with the suggestion to go to the Lagrange Points. I have yet to hear a reason to go there, except that man has never been. The gaping hole in that reason is that man has also never been to 99.9% of the lunar surface. Claiming that we've been there is like landing in half a dozen spots in Central Africa for two days each, and claiming that you've been to Africa. On one level, true, but you haven't explored it. And there's a lot more to do there then there is at a piece of empty space that is of primary interest only to astrodynamicists, who have no need for field tests. (Others do use it for its properties, but doing space exploration that way is akin to letting the oceanographers run the Age of Discovery.) If we happen to pick a Lagrange point as suitable for something like my near-earth checkout mission, then do so, but "go to a Lagrange point" has about as much scientific purpose as "go to 0,0".

Tony said...

Byron:

"So what if not hitting Mars with the stage is an objective? Again, it really doesn't matter. The advantages to being able to work in gravity are likely to be substantial, and the penalties are likely to be minor. It costs a couple dozen m/s, and the tether, plus a little bit of structure.

The technology dates back to the 60s. Gemini 11 attached a tether to their Agena. Why is it such a risk? For that matter, what about the risks to crew health from six months of zero-G before landing? If somebody breaks a bone, it's a year before they can get to a hospital."


The initial non-intercept trajectory is a mission design requirement that is not susceptible to analysis. It's driven by a political policy of avoiding needless contamination of Mars with Earth-originated space junk. So it can't be avoided or significantly modified.

As for prior art, the failure of the Gemini-Agena tether, at .00015 g, wouldn't have been very likely, nor would it have significantly altered either spacecraft's orbit. The failure of a tether on an interplanetary transfer trajectory, at a rotational velocity sufficient to impart .4 or higher g, would be much more likely, and could cause a significant -- and perhaps unrecoverable -- change in trajectory. I'm not saying its an absolutely dangerous proposition, but the downside has to be investigated with care, not simply dismissed out of hand.

"Let me rephrase that. I see no pressing need to use that particular mission mode. What is so important about Mars orbit?"

It ain't Earth or Lunar orbit. It demonstrates a previously undemonstrated capability. At the same time, it's less risky than a landing mission, and requires less delta-v. It's simply a significant accomplishment.

"The question I have is this: What scientific objectives do we have that can be served only by men in Mars orbit? Why should we go, except to test hardware? If that's all we're going to do, what can we not do just as well closer to home?"

It's not all about scientific objectives. Just getting into Mars orbit and returning with a live crew is a significant technical objective, both for engineering and program advancement reasons. I simply can't see not doing it first.

"The problem is that you're confusing place and environment. Mercury and Gemini went into environments we'd never been in before. So did Apollo. But nobody went to sea to sail to a spot in the open ocean that nobody had been to before. (Except oceanographers, but the analogy breaks down there. We have no need to survey deep space.) To put it another way, what will going to a Lagrange point show? Nobody doubts our capability to do so, and we have no immediate need to."

Nothing actually in space is a place. Earth is a place. The Moon is a place, Mars is a place. Everything else is just orbits. I admit that a Lagrange outpost doesn't demonstrate much WRT propulsion or orbital operations in general. But it does afford the opportunity to demonstrate operation of manned craft outside of the Van Allen belt for a long duration, while retaining a relatively immediate and straightforward bail-out capability.

Tony said...

Byron:

"The problem is that the drawbacks outweigh the benefits. First off, any Mars orbit test is going to take at least six months off the development schedule. The launch windows are about every two years, and an earth orbital test could launch six months later. Second, the relative risks for a Mars test are much higher than for a Lunar test. Something like Apollo 13 would probably not be survivable on a Mars mission. The key point is that you can come home from Luna any time you like. If something goes wrong, you're three days out. There are lots of problems that can be dealt with for that long, but would be fatal over a year. So you're increasing the risk to the crew, not gaining any new data, and putting more pressure on the schedule. Also, new hardware would have to be built for this mission, which drives up the cost.
A useful mission for hardware checkout is as follows. The capsule goes up and flies out past Luna. It stays in orbit for six months. A box between the radio antenna and mission control records everything that comes in, and plays it out to the other side a few minutes later, depending on where they are in the sim. After the first six months, they land on the Lunar farside, and a satellite relays for them. Six months later, they lift off and repeat the first six months again.
This checks out surface operations procedures and produces useful science, as well as having better abort options in an emergency. And it does just as well in human-factor tests, which is the only other thing that a Mars mission would be useful for. It would require some hardware modifications, but probably no more than Mars orbit (because, IIRC, you'd be doing direct descent, so you need a booster to stop in Mars orbit.)"


A couple of things...

First off, you're presuming something like a Mars Direct technological pathway as the design reference mission for the entire program. That's not what Flexible Path is about. Flexible Path is about doing something, then doing something else, then doing something else, all on a base of general-purpose hardware. An interplanetary hab should be an interplanetary hab, regardless of whether th mission goes to a Lagrange point, a NEO, or Mars, and also regardless of whether a surface excursion takes place. A Mars surface lander and a surface stay capability are an addition to that, not an integral part.

Is it the most efficient way to go to Mars, specifically? No, of course not. Is it a good way to build a long-term, sustainable program, with minimal up-front development cost? You betcha. And I'm for doing what can be done within reasonable funding expectations, rather than holding out for the perfect mission that never gets funded.

Secondly, surface operations on the Moon don't tell you much about surface operations on Mars. There's no atmosphere, the gravity is different, the surface composition is different, the light/dark cycle is different, the insolation is different, etc. "We can test on the Moon" is simply a straw man.

"I understand that ISS has a point. I can grant that a Mars orbital mission has a point as a testbed for landings (though I remain unconvinced that it's a good idea). But why would we go to a Lagrange point? I don't know. Except that we can, and NASA hopes that 'Lagrange Point' sounds cool enough to buy it political support. Or that it will get squashed so they can do more studies."

I sometimes question NASA institutional motives myself. But it remains a fact that a long, manned Lagrange stay is a capability extender.

"The one they're going to use will have been operating for about a year before they launch. I don't see a big problem."

Presuming it actually is operating on the surface. The point I was making isn't about problems with the specific process, but in making a machine to do that process that's reliable in an unattended state for years at a time. That's an all-of-spaceflight concern.

Tony said...

Byron:

"If we happen to pick a Lagrange point as suitable for something like my near-earth checkout mission, then do so, but 'go to a Lagrange point' has about as much scientific purpose as 'go to 0,0'."

To reiterate, it's not about scientific objectives, it's about technical objectives. To play in your ballpark for a moment, when determination of longituded was an iffy proposition, going to 0,0 would have been -- indeed was -- a significant technical achievement, relevant to both the safety and efficiency of navigation. Similar developpmental tests have been a part of spaceflight all along as well.

So please, let's stop deprecating field tests of developmental hardware and procedures, as if they don't mean anything. They do mean something -- often a lot.

Byron said...

Tony:
The initial non-intercept trajectory is a mission design requirement that is not susceptible to analysis. It's driven by a political policy of avoiding needless contamination of Mars with Earth-originated space junk. So it can't be avoided or significantly modified.
That's not what I meant. The penalties for changing course later on (a few weeks out, say), as opposed to early in the mission, are probably fairly minimal.

As for prior art, the failure of the Gemini-Agena tether, at .00015 g, wouldn't have been very likely, nor would it have significantly altered either spacecraft's orbit. The failure of a tether on an interplanetary transfer trajectory, at a rotational velocity sufficient to impart .4 or higher g, would be much more likely, and could cause a significant -- and perhaps unrecoverable -- change in trajectory. I'm not saying its an absolutely dangerous proposition, but the downside has to be investigated with care, not simply dismissed out of hand.
That is true, but what is the chance of failure? It's a big, high-tech rope. I'd classify it breaking as a very low-risk scenario.

It ain't Earth or Lunar orbit. It demonstrates a previously undemonstrated capability. At the same time, it's less risky than a landing mission, and requires less delta-v. It's simply a significant accomplishment.
My problem is that NASA is substituting "significant accomplishment[s]" for any sort of sustainable plan. If we're going to go to Mars orbit, we should have a good reason, and I'm still not convinced that we really need to. The Vikings got to America before anyone else. And Denmark therefore rules the world, right?

It's not all about scientific objectives. Just getting into Mars orbit and returning with a live crew is a significant technical objective, both for engineering and program advancement reasons. I simply can't see not doing it first.
The problem is that if we do lots of things for Program Advancement, we end up with some red rocks to match our grey set, and my grandchildren are in the same place we are today.

Nothing actually in space is a place. Earth is a place. The Moon is a place, Mars is a place. Everything else is just orbits. I admit that a Lagrange outpost doesn't demonstrate much WRT propulsion or orbital operations in general. But it does afford the opportunity to demonstrate operation of manned craft outside of the Van Allen belt for a long duration, while retaining a relatively immediate and straightforward bail-out capability.
OK, but why not park in Lunar orbit instead? The exact same thing applies, but we might be able to do some science as well.

Byron said...

First off, you're presuming something like a Mars Direct technological pathway as the design reference mission for the entire program. That's not what Flexible Path is about. Flexible Path is about doing something, then doing something else, then doing something else, all on a base of general-purpose hardware. An interplanetary hab should be an interplanetary hab, regardless of whether th mission goes to a Lagrange point, a NEO, or Mars, and also regardless of whether a surface excursion takes place. A Mars surface lander and a surface stay capability are an addition to that, not an integral part.
I understand the purpose of Flexible Path just fine. It's to give a neat-sounding space program that gives us lots of cool 'firsts' without costing a lot of money. If we have everything but the landing hardware for Mars, and we know it works, then there is no need to fly to Mars to test it.
And for some reason, Lunar missions are out. Maybe because Bush liked them. Maybe because they aren't exciting enough. I don't know. But we need to have a rational reason for a space program. Is it to learn about the cosmos? Then we should be looking at more Lunar landings. Learning to survive and use the resources of space? Ditto. Go where no man has gone before? OK, now Flexible Path makes sense. And remind me why it's so vital that we get footprints on Mars?

Secondly, surface operations on the Moon don't tell you much about surface operations on Mars. There's no atmosphere, the gravity is different, the surface composition is different, the light/dark cycle is different, the insolation is different, etc. "We can test on the Moon" is simply a straw man.
No, but they tell us a bunch about surface operations in general. Yes, the gravity is different, but there is still up and down. The atmosphere is different, but we're still going to need spacesuits and airlocks. And a lot of the hardware will be the same. A spacesuit built for Mars should handle Luna just fine, though the cooling system might have to be modified. The hab should be largely the same. We can debug probably 75% of the hardware on Luna, not to mention techniques, and get useful science at the same time. And with some simple computers, we can replicate the communications challenges of Mars. Also, biomedical data. We have two data points for the effects of gravity on humans: 0 and 1. If we get a third, it at least tells us something for Mars.
I guess the question is this. Why is deep-space experience so important, but hostile surface experience isn't?

Byron said...

I sometimes question NASA institutional motives myself. But it remains a fact that a long, manned Lagrange stay is a capability extender.
The same could be said of any stay outside LEO. Around the moon, say. Or even on the moon. Then we at least get some science.

Presuming it actually is operating on the surface. The point I was making isn't about problems with the specific process, but in making a machine to do that process that's reliable in an unattended state for years at a time. That's an all-of-spaceflight concern.
It's called engineering. Occasionally, I'm told, people go to college to study it. I think may even know a few people who have done so.
Modern satellites go unattended for years, too. The thruster systems and such are probably of a similar complexity to the reactor. It will take some work, but what Brett said was that unexpected interactions between gravity and atmosphere might develop. I doubt that, and if they do, they should show up before the astronauts go.

To reiterate, it's not about scientific objectives, it's about technical objectives. To play in your ballpark for a moment, when determination of longituded was an iffy proposition, going to 0,0 would have been -- indeed was -- a significant technical achievement, relevant to both the safety and efficiency of navigation. Similar developpmental tests have been a part of spaceflight all along as well.

So please, let's stop deprecating field tests of developmental hardware and procedures, as if they don't mean anything. They do mean something -- often a lot.

I understand that, and I came on a bit strong at the beginning. My problem is mostly with NASA scheduling a Lagrange point mission now, under a flimsy rationale. A Lagrange mission should be scheduled when the hardware is well along, and they're going 'OK, we need to do a deep-space test. Why not go to a Lagrange point?' Apollo was not sent 'to Lunar orbit, and then to the Lunar surface' it was sent to the Lunar surface, and Apollo 8 was a necessary step along the way.
NASA seems to be trying to use it as a thing they can point at and say 'Look what we did'. Which is not a reason to go into space.

Tony said...

Just a few things, to bring this back on focus...

1. The risks with a tether are not just the tether itself, but the attachments, deployment and recovery, and spin-up and spin-down, to name just a few of the most obvious ones. It's added risk that doesn't need to be courted, except for an overwhelming and mission-critical reason to have artifical gravity. Maybe that reason exists, but I wouldn't assert it as an established fact, without careful analysis.

2. The reason to put lunar landing missions off to the side of the critical path is that they've been done. They demonstrate nothing new. Flexible Path missions, even something as simple as a Lagrange point stay, do in fact demonstrate new capabilities. I really don't understand how anyone can characterize this as a punt -- it certainly beats more indefinite years of LEO thumb-twiddling.

3. I have to strongly disagree that being able to claim an accomplishment is not a good enough reason. It's really the only reason that makes sense from a financing point of view. You can make a claim to scores and hundreds of billions in order to be first and best. More science? Not so much.

Before anybody goes there, Apollo was no different. It was sold as a demonstration of technical achievement and national will. The science was either necessary to enable the technical goals -- you gotta know something about the Moon to land and operate on it -- or just because once the astronauts got there, it was a margianl additional cost to get a science return.

Byron said...

Tony:
1. The risks with a tether are not just the tether itself, but the attachments, deployment and recovery, and spin-up and spin-down, to name just a few of the most obvious ones. It's added risk that doesn't need to be courted, except for an overwhelming and mission-critical reason to have artifical gravity. Maybe that reason exists, but I wouldn't assert it as an established fact, without careful analysis.
I agree on the careful analysis part, but I really think you overestimate the potential problems. Still, that's not for either of us to decide.

2. The reason to put lunar landing missions off to the side of the critical path is that they've been done. They demonstrate nothing new. Flexible Path missions, even something as simple as a Lagrange point stay, do in fact demonstrate new capabilities. I really don't understand how anyone can characterize this as a punt -- it certainly beats more indefinite years of LEO thumb-twiddling.
But the simple fact is that from a technical standpoint, a Lunar landing is far more outside our capabilities than is a Lagrange point mission. Admittedly, I would prefer Flexible Path to doing virtually nothing in LEO. On the other hand, the fact that something has previously been done does not change its intrinsic value. Luna is vital to any sustained space presence, and if the technology is developed to be intentionally dual-role (as far as practical), doing it and Mars together is the cheap way. There was no reason that Apollo couldn't have gone to a Lagrange point. They just didn't.

3. I have to strongly disagree that being able to claim an accomplishment is not a good enough reason. It's really the only reason that makes sense from a financing point of view. You can make a claim to scores and hundreds of billions in order to be first and best. More science? Not so much.
And doing this for national pride means it won't happen this time. We don't have an enemy to motivate us. It will run over budget, be cancelled, and we'll have this discussion again in five years. I'm hopeful that my kids won't still be doing this when they're my age, which means we need to get it right.

Before anybody goes there, Apollo was no different. It was sold as a demonstration of technical achievement and national will. The science was either necessary to enable the technical goals -- you gotta know something about the Moon to land and operate on it -- or just because once the astronauts got there, it was a margianl additional cost to get a science return.
Yes. And Flexible Path is the budget version of that. At least they figured out that they weren't going to get Apollo budgets again. The problem is that people aren't going to be excited by human missions to empty space. We connect to people on the ground.

We will probably go to the moon again when nobody in power remembers Apollo. Not before.

Anonymous said...

Tony, your incessant pessimism, aggressive attacks, and general negative outlook on every thing that I or anyone else says about anything has really lowered the amount of pleasure that I derive from this site; in fact, I've almost quit visiting the site simply because of your negativaty. You're such an ass, Tony; why do you even write on this blog if all you're going to do is attack others and tear down (or attempt to), every other commenters' posts. And the worst of it is, is that you're so enarmored of your own opinion that you can't seperate them from actual facts. You need help, and I'm sorry that you've decided to inflect yourself on us.

Ferrell

Rick said...

Welcome to a new commenter! Possibly more than one - I'm still catching up to this thread.

To a question I saw in passing, 'do people on this blog believe X,' the answer probably is 'some do.' I have my own biases, expressed unabashedly in front-page posts. But laziness usually triumphs over the impulse to impose them on the commenter community.

Anonymous said...

Rick, I'm sorry, but this isn't fun anymore; maybe I'll see you in a few months. Goodbye.

Ferrell

Brett said...

@Byron
It would absolutely be cheaper. In fact, why not just install the AI on the rover. (Raw processing power does not seem to be the key to AI, so power and weight are not the issue. And the money saved on operations would be available to add the hardware.) However, it requires AI we don't have yet.

I figured that we could give the AI more equipment and capabilities to work with if we kept it parked in orbit, without putting it through atmosphere re-entry on Mars. If it has real-time or near-real-time control of the surface rovers, then you don't really need them to be too smart anyways.

@Ferrell
Rick, I'm sorry, but this isn't fun anymore; maybe I'll see you in a few months. Goodbye.

Oh, don't be like that. Even when I don't understand the discussion, or think someone is being a jerk, I still keep on reading. I've never understood this whole "storming off into the dark night of the internet" thing that people do.

Byron said...

Ferrell:
Your last post did come off as sort of rude, and Tony has always been kind of direct. And I'm in agreement with him on the fundamental point. SSTO is really hard (I'm on the fence WRT impossible) and we have no reason to believe that the next few years will get better.
And while I've been annoyed by him many times, your ad hominem is uncalled for.

Brett:
I figured that we could give the AI more equipment and capabilities to work with if we kept it parked in orbit, without putting it through atmosphere re-entry on Mars. If it has real-time or near-real-time control of the surface rovers, then you don't really need them to be too smart anyways.
The problem is that AI is a big question mark. We don't know anything about what it will look like. That said, I doubt that the requirements will be so large that there will be significant savings by keeping it in orbit. If it's just improved software (probably derived from autonomous vehicle research) it should be no trouble at all to incorporate into the rovers own computer. After all, why can an AI naturally handle multiple rovers?

jollyreaper said...

Crosspost from slashdot:

http://tech.slashdot.org/comments.pl?threshold=-1&mode=flat&commentsort=0&op=Change&sid=3155857&cid=41513819&pid=41513819

There is at least one 3D printing company that I know of that offers 'printing' in brass, bronze and titanium.

They're using a very old and well known technique, the lost wax - but the wax is printed with the 3D printer, and then the metal poured into the mold.

This is not only an amazing evolution on an existing technology, but because the final products aren't built up layer by layer, they're structually equivalent to anything coming out of a foundry.

The ability to print custom tools, gears and moving parts in titanium is incredible.


I would also concur with another point brought up in the thread, the first home computers were fairly useless things, of little interest to anyone outside of hobbyists. They had little practical use but much potential. It would have taken a very, very far-sighted visionary to look at the Altair and see a precursor to the smartphone.

I feel 3D manufacturing is in that same, primitive state. Much, much potential.

Tony said...

Byron:

"I agree on the careful analysis part, but I really think you overestimate the potential problems. Still, that's not for either of us to decide."

I was just pointing out the technical and mission risk, because a lot of people (not necessarily you) don't consider that kind of thing.

"But the simple fact is that from a technical standpoint, a Lunar landing is far more outside our capabilities than is a Lagrange point mission."

It is? We've done it already. It's outside our current equipment set. But we know how to do it and have demonstrated the capability repeatedly. The only roadblock to doing it again is specific funding.

"Admittedly, I would prefer Flexible Path to doing virtually nothing in LEO. On the other hand, the fact that something has previously been done does not change its intrinsic value."

Say that out loud. Flexible Path is better than doing LEO over and over again, but having done something previously does not reduce its value.

"Luna is vital to any sustained space presence,"

It depends on what that presence is and how the infrastructure is built up. It also depends on whether a sustained presence -- anti sustained effort, which is a different thing -- is considered feasible and worthwhile.

"and if the technology is developed to be intentionally dual-role (as far as practical), doing it and Mars together is the cheap way."

Not much dual-use to be had there, really. A hab on Mars can produce oxygen from the surrounding atmosphere. Producing oxygen on the Moon is a bit of a different proposition. Pressure suits for Mars have to be optimized for a different environment as well. Hardware reliability is in a whole 'nuther ballpark where near-immediate bailout isn't a possibility.

There was no reason that Apollo couldn't have gone to a Lagrange point. They just didn't."

Actually, aside from propulsion, going to a Lagrange point for any length of time with Apollo hardware would have involved unacceptable crew risk. The solar radiation and cosmic ray risk outside of the Van Allen belt was acceptable for a couple of weeks on Apollo. So they just didn't design to mitigate it. If we want to send troops out there for weeks or months at a time, it's going to be a real challenge, even today.

"And doing this for national pride means it won't happen this time. We don't have an enemy to motivate us. It will run over budget, be cancelled, and we'll have this discussion again in five years. I'm hopeful that my kids won't still be doing this when they're my age, which means we need to get it right."

We did Shuttle for thirty years essentially for reasons of national pride, ISS as well. The whole point of Flexible Path is to do something new, in a reasonable time frame, within a realistic manned space budget. There's much more hope for that than a big bang Mars or Moon program.

"Yes. And Flexible Path is the budget version of that. At least they figured out that they weren't going to get Apollo budgets again. The problem is that people aren't going to be excited by human missions to empty space. We connect to people on the ground."

Not quite. Flexible Path is about going to new places. A Lagrange point mission isn't the ultimate destination. It's just a place to validate interplanetary technology in an interplanetary environment. Nobody is selling it as the ultimate objective, just like nobody sold Apollo 9 as the ultimate use of the LM.

"We will probably go to the moon again when nobody in power remembers Apollo. Not before."

And hopefully a long time after that. The Moon has a certain romance for people of my generation, but it's really a dsitraction from interplanetary exploration.

Byron said...

Tony:
It is? We've done it already. It's outside our current equipment set. But we know how to do it and have demonstrated the capability repeatedly. The only roadblock to doing it again is specific funding.
So is Luna a worthy goal for anyone outside the US, or is it just not a good thing anymore?
My point is that at the moment we do not have the capability to do either. The fact that we previously had the capability is irrelevant. And a Lunar landing is far harder technically than is a Lagrange mission. We would only find it marginally easier to re-do Apollo than would anyone else, as much of the documentation is publicly available, and what isn't is (probably) not terribly relevant.

Say that out loud. Flexible Path is better than doing LEO over and over again, but having done something previously does not reduce its value.
And what is the intrinsic value of LEO missions? I really can't see any beyond gathering data to support farther operations. Which we've already done.

It depends on what that presence is and how the infrastructure is built up. It also depends on whether a sustained presence -- anti sustained effort, which is a different thing -- is considered feasible and worthwhile.
I'm assuming that we want a sustained space presence. Yes, it's an assumption. I (and most everyone here) agree that it is desirable, for reasons including the survival of our species.

A hab on Mars can produce oxygen from the surrounding atmosphere. Producing oxygen on the Moon is a bit of a different proposition.
Point. On the other hand, that's one element of the life-support system. Oh, and it's part of the ISRU fuel system, so it gets tested anyway.

Pressure suits for Mars have to be optimized for a different environment as well.
Care to elaborate?


Actually, aside from propulsion, going to a Lagrange point for any length of time with Apollo hardware would have involved unacceptable crew risk. The solar radiation and cosmic ray risk outside of the Van Allen belt was acceptable for a couple of weeks on Apollo. So they just didn't design to mitigate it. If we want to send troops out there for weeks or months at a time, it's going to be a real challenge, even today.

Apollo was designed as a general-purpose spacecraft. There were serious missions for things like a manned Venus flyby. (Why, I'm not sure.) They didn't seem terribly worried about the radiation risk. Or maybe they could, say, replace the LM with a hab/shelter?

We did Shuttle for thirty years essentially for reasons of national pride, ISS as well. The whole point of Flexible Path is to do something new, in a reasonable time frame, within a realistic manned space budget. There's much more hope for that than a big bang Mars or Moon program.
That's fine. I can accept that Flexible Path makes the most sense in the environment that it was planned in. That doesn't mean that the environment is a good one. Why is novelty so important? If we justify it that way, NASA gets some red rocks it can put into the vault beside the grey ones, and we sit here for another few decades.

Not quite. Flexible Path is about going to new places. A Lagrange point mission isn't the ultimate destination. It's just a place to validate interplanetary technology in an interplanetary environment. Nobody is selling it as the ultimate objective, just like nobody sold Apollo 9 as the ultimate use of the LM.
No, but why was it listed in the destinations in Obama's speech? A hardware check mission is one thing. This is something else. Going for milestones is not a good plan.
As for "budget Apollo" I meant in the sense of going somewhere new. That is exactly what it is.

And hopefully a long time after that. The Moon has a certain romance for people of my generation, but it's really a dsitraction from interplanetary exploration.
That's what I meant. You view it as a distraction. I view it as an objective.

Tony said...

Byron:

"So is Luna a worthy goal for anyone outside the US, or is it just not a good thing anymore?"

As a demonstration of manned space program competence? It's a worthy goal for anybody that hasn't done it before.

For somebody that's been there and done that? The response -- within the US as much as anywhere else -- would be, "That's nice, but you need to show me something new." It's simple human nature.

"My point is that at the moment we do not have the capability to do either. The fact that we previously had the capability is irrelevant. And a Lunar landing is far harder technically than is a Lagrange mission. We would only find it marginally easier to re-do Apollo than would anyone else, as much of the documentation is publicly available, and what isn't is (probably) not terribly relevant."

Shuttle was in many ways a technically much more robust accomplishment than Apollo. Using Shuttle technology -- which is what we are doing -- it should be no problem to put men back on the Moon. Yes, the reconstitution of an abandoned technological capability is far from trivial, but it still isn't demonstrating something new. That's why the Moon is off to the side of the Flexible Path. It just doesn't have the attraction of building advanced capabilities.

"And what is the intrinsic value of LEO missions? I really can't see any beyond gathering data to support farther operations. Which we've already done."

But that's essentially what you're advocating for the Moon -- do it because it will bear fruit in the future, not because it's extending our capabilities into new areas of application now.

"I'm assuming that we want a sustained space presence. Yes, it's an assumption. I (and most everyone here) agree that it is desirable, for reasons including the survival of our species."

In the abstract, I agree with you. In the concrete, it just doesn't seem feasible anytime soon. IMO it will have to grow organically out of a sustained effort, as it becomes necessary. That was essentially how the Mir space station developed out of Almaz/Salyut. Doing things for their philosophical goodness just isn't the way to operate in space, as Shuttle demonstrated.

"Point. On the other hand, that's one element of the life-support system. Oh, and it's part of the ISRU fuel system, so it gets tested anyway."

How does ISRU of a palpable (even if extremely low pressure) Martian atmosphere get tested on the Moon?

"Care to elaborate?"

It's that Martian atmopsphere again. Even at 1% of Earth atmosphere pressure, it's enough to cause serious convection cooling of the suit. It's also the increased distance from the Sun, decreasing the impact of insolation. So you have to design the suit differently than you would design it for a vacuum environment with twice the insolation.

"Apollo was designed as a general-purpose spacecraft. There were serious missions for things like a manned Venus flyby. (Why, I'm not sure.) They didn't seem terribly worried about the radiation risk. Or maybe they could, say, replace the LM with a hab/shelter?"

They would have had to use a shielded hab. Now that was understood and in the palnning stages, but it wasn't beyond the conceptual design phase, much less bent metal.

Tony said...

Byron:

"That's fine. I can accept that Flexible Path makes the most sense in the environment that it was planned in. That doesn't mean that the environment is a good one."

It's the realistic environment that has to be lived with. Whenever I have this conversation, I always come to the conclusion that people are complaining about crackers because they aren't cookies, never considering that fact that at least they're getting something rather than nothing.

"Why is novelty so important? If we justify it that way, NASA gets some red rocks it can put into the vault beside the grey ones, and we sit here for another few decades."

Novelty is important because that's what the people will pay for. It needs no further justification than that.

WRT the end result, I'm just not that interested in what the philosophical goal should be if it can't be achieved in a realistic technical and economic sense. Flexible Path is something we can do, right now, with the technology we possess. Maybe something bigger will grow out of it. Maybe nothing bigger will come from it. But it's still doing something, right now, rather than waiting for a perfect storm that may never come. That's enough of a bottom line for me.

"No, but why was it listed in the destinations in Obama's speech? A hardware check mission is one thing. This is something else. Going for milestones is not a good plan."

Because it is a major milestone. And milestones demonstrate progress. Were the HMS Warrior and USS Monitor insignificant simply because they weren't dreadnoughts? Was Henry Ford's assembly line insignificant simply because it wasn't William Knudsen's more advanced GM version, that built all those tanks and airplanes to win WW2?

"As for 'budget Apollo' I meant in the sense of going somewhere new. That is exactly what it is."

And that has value, because it generates more dollars to go to even newer places.

"That's what I meant. You view it as a distraction. I view it as an objective."

It's an achieved objective. Recapitulating achieved objectives is a distraction. That's why the Apollo program, just within it's own context, kept shooting for higher latitudes, more constricted landing zones, longer surface stays, longer traverses, etc. Within the context of manned spaceflight, the Moon is as done as equitorial landings and one or two days on the surface was done after Apollo 14.

Geoffrey S H said...

Anyone notice that whenever we talk about green/purple, our discussions are constructive and well thought out, but anything not to do with war just dissolves into squabbling and "you're too negative/positive" arguments? Anyone notice the irony in this?

For the record, my own post above may have seemed somewhat trollish/ extreme, so I apologize. But can we please try and be civil for goodness sake? Tony's "negativity" can be used for very constructive purposes, and I can think of historical arguments to back some of his points up. Everyone here has something useful to say, and if we use everything here as a model for our stories and future histories, then we will end up with something far richer and complex than if we stuck to our own view points or selectively used a few others. It might even predict the future more accurately, you never know...

Tony said...

Geoffrey S H:

"...but anything not to do with war just dissolves into squabbling and "you're too negative/positive" arguments?"

War, especially space war in some not-too-carefully-defined future, is sufficiently abstract to avoid engaging strong emotions. When you're talking about our own real technological future, you're talking about closely held dreams and expectations. And sometimes being realistic about that is in fact a direct attack on those dreams and expectations. So I can understand the "pessimism, aggressive attacks, and general negative outlook" that Ferrell perceives. I personally think that's a highly subjective judgment, based on a desire not to hear -- and especially not to engage -- what seems to me, after years of study and consideration, just plain fact. YMMV.

Brett said...

I used to be like that. Then I went through a period of pessimism where I thought it was all pointless - space travel would always be too expensive for anyone other than the government and a few rich space tourists, and it would largely evaporate once the robots got good enough anyways. Now I'm slightly more optimistic again about what we might be able to do, although the challenges are pretty daunting.

It's sad, though, to think that something like a manned Mars mission is probably a pipe dream during my lifetime. Imagine how someone like Zubrin, who has dedicated a big chunk of his life to it and even influenced NASA, feels. It's especially maddening because the actual amount of money it would take for Awesome Space Stuff seems so little compared to other expenditures - just doubling NASA's budget would be a massive boon to manned and unmanned space exploration.

Byron said...

Brett:
It's sad, though, to think that something like a manned Mars mission is probably a pipe dream during my lifetime. Imagine how someone like Zubrin, who has dedicated a big chunk of his life to it and even influenced NASA, feels. It's especially maddening because the actual amount of money it would take for Awesome Space Stuff seems so little compared to other expenditures - just doubling NASA's budget would be a massive boon to manned and unmanned space exploration.
I would say that maddening is exactly the word in his case. He simply wants it too bad. On the other hand, doubling NASA's budget would probably waste money at about twice the current rate, and we really wouldn't get anywhere. The management there prefer to do studies, and were not happy when Kennedy handed them Apollo.

Tony:
So I can understand the "pessimism, aggressive attacks, and general negative outlook" that Ferrell perceives.
I can't understand the part about aggressive attacks. Looking over the various exchanges, you were probably more civil than I would have been in your place. I'm not going to debate the pessimism and negative outlook part, of course. We've clashed over that often enough. It would be less fun if we all agreed.

Tony said...

Byron:

"I can't understand the part about aggressive attacks. Looking over the various exchanges, you were probably more civil than I would have been in your place."

As unfortunate as it is that it upsets Ferrell so much, I can really understand his anger. I'm basically going on, and on, and on telling him that this is the future, and it doesn't get much better -- not without magitech. That's really a blow to a lot of people's dreams and expectations.

So I get the lashing out in frustration. I'm sure Ferrell wants me to recant, and tell him everything's going to be fine. Since I won't, it's all my fault that he's so upset. But I'm not going to judge -- I've reacted that way myself more than once in my life.

Byron said...

Tony:
For somebody that's been there and done that? The response -- within the US as much as anywhere else -- would be, "That's nice, but you need to show me something new." It's simple human nature.
Maybe the problem here is that I just don't get humanity some days.

Shuttle was in many ways a technically much more robust accomplishment than Apollo. Using Shuttle technology -- which is what we are doing -- it should be no problem to put men back on the Moon. Yes, the reconstitution of an abandoned technological capability is far from trivial, but it still isn't demonstrating something new. That's why the Moon is off to the side of the Flexible Path. It just doesn't have the attraction of building advanced capabilities.
But I would say that the Lunar surface and Mars orbit are going to be similar in the area of "building advanced capabilities". We don't have the capability to go to either now, and they both have significant challenges. It may not look like that, but we'll be doing stuff on Luna that we didn't before.

But that's essentially what you're advocating for the Moon -- do it because it will bear fruit in the future, not because it's extending our capabilities into new areas of application now.
The difference is that we've done about as much as we can in LEO. The same cannot be said of Luna. After we've carefully studied it, tried ISRU, lived there, etc, then, assuming no commercial applications develop, I'll call it done.
My other big problem is the question nobody wants to ask. What happens after Mars? Is going somewhere and yelling 'First!' the goal of the space program? If not, then what is? And why does that dictate Mars? If so, what is wrong with you?

In the abstract, I agree with you. In the concrete, it just doesn't seem feasible anytime soon. IMO it will have to grow organically out of a sustained effort, as it becomes necessary.
A sustained effort will not happen on the basis of 'First!' We'll get to Mars, yell 'First!' and go home. And our grandchildren will do this whole discussion again.

How does ISRU of a palpable (even if extremely low pressure) Martian atmosphere get tested on the Moon?
Sorry, tested on Mars. The reactor that runs for a year or so before the people leave. Though you could separate out the astronaut's CO2 and feed it to the reactor on Luna. Probably just dump the methane. It won't run at full capacity, but it would be a useful test.

It's that Martian atmopsphere again. Even at 1% of Earth atmosphere pressure, it's enough to cause serious convection cooling of the suit. It's also the increased distance from the Sun, decreasing the impact of insolation. So you have to design the suit differently than you would design it for a vacuum environment with twice the insolation.
I still think you could probably get 75% commonality in the pieces. The outer shell and cooling stuff would have to be different, but that's about it.

Byron said...

It's the realistic environment that has to be lived with. Whenever I have this conversation, I always come to the conclusion that people are complaining about crackers because they aren't cookies, never considering that fact that at least they're getting something rather than nothing.
In the long term, we're getting some pretty red rocks in a vault in Houston. It's better than nothing, but it's not a sound basis for the future of our species.

WRT the end result, I'm just not that interested in what the philosophical goal should be if it can't be achieved in a realistic technical and economic sense. Flexible Path is something we can do, right now, with the technology we possess. Maybe something bigger will grow out of it. Maybe nothing bigger will come from it. But it's still doing something, right now, rather than waiting for a perfect storm that may never come. That's enough of a bottom line for me.
That is a good point. I will admit that it's probably the best we can do at the moment and have it have even a chance at getting funded. That doesn't mean that I agree with the philosophical underpinnings.
On the other hand, Constellation got funded and if not for NASA it probably would have worked.

As unfortunate as it is that it upsets Ferrell so much, I can really understand his anger. I'm basically going on, and on, and on telling him that this is the future, and it doesn't get much better -- not without magitech. That's really a blow to a lot of people's dreams and expectations.

So I get the lashing out in frustration. I'm sure Ferrell wants me to recant, and tell him everything's going to be fine. Since I won't, it's all my fault that he's so upset. But I'm not going to judge -- I've reacted that way myself more than once in my life.

I understand his anger. I was trying to complement you in staying civil in spite of his aggressive attacks.

Thucydides said...

This entire Mars tangent is interesting but could probably go to its own thread (hint) ;)

As for how we are going to get to Mars, I suspect that NASA, the ESA or any other State agency isn't going to be the crew that does it, but rather Planetary Resources as an offshoot of sending uprated Dragon capsules and Bigelow inflatable Habs to NEO's on prospecting and surveying missions. It would also be a huge publicity stunt to focus attention on Planetary Resources without mentioning they havn't actually turned a profit, yet....

Tony said...

Byron:

"But I would say that the Lunar surface and Mars orbit are going to be similar in the area of 'building advanced capabilities'. We don't have the capability to go to either now, and they both have significant challenges. It may not look like that, but we'll be doing stuff on Luna that we didn't before."

By "advanced capability", I mean something we've never done before. We've been to the Moon. We haven't been into interplanetary space for any length of time. Why a Lagrange point instead of Lunar oribt? Because orbiting the Moon puts you in shadow half the time, which diminishes by 50% the workout that the radiation protection of the hab. IOW, it may be outside of the Van Allen belt, but lLunar orbit is still not really the interplanetary environment.

"The difference is that we've done about as much as we can in LEO. The same cannot be said of Luna. After we've carefully studied it, tried ISRU, lived there, etc, then, assuming no commercial applications develop, I'll call it done."

It's "done" for putposes of firsts. And on that note...

"My other big problem is the question nobody wants to ask. What happens after Mars? Is going somewhere and yelling 'First!' the goal of the space program? If not, then what is? And why does that dictate Mars? If so, what is wrong with you?"

I'm thinking in terms of what people will pay for. One can't sell infrastructure or a sustained space presence that might involve 100 - 1,000 people by the end of the Century. "Flags and Footprints", as much as people love to sneer at it, is something about which the average Joe or Jane can say, "We did that," much like people in Oakland today are saying, "We won the American League West." It's inclusive, even if it's not preciseley true. People will pay for that feeling, whether it's baseball game tickets or space programs.

"A sustained effort will not happen on the basis of 'First!' We'll get to Mars, yell 'First!' and go home. And our grandchildren will do this whole discussion again."

We won't do anything more than LEO if we don't engage people in its accomplishment. "First!" can be sold. Not much else can be. And if we're still just sending out the occasional raid decades from now, rather than engaging in a decisive offensive, it's because that's what can be sold to the people, not because the wrong thing was sold to them.

"I still think you could probably get 75% commonality in the pieces. The outer shell and cooling stuff would have to be different, but that's about it."

The "cooling stuff" is what keeps the astronaut alive, by keeping him from either freezing or boiling in his suit. And that's the part that can't be tested in the wrong environment.

"In the long term, we're getting some pretty red rocks in a vault in Houston. It's better than nothing, but it's not a sound basis for the future of our species."

Nobody is going to buy the "future of our species". And if nobody buys it, it ain't happening anyway. Sorry.

A sustained space presence will have to develop organically out of what can be sold. There's literally no other way.

"That is a good point. I will admit that it's probably the best we can do at the moment and have it have even a chance at getting funded. That doesn't mean that I agree with the philosophical underpinnings."

It's the philosophy that can be sold. And selling the program is a requirement for the program to go somewhere. If you have a philosophically acceptable (for certain values of "acceptable") program that you can't sell, nobody is going anywhere.

Please don't respond to that. Just think about it for a while.

Tony said...

Byron:

"On the other hand, Constellation got funded and if not for NASA it probably would have worked."

Constellation was probably a technically workable program. It wasn't a fiscally realistic one. It required too much to be done in too short a time span, against a budget that just wasn't going to be funded.

Maybe more importantly, the most significant hardware piece of the program, a super heavy lift launch vehicle, has not been cancelled, and will in fact fly later this decade, with concrete missions for it in the planning stages. Let's be happy that we've got the crackers, even if they aren't cookies.

"I understand his anger. I was trying to complement you in staying civil in spite of his aggressive attacks."

I got that. I should have thanked you. My bust.

Look, I knew what I was doing. The points I was making to Ferrell were concrete and legitimate. It was enough to just make them. That they worked to exercise Ferrell so much, without becoming personal, is I think illustrative in itself. Maybe I thought there was value in that illustration...

Tony said...

Thucydides:

"...Planetary Resources..."

Right on their website, they say that "the success of commercial spaceflight causes the cost of Earth-originating launches to drop by a factor of 100" is a "conservative assumption". They're snake oil salesmen.

Byron said...

Tony:
By "advanced capability", I mean something we've never done before. We've been to the Moon. We haven't been into interplanetary space for any length of time. Why a Lagrange point instead of Lunar oribt? Because orbiting the Moon puts you in shadow half the time, which diminishes by 50% the workout that the radiation protection of the hab. IOW, it may be outside of the Van Allen belt, but lLunar orbit is still not really the interplanetary environment.
Fair enough, though I'd rate the thermal stuff as more important than the radiation shielding. That is really, really easy to do math on.

I'm thinking in terms of what people will pay for. One can't sell infrastructure or a sustained space presence that might involve 100 - 1,000 people by the end of the Century. "Flags and Footprints", as much as people love to sneer at it, is something about which the average Joe or Jane can say, "We did that," much like people in Oakland today are saying, "We won the American League West." It's inclusive, even if it's not preciseley true. People will pay for that feeling, whether it's baseball game tickets or space programs.
Now I understand the difference between us on this. You saw it happen. I didn't. So that first means a lot less to me.

The "cooling stuff" is what keeps the astronaut alive, by keeping him from either freezing or boiling in his suit. And that's the part that can't be tested in the wrong environment.
And that changes what I said how? We can test that on Earth quite effectively. Things like the atmospheric system, the gloves, the actual operation of the suit (getting in and out of it) can be tested on the moon.

Nobody is going to buy the "future of our species". And if nobody buys it, it ain't happening anyway. Sorry.
I know that. I'm just griping at this point.

Maybe more importantly, the most significant hardware piece of the program, a super heavy lift launch vehicle, has not been cancelled, and will in fact fly later this decade, with concrete missions for it in the planning stages. Let's be happy that we've got the crackers, even if they aren't cookies.
That annoys me more than anything else. They did cancel it, and run another set of studies that said the same thing as the first ones. So we threw away a few years of (admittedly slow) progress, and had to pay more. If it was so unsalvageable, then why didn't they cancel Orion as well? And looking at the entire thing, the way Constellation did it made more sense. They didn't have to man-rate the Ares V, and could send up people without using a huge booster.

Tony said...

Byron said...
Tony:
Byron:

"Now I understand the difference between us on this. You saw it happen. I didn't. So that first means a lot less to me."

I was barely aware of it when it happened. I was four years old and got to stay up late. I didn't quite get what was happening. I was only really conscious of the Moon from about Apollo 14 on. The big space poster on my bedroom wall when I was a kid? Space Shuttle. And I consider it kinda funny that many people, in the younger generation I share the Shuttle experience with, are almost a starry-eyed about it as we were back before it ever flew. Yet many in my generation were fed up with it by the end. Wierd juxtaposition of perspectives, huh?

What that says is that many people of my age want to break out of the LEO retrenchment that Shuttle and space stations sucked us into, and actually cross new ground, bypassing old ground as much as possible. So you're right -- to a lot of guys like me, landing on the Moon is, "F*ck, we've been here!" What I'm trying to do here, if I'm trying to do anything at all, is to entice younger people like yourself to come over to that dark side. ;-)

"And that changes what I said how? We can test that on Earth quite effectively. Things like the atmospheric system, the gloves, the actual operation of the suit (getting in and out of it) can be tested on the moon."

You can't field test it on the Moon. That's significant.

"That annoys me more than anything else. They did cancel it, and run another set of studies that said the same thing as the first ones. So we threw away a few years of (admittedly slow) progress, and had to pay more. If it was so unsalvageable, then why didn't they cancel Orion as well? And looking at the entire thing, the way Constellation did it made more sense. They didn't have to man-rate the Ares V, and could send up people without using a huge booster."

Let's look at this as a system of components. The major components for a full range of mission architectures would be:

Super heavy launch vehicle

Heavy lift launch vehicle (nominally for the crew module only)

Crew module with Earth return capability (and it's service module)

Interplanetary orbit injection stage, Earth-X

Interplanetary orbit injection stage, X-Earth

Interplanetary transit hab, generic

Interplanetary transit hab, Earth-Mars

Interplanetary transit hab, Mars-Earth

Lander, Moon

Lander, Mars

Surface stay infrastructure, Moon

Surface stay infrastructure, Mars

Nota bene: many of the above components are mission architecture specific. The components that are absolutely necessary for moving people from here to there over interplanetary distances are:

Super heavy launch vehicle

Crew module with Earth return capability (and it's service module)

Interplanetary orbit injection stage, Earth-X

Interplanetary orbit injection stage, X-Earth

Interplanetary transit hab, generic

That minimum list is enabled by giving the super heavy lifter a capability to carry the manned module. But to do that the LV had to be substantially redesigned. But it wasn't any more clean-sheeted than was already necessary to complete Ares V, because Ares V was essentially still in CD at the time of the lane shift.

On the other hand, there is in fact ongoing work to use the NASA crew vehicle with a heavy launcher, in the form of the Atlas V. So even that component wasn't lost -- just shifted to a less technically and operationally risky LV than the Stick (Ares I).

Brett said...

@Tony
Right on their website, they say that "the success of commercial spaceflight causes the cost of Earth-originating launches to drop by a factor of 100" is a "conservative assumption". They're snake oil salesmen.

Does their near-term plan of selling cheap telescopes require that as well? I thought it didn't, which gives them some possibility of making money even if it never becomes profitable to mine asteroids.

Tony said...

Brett:

"Does their near-term plan of selling cheap telescopes require that as well? I thought it didn't, which gives them some possibility of making money even if it never becomes profitable to mine asteroids."

Selling time on a telescope they put in LEO -- I don't think they have any plans to actually sell the scope to another operator -- is a pretty mundane space application, by today's standards. And it ain't gonna be very inexpensive time, if they're going to rely on current launch services to get it up there.

Thucydides said...

Regardless of their salesmanship, Planetary Resources is the only group which combines actual technology, a business plan and the deep pockets required to get started.

Some of the stuff like their "factor of 100" (and the related efforts of SpaceX to build a returnable spacecraft on their Falcon platform) don't strike me as being very realistic (I'm on record as suggesting the entire returnable Falcon program is vapourware to disguise the real objectives from LocMart and Boeing). OTOH, the Falcon and Dragon capsule are now flight tested hardware, and the Falcon 9 heavy configuration is pretty straightforward evolution of existing designs, so there is no real technical reason they can't do this.

Going out and doing something like that as a publicity stunt is good business in terms of keeping attention focused on the company and creating a spectacular which diverts attention from the "nuts and bolts" issues of quarterly profit/loss ratios and so on.

I would also think SpaceX/Planetary Resources is also well positioned (with Bigelow) in creating the infrastructure for longer term Space Tourism (beyond the joyrides that Virgin Galactic will sell), another potential source of revenue.

Locki said...

1. Byron: re centrifugal tethers for long endurance missions.

I see two big problems with this.

1. No evidence.
The most obvious one. There is no evidence low simulated G from centrifugal force will actual prevent the long term problems that occur with prolonged micro-gravity. We have exactly zero evidence

2. Difficult
Like Tony I've assumed this is a fiendishly difficult thing to do as no one was ever tried! There was a proposal for some sort of centrifuge on the ISS but it got ditched for economic reasons.

The way Tony has broken it down the problems with spinning boosters and habitats around tethers makes me thing it'll be an engineering accoplishment which will be both expensive and complex.

I mentioned in an earlier thread medicine rather than engineering may provide us some relief from long term micro-gravity. Maybe some sort of bisphosphanate will stabilise the skeleton (without the jaw necrosis side effects) and anabolic steroids will maintain the muscle bulk of both heart and skeletal muscle.

===============

The state of space technology.


A lot of us have been reminiscing about our romantic space travel childhood dreams.

The more you look into it the more likely it seems cheap and reliable space travel is slightly beyond our current technology level. Its just some damn risky and expensive with current tech levels.

I've got quite a few technical books lying around my study as well and my gut feeling is historically speaking we are at the rowboat stage of exploration.

By that I mean we can design rowboats. Sure if you are a crazy viking and you have a superb longboat you may cross the Atlantic – once. If you are lucky.
But really those crazy danes should have just waited for the full rigged sailboat to be invented.
Similarly, for our dreams of manned space travel to become a reality we have to wait till our entire tech base advances and we can leave behind muscle-powered boats and build full rigged sail boats. We have to wait till our astronomy has advanced enough so we can navigate reliably and fix both longtitude and latitide. We have to wait till the social situation favours humans taking a risk en masse and exploring the next frontier.
With the current tech base I'm in favour of exploration focusing on unmanned missions. Its safer and more economical.
To keep the public interested and keep the trickle of funding online NASA's primary aim should be to look for signs of life elsewhere in the solar system or even outside the solar system. I believe this is a strong enough motivation.

This is something that is achievable safely (no humans risked) and relatively affordably.

It will also raise some very interesting philosophical questions which might just spur on the human race to advance enough to where space exploration is economical. At the very least it will buy us time to allow our tech base to advance.

The very far sighted us would love for the human race to one day be able to leave the fragile earth behind to guarantee species survival.A worthy goal. It appeals to teh Sci-fi fan as well.

My fear is if we try to leave too early, if we invest too heavily in engineering projects not really achievable by our civilisation then we will end up bankrupting ourselves like past societies that over-stretched themselves before the technologies were ready. (Egyptians with pyramids, Mayans with their temples, Viking settlements in Greenland/North America)

longbeast said...

re: the ISS centrifuge unit, it would have been dangerous to the station had it been installed.

There would have been a minor risk to the station from an unbalanced load being put into the unit, as it could have wobbled the station frame faster than the structure could damp out the vibrations, but that would have been a manageable problem.

The much greater risk is the bearing on the centrifuge axle seizing up and suddenly dumping the entire momentum of the centrifuge into the rest of the station. That could tear the entire thing apart. There's really not much you can do to prevent that other than using a tiny centrifuge entirely contained within the hull.

Tethers have much nicer failure modes than rigid hulls with both static and rotating sections.

Geoffrey S H said...

"I've got quite a few technical books lying around my study as well and my gut feeling is historically speaking we are at the rowboat stage of exploration.

By that I mean we can design rowboats. Sure if you are a crazy viking and you have a superb longboat you may cross the Atlantic – once. If you are lucky.
But really those crazy danes should have just waited for the full rigged sailboat to be invented.
Similarly, for our dreams of manned space travel to become a reality we have to wait till our entire tech base advances and we can leave behind muscle-powered boats and build full rigged sail boats. We have to wait till our astronomy has advanced enough so we can navigate reliably and fix both longtitude and latitide. We have to wait till the social situation favours humans taking a risk en masse and exploring the next frontier.
With the current tech base I'm in favour of exploration focusing on unmanned missions. Its safer and more economical.
To keep the public interested and keep the trickle of funding online NASA's primary aim should be to look for signs of life elsewhere in the solar system or even outside the solar system. I believe this is a strong enough motivation.

This is something that is achievable safely (no humans risked) and relatively affordably.

It will also raise some very interesting philosophical questions which might just spur on the human race to advance enough to where space exploration is economical. At the very least it will buy us time to allow our tech base to advance.

The very far sighted us would love for the human race to one day be able to leave the fragile earth behind to guarantee species survival.A worthy goal. It appeals to teh Sci-fi fan as well.

My fear is if we try to leave too early, if we invest too heavily in engineering projects not really achievable by our civilisation then we will end up bankrupting ourselves like past societies that over-stretched themselves before the technologies were ready. (Egyptians with pyramids, Mayans with their temples, Viking settlements in Greenland/North America)"

Cannot agree with this enough. Why are we so obsessed with seeing Mars landings in our lifetime anyway? Knowing we might do it in the next millenia if we are lucky should be enough. Cheaper, more technologically advanced superheavy lifters are the only way to go, and we may not see anything that good for a long time.

Tony said...

Thucydides:

"Regardless of their salesmanship, Planetary Resources is the only group which combines actual technology, a business plan and the deep pockets required to get started."

What "actual technology"? They don't have a real product yet. When they do, it will be timesharing a LEO telescope satellite.

What "business plan"?

They can't show a profit -- or even significant revenues -- until they put their telescope in orbit. After that, it's all expenditures with no return until they start delivering resources to customers, presumably in LEO. That's presupposing those customers even exist, with a large enough demand to make the exercise pay. Talk about vaporware...

What "deep pockets"?

A few rich guys who don't mind a few million here, a few million there, in order to have their names associated with the idea (for whatever reason that is important to them)? You could liquidate Bill Gates's entire fortune and maybe, maybe put a man on the Moon. Establishing a space mining infrastructure on a few tens of millions of dollars -- or even a few hundreds of millions -- of angel investment? Really?

"Some of the stuff like their "factor of 100" (and the related efforts of SpaceX to build a returnable spacecraft on their Falcon platform) don't strike me as being very realistic (I'm on record as suggesting the entire returnable Falcon program is vapourware to disguise the real objectives from LocMart and Boeing)."

Maybe you're expressing yourself poorly, but that reads like conspiracy-mongering.

"OTOH, the Falcon and Dragon capsule are now flight tested hardware, and the Falcon 9 heavy configuration is pretty straightforward evolution of existing designs, so there is no real technical reason they can't do this."

There's no technical reason that anybody in the space industry couldn't have done it in the last thirty years. (Setting aside for the moment the engineering challenges and risks of remote controlled mining technology in space.) The roadblock is money to do it with current and foreseen technologies.

"Going out and doing something like that as a publicity stunt is good business in terms of keeping attention focused on the company and creating a spectacular which diverts attention from the 'nuts and bolts' issues of quarterly profit/loss ratios and so on."

Not for anywhere near long enough, and not without a market for their end product, which simply doesn't exist without their wildest speculations (like 1/100 cost to orbit) coming true.

"I would also think SpaceX/Planetary Resources is also well positioned (with Bigelow) in creating the infrastructure for longer term Space Tourism (beyond the joyrides that Virgin Galactic will sell), another potential source of revenue."

Too small a market, even at full staturation, to generate enough revenue to fund other stuff. Remember, even operated at a profit, space tourism still eats up 95% or better of its revenue on operations and other expenses. And the investors are going to want ROI from the space tourism revenue stream, not plow it all back into NEO resource utilization. After that, and after taxes, how much real investment capital is left for other projects, even on a $10B/yr (which I think is wildly optimistic) industry?

BTW, for those of you that think I'm being irremediably negative, these are just the questions that have to be answered before this venture has even a sliver of a chance of success. I don't see any rational, concrete answers materializing any time soon. Just a lot of handwaving and, "Trust us, you'll see..." Bleh.

Tony said...

Locki:

"Like Tony I've assumed this is a fiendishly difficult thing to do as no one was ever tried!"

I didn't say it was particularly difficult. It seems rather straightforward and has in fact been substantially demonstrated for over forty years (Gemini 11). I'm more concerned with the technical and program risk, compared to the perceived benfit. Space flight is hard enough without courting additional risk. Maybe the benefits outweigh the risks. In fact, I wouldn't be surprised if that was the case. But that's a subject for analysis, not assertion.

As for the rest, I don't think we are in as primitive a state as you suggest. We just have to develop our capabilities slowly and steadily, within our discretionary speniding budget.

WRT manned spaceflight in particular, if people don't go, what's the point? Scientific knowledge may be factual, and even interesting in an abstract way. But it's still sterile. Personal human involvement is what's worth the big bucks. It's having a member of the tribe go and come back, to tell us the story first hand, instead of sending a letter home (which is little better than robotic exploration).

jollyreaper said...

Why do we want to see a Mars landing? Because it's romantic.

We're back to that Shaw quote: "The reasonable man adapts himself to the world; the unreasonable one persists in trying to adapt the world to himself. Therefore all progress depends on the unreasonable man."

And to reiterate, we won't remember the unreasonable men who fail, only the ones who succeed.

To bring things out of the celestial realm, just consider parents with kids. Becoming a dentist or accountant seems like a safe career choice. That's what a parent would advise the child to shoot for. Becoming a basketball great, a rockstar, a famous actor? Not likely. Even going into the sciences seems like a rough go. Nevertheless, there are people who pursue such dreams and succeed. It's a poor gamble but people have their dreams.

The real question when you see someone strike out with a crazy idea and a dream and he succeeds, was he so smart he'd thought of everything and it wasn't actually as big of a risk as it looked like or was he just really, really, fantastically lucky along with everything else?

Tony said...

jollyreaper:

"Why do we want to see a Mars landing? Because it's romantic.

We're back to that Shaw quote: "The reasonable man adapts himself to the world; the unreasonable one persists in trying to adapt the world to himself. Therefore all progress depends on the unreasonable man."

And to reiterate, we won't remember the unreasonable men who fail, only the ones who succeed.

To bring things out of the celestial realm, just consider parents with kids. Becoming a dentist or accountant seems like a safe career choice. That's what a parent would advise the child to shoot for. Becoming a basketball great, a rockstar, a famous actor? Not likely. Even going into the sciences seems like a rough go. Nevertheless, there are people who pursue such dreams and succeed. It's a poor gamble but people have their dreams.

The real question when you see someone strike out with a crazy idea and a dream and he succeeds, was he so smart he'd thought of everything and it wasn't actually as big of a risk as it looked like or was he just really, really, fantastically lucky along with everything else? "


There are some days I just want to say, "Who are you, and what have you done with my jollyreaper!?" :-P

In any case, I think the rebel-visonary-fighting-the-power-and-winning legend is a case of confirmation bias. Yes, it does happen, but mostly -- by a very long margin -- it doesn't. Progress is generally made by competent, well supported actors working from within the system. That's what put men on the Moon, not a Heinleinian individualist industrialist. It's what will put men on Mars, whenever that happens.

Or, to put it another way, Jobs made the computer personal, but IBM and the clone manufacturers put it on the desk and put it to work. Plenty of small-time guys built motor cars, but Ford and GM brought them to the masses. The Wrights flew, but government investment in military aircraft and general aviation research made the commercial aviation industry.

jollyreaper said...

Along the lines of the surprising...

The tech disruption in entertainment is astounding. The VCR came out of nowhere. It revolutionized home entertainment. Mom and pop rental stores sprung up. Corporate money saw it was a good business and Blockbuster was the galactic empire.

I never thought I would have seen the day such a monster could be slain but in a shorter period of time than could have been imagined blockbuster is now a read man walking, a punch line. And they were very big players, huge money. Completely destroyed.

http://arstechnica.com/business/2012/10/too-little-too-late-blockbuster-gives-up-on-trying-to-beat-netflix/

Fascinating to behold.

The onion take on blockbuster is also astoundingly good.

Thucydides said...

Planetary Resources has actual technology in the form of the Falcon 9 launcher and Dragon space capsule, as well as the Bigelow inflatable Hab (although that part hasn't been flight tested yet to my knowledge).

The business plan, however improbable it might seem, is still vitally important because it prevents the NASA problem of spending billions of dollars to create endless PowerPoint presentations. NASA management has no incentives to control costs, or even to have a successful program. The Planetary Resources management team and investors do have a positive incentive to succeed.

I'm pretty sure that the same sort of techniques that have allowed SpaceX to start up with a relatively modest investment (in aerospace terms) is being applied to Planetary Resources, and by extension through the various partnership companies which encompass Planetary Resources, so the deep pockets will go quite a bit further than most of us will expect. I also reflect on Robert Zubrin's observation that the bulk of the spending at his former employer Lockheed's space division was not related to space hardware or the factory floor at all, but rather in bureaucratic overhead to satisfy government customers and regulations.

How well Planetary Resources does and if their business plan is doable isn't something that any of us can answer in any definitive way. We may have our suspicions, or our hopes, but several very smart people have put their own personal money on the line, so they obviously have reason to believe this is an acceptable use of their capital. Time will tell.

Anonymous said...

Thucydides:

Planetary Resources has actual technology in the form of the Falcon 9 launcher and Dragon space capsule, as well as the Bigelow inflatable Hab (although that part hasn't been flight tested yet to my knowledge).

Bigelow launched Genesis I in 2006 and Genesis II in 2007. Both prototypes are still in orbit.

If you want to keep up with spaceflight technology, www.nasaspaceflight.com is a good website.

Ron

Anonymous said...

Quoted from Thucydides:

How well Planetary Resources does and if their business plan is doable isn't something that any of us can answer in any definitive way. We may have our suspicions, or our hopes, but several very smart people have put their own personal money on the line, so they obviously have reason to believe this is an acceptable use of their capital. Time will tell.

Why is it that if I had suspicions or doubts about the reasonableness of Planetary Resource's plan, that I should take heed of what some smart person did? He could be very smart indeed, and had put a lot of money down on Planetary having something big, but what? It's what the crowd is looking at, not the crowd itself that should important in making a financial discussion. If Steve Jobs were alive today, and everyone saw him fighting with Bill Gates over who is first in line to invest, would you take that as a sign to invest in this company, even if it was some sort of Company XYZ you never heard of before, and had less info to look at than Planetary has now? Saying that some smart person has looked at it and bought in doesn't prove much. Planetary Resources could have a great, truly kick-ass and workable idea; but that has to be seen with one's own eyes and the sums tallied up. Trusting a rich person to pick the right company is silly, especially since he can lose money and still keep his shirt(to a point).

OrbitingPluto

mejobo said...

Tony- "There's no technical reason that anybody in the space industry couldn't have done it in the last thirty years. (Setting aside for the moment the engineering challenges and risks of remote controlled mining technology in space.) The roadblock is money to do it with current and foreseen technologies."

Huh, thats funny. Last time we discussed something similar I thought you thought the issue was that it was physically impossible for any form of heat engine to go into space with any seriousness...

Similarly, you've gone from "human space flight is impossible beyond a few people in LEO" to "human space flight should be the goal, who needs robotic exploration."

I have no problem with this.

Personally I think SSTO and/or entirely reusable is going to need something bizarre... Skylon is appropriately bizarre, but that doesn't mean it will succeed (way too early to say either way).

I don't really see why we need SSTO or reusable rockets, though. If they aren't as economically feasible as several stages to orbit disposable rockets, then why would we invest in something that will only slow down our expansion into space?

Planetary Resources is basically all smoke and mirrors right now, but they're basic plan (establish mining in Earth orbit to help develop infrastructure in space, and if anything is valuable enough to send it back to Earth, you get that too) is generally fine, even if they're not the ones who'll eventually implement it.

Tony said...

Thucydides said...
Thucydides:

"Planetary Resources has actual technology in the form of the Falcon 9 launcher and Dragon space capsule,"

How is that their technology? It's technology that anyone who can pay for SpaceX launch services has. And they have to come up with money to pay for it.

"as well as the Bigelow inflatable Hab (although that part hasn't been flight tested yet to my knowledge)."

Small, unmanned versions have been put into orbit and tested there. But once again, it's not anybody's technology that doesn't pay for it, and everybody's technology who can.

"The business plan, however improbable it might seem, is still vitally important because it prevents the NASA problem of spending billions of dollars to create endless PowerPoint presentations. NASA management has no incentives to control costs, or even to have a successful program. The Planetary Resources management team and investors do have a positive incentive to succeed."

NASA conceptual designs, even when they never get funded, are substantially more detailed and specific than anything PR has offered. As for incentives, everybody who participated in the dot com boom had great incentive to succeed. But when you have no real product, guess what?

"I'm pretty sure that the same sort of techniques that have allowed SpaceX to start up with a relatively modest investment (in aerospace terms) is being applied to Planetary Resources, and by extension through the various partnership companies which encompass Planetary Resources, so the deep pockets will go quite a bit further than most of us will expect."

SpaceX has a real product that can actually attract a revenue stream. PR has no revenue stream (except for telescope time that will probably barely pay for the telescope deployment on operations). It has a deep, deep hole in which to throw money, and no revenue until and unless a highly unlikely confluence of events puts large market for water in Earth orbit.

"I also reflect on Robert Zubrin's observation that the bulk of the spending at his former employer Lockheed's space division was not related to space hardware or the factory floor at all, but rather in bureaucratic overhead to satisfy government customers and regulations."

Zubrin is full of magnificently odifferous crap. Most of the money may not go into hardware per se, but a whole lot of the money that goes into quality assurance is absolutely necessary, because mere hardware is not what the customer wants. He wants hardware that will stand the space environment and reliably perform there as well. I'm sure some of what the NASA and military customer demands is bureaucratic nonsense. But a lot of it is necessary for ensuring the money that is invested isn't totally lost.

IOW, if the environment demands $60 insurance on $40 direct operational expenditure, well, that's what it demands. You may have to pay $100 for what otherwise might be $40 worth of work, but you can't send out a tech rep to LEO, much less a NEO or Mars.

"How well Planetary Resources does and if their business plan is doable isn't something that any of us can answer in any definitive way. We may have our suspicions, or our hopes, but several very smart people have put their own personal money on the line, so they obviously have reason to believe this is an acceptable use of their capital. Time will tell."

A lot of very smart, experienced people bought a slick PowerPoint "business plan" during the dot com boom. The vast majority of them lost their investment, because none of the schemes for monetization could actually work in real life. Forgive me for thinking so, but I'm not seeing a viable revenue stream in any of PR's plans.

Tony said...

Joe Beute:

"Huh, thats funny. Last time we discussed something similar I thought you thought the issue was that it was physically impossible for any form of heat engine to go into space with any seriousness..."

Hardly. What I have said is that heat engines are a mature technology and we have to live within their limitations, rather than just handwave them away.

"Similarly, you've gone from 'human space flight is impossible beyond a few people in LEO' to 'human space flight should be the goal, who needs robotic exploration.'"

If you can establish those as direct quotes, I'd love to see it.

What I have said is that human spaceflight to wherever (not just LEO) is going to be limited to a few people at a time for the foreseeable future. But it is still a desirable thing, because sending people is preferable to robots, for reasons of pride and accomplishment.

"I have no problem with this."

I hope you have no problems with what I actually have been saying either.

"I don't really see why we need SSTO or reusable rockets, though. If they aren't as economically feasible as several stages to orbit disposable rockets, then why would we invest in something that will only slow down our expansion into space?"

Whatever we use, as long as it is in the same ballpark WRT cost as what we're spending now, not much beyond exploration by a few people at a time is going to be feasible.

Thucydides said...

WRT Planetary resources, I would define it almost as a consortium of various areospace companies, since many of its primary backers are the owners of these companies. This is a bit different from the .com bubble, so when I said some very smart people were investing, they are investing in their own industry, an area they know about. If they were investing in some social media startup or undersea mining (to pull some examples from the air), then I would be much more skeptical.

Their business plan does seem improbable, and they will almost certainly have to branch out into some other sideline (space tourism is the only direct example I can call to mind for the short and medium term), but once again. only time will tell.

Tony said...

Thucydides:

"WRT Planetary resources, I would define it almost as a consortium of various areospace companies, since many of its primary backers are the owners of these companies."

Hardly:

http://en.wikipedia.org/wiki/Planetary_Resources

The company seems more a corporate umbrella, under which to promote speculative investment, than it seems to be anything else.

"This is a bit different from the .com bubble,"

It's hardly different at all. It has all of the hallmarks -- no real product, visionary BS, and The Big Reveal.

"so when I said some very smart people were investing, they are investing in their own industry, an area they know about."

Since when have any of the listed investors (http://www.planetaryresources.com/team/) been in the aerospace industry? They're all infotech people. Which figures, because real aerospace people don't make enough money to invest millions, and wouldn't invest in such a pie-in-the-sky scheme anyway.

"If they were investing in some social media startup or undersea mining (to pull some examples from the air), then I would be much more skeptical."

Plenty of reason to be skeptical. The founders were the chief officers and promoters of Space Adventures, which is nothing but rocket rides for the ultra rich. And the only reason they can even offer that service is because the Russian space program isn't above turning a seat in a Soyuz into a little bit of extra revenue.

"Their business plan does seem improbable, and they will almost certainly have to branch out into some other sideline (space tourism is the only direct example I can call to mind for the short and medium term), but once again. only time will tell."

It doesn't take time to tell that it has all of the indicators of a specualtive investment scheme (and I mean "scheme" in every possible perjorative sense).

Tony said...

Re: SpaceX

http://www.youtube.com/watch?v=y6zsZiVa998

The engine failure at 0:29 in the video is described as an "explosion" in several sources. SpaceX is calling it a "flameout" and comparing it to Saturn V operational engine shutdowns. Thing is, Saturn V never had an engine self-destruct in flight. Both shutdowns on that vehicle were precautionary, and initiated by flight software. The engine-out routine of the flight control software obviously did its job, but looking at all the debris flying around after the failure. How lucky was SpaceX not to lose neighboring engines?

On top of all of that, the second stage failed to restart as planned, in order to position a piggyback payload.

An honest assessment would be that this ain't good from a reliability engineering standpoint, much less from a man-rating perspective. It's way too early to tell for sure, and I ain't going to be saying, "I told you so," for quite a while yet, but its entirely possible that SpaceX's budget operational approach just caught up with them.

Byron said...

I'm with Tony WRT Planetary Resources. It doesn't seem to have much of a business plan, and the people involved are not aerospace.

Falcon 9 Flight:
The engine failure at 0:29 in the video is described as an "explosion" in several sources. SpaceX is calling it a "flameout" and comparing it to Saturn V operational engine shutdowns. Thing is, Saturn V never had an engine self-destruct in flight. Both shutdowns on that vehicle were precautionary, and initiated by flight software. The engine-out routine of the flight control software obviously did its job, but looking at all the debris flying around after the failure. How lucky was SpaceX not to lose neighboring engines?
Apollo got really lucky that the shutdown on Apollo 13 happened when it did. The engine was about to pogo and tear the rocket apart, except for a pressure sensor that happened to trip.
I'm not sure about how much of the mission's survival was luck, and how much was proper design. The debris seemed to be mostly going backwards, and given that it was doing about Mach 1, that means it was propelled out fairly slowly. It happened about Max Q, which means that it probably wasn't just the engine itself, but rather an aerodynamic event. As to what exactly, I don't know.
Also, this should be put in context. Compare the Falcon 9 to just about every other launcher at this stage of its development. Saturn and the Shuttle were about the only ones that didn't have some sort of catastrophic failure early on. Saturn had a mix of lucky and good (plus most of the engines had already been tested), and we all know about the shuttle.
And keep in mind that it made orbit, and that the launch vehicle is still relatively young.

An honest assessment would be that this ain't good from a reliability engineering standpoint, much less from a man-rating perspective. It's way too early to tell for sure, and I ain't going to be saying, "I told you so," for quite a while yet, but its entirely possible that SpaceX's budget operational approach just caught up with them.
Some stuff just can't be tested on the ground. If this was an aerodynamically-induced failure, I'm not surprised they didn't find it. Things like that don't scale well. I remain somewhat optimistic.

Tony said...

Byron:

"I'm not sure about how much of the mission's survival was luck, and how much was proper design. The debris seemed to be mostly going backwards, and given that it was doing about Mach 1, that means it was propelled out fairly slowly. It happened about Max Q, which means that it probably wasn't just the engine itself, but rather an aerodynamic event. As to what exactly, I don't know."

Well, the debris would go backward eventually, given the dynamics of atmospheric flight. The concern is what it gets propelled into before falling away from the vehicle. According to the most recent report I read, SpaceX is claiming that the engine suffered a sudden low pressure fault and was commanded to shut down. They're further claiming that it wasn't an explosion, because the sensors were still delivering data. Hmmm. How much pressure was lost? All? Some? To what degree?

SpaceX further claims that the visible debris in the video was aerodynamic fairing that failed due to the engine's "pressure release". Uhhh-huh. What kind of physical event causes a sudden loss of most or all of the engine pressure, creating a large enough overpressure in a well-ventilated (essentially open at the ass end) fairing, leading to the fairing failing in a visibly spectacular way? Even without all of the data, I'd guess a seal in a propellant line, past the pump -- or mayber something in the pump itself. I think that comes under the heading of "not good".

"Apollo got really lucky that the shutdown on Apollo 13 happened when it did. The engine was about to pogo and tear the rocket apart, except for a pressure sensor that happened to trip."

(taken a little out of order)

Pogo was a known structural dynamics issue. The propulsion system performed exactly as designed to mitigate it.

"Also, this should be put in context. Compare the Falcon 9 to just about every other launcher at this stage of its development. Saturn and the Shuttle were about the only ones that didn't have some sort of catastrophic failure early on. Saturn had a mix of lucky and good (plus most of the engines had already been tested), and we all know about the shuttle."

Atlas V has not had a booster failure in 33 flights, catastrophic or otherwise.

Delta IV had a booster early shutdown on flight 4, caused by a sensor failure, not an engine failure. Not catastrophic, in terms of vehicle loss, though it did cause a mission failure.

"And keep in mind that it made orbit, and that the launch vehicle is still relatively young."

Of course we should keep that in mind. We should also keep in mind that prime time Western LVs don't have mechanical failures in booster rocket engines very often anymore, even in early flights. It remains to be seen whether SpaceX can write this one off as truly anomalous, or whether it's an indication of something systemic in the Merlin design.

"Some stuff just can't be tested on the ground. If this was an aerodynamically-induced failure, I'm not surprised they didn't find it. Things like that don't scale well. I remain somewhat optimistic."

We shall indeed have to wait and see. But this was a 1960s style engine failure in a 21st Century rocket. There's ample reason to be genuinely concerned.

Byron said...

Tony:
Well, the debris would go backward eventually, given the dynamics of atmospheric flight. The concern is what it gets propelled into before falling away from the vehicle. According to the most recent report I read, SpaceX is claiming that the engine suffered a sudden low pressure fault and was commanded to shut down. They're further claiming that it wasn't an explosion, because the sensors were still delivering data. Hmmm. How much pressure was lost? All? Some? To what degree?
My point was that the debris was not visibly propelled into anything, which means that it wasn't a threat to the rocket. I'm not sure exactly what happened to the engine.

SpaceX further claims that the visible debris in the video was aerodynamic fairing that failed due to the engine's "pressure release". Uhhh-huh. What kind of physical event causes a sudden loss of most or all of the engine pressure, creating a large enough overpressure in a well-ventilated (essentially open at the ass end) fairing, leading to the fairing failing in a visibly spectacular way? Even without all of the data, I'd guess a seal in a propellant line, past the pump -- or mayber something in the pump itself. I think that comes under the heading of "not good".
I'm not sure either. It is possible that there was some pressure being generated by the engine behind the fairing, and that when the engine shut down, the loss of pressure (remember that this was during Max-Q) caused the fairing to rip off.
Oh, you mean it ripped off from behind. That would be bad.

Pogo was a known structural dynamics issue. The propulsion system performed exactly as designed to mitigate it.
No, there was a fix in the pipeline, but the center engine came close to wrecking the stage.

I forgot the EELVs. Oops.

Of course we should keep that in mind. We should also keep in mind that prime time Western LVs don't have mechanical failures in booster rocket engines very often anymore, even in early flights. It remains to be seen whether SpaceX can write this one off as truly anomalous, or whether it's an indication of something systemic in the Merlin design.
I don't think it can be called systemic, as they've flown about 40 and one has failed. They'll find out what happened, fix it, and move on.

We shall indeed have to wait and see. But this was a 1960s style engine failure in a 21st Century rocket. There's ample reason to be genuinely concerned.
I still don't think that the engine actually exploded. It would have done more damage if it did. And, in the end, it got to orbit.

Tony said...

Byron:

"My point was that the debris was not visibly propelled into anything, which means that it wasn't a threat to the rocket. I'm not sure exactly what happened to the engine."

Not this time, no. But we shouldn't assume that a non-catastrophic failure is the only outcome -- or even the most likely outcome -- of this species of failure. What could have happened this time? What happens next time? Those are questions that need to be answered.

"I'm not sure either. It is possible that there was some pressure being generated by the engine behind the fairing, and that when the engine shut down, the loss of pressure (remember that this was during Max-Q) caused the fairing to rip off.
Oh, you mean it ripped off from behind. That would be bad."


I guess it's not beyond the realm of possibility, but I don't think that a competent engineer would rely on engine back-pressure to keep an otherwise insufficiently strong fairing inflated. Consider this -- the likely maximum back-pressure is experienced on the ground, after engine start and before liftoff, as some of the exaust gasses (and steam pressure from vaporized sound suppression water streams) are reflected out of the flame trench and back into the rocket. A fairing strong enough to withstand that from the inside can probably withstand dynamic pressures from the outside. (It's shape is inherrently stronger in compression than tension.)

In any case, I think I'll stick with a seal failure somewhere in the pressurized part of the propellant delivery system.

"No, there was a fix in the pipeline, but the center engine came close to wrecking the stage."

No it didn't, because pressure oscillations were detected in the engine and it was shut down before it could do harm. Since those pressure oscillations were a known irreducible contributor to pogo -- other contributing causes were reducible and were reduced over time -- I think it's fair to say that the shutdown was part of a pogo mitigation strategy.

"I don't think it can be called systemic, as they've flown about 40 and one has failed. They'll find out what happened, fix it, and move on."

Just because a failure is occasional, that doesn't mean it isn't also systemic. It just means that it's relatively low rate.

"I still don't think that the engine actually exploded. It would have done more damage if it did. And, in the end, it got to orbit."

I don't think it exploded either, in terms of mechanically disintegrating. But something busted and released a lot of pressure very quickly. Hope they have enough telemetry, and enough of the right kind, to pin the cause down.

Byron said...

Tony:
Not this time, no. But we shouldn't assume that a non-catastrophic failure is the only outcome -- or even the most likely outcome -- of this species of failure. What could have happened this time? What happens next time? Those are questions that need to be answered.
We have no clue what the species of failure is as of yet. It could be that a bad bolt got in somewhere. It could be that the Merlin is a piece of junk, and SpaceX got lucky before. I'm betting it's somewhat closer to the first than the second.

In any case, I think I'll stick with a seal failure somewhere in the pressurized part of the propellant delivery system.
I understand. I guess I can't quite get over the fact that it happened at Max-Q. They test the engines enough that I'm betting it's somehow related to that.

Just because a failure is occasional, that doesn't mean it isn't also systemic. It just means that it's relatively low rate.
Point. But even then, I think you're being too harsh. We know virtually nothing beyond the fact that the engine shut down and the fairing disintegrated (I wouldn't be surprised if the order was the other way around, though).

Tony said...

Byron:

"We have no clue what the species of failure is as of yet. It could be that a bad bolt got in somewhere. It could be that the Merlin is a piece of junk, and SpaceX got lucky before. I'm betting it's somewhat closer to the first than the second."

Because we don't know what the failure was, it's a bit early to speculate what exactly the cause is and what the fix is. But regardless of the failure's nature, just because the vehicle made it to orbit this time doesn't mean that was the only possible outcome, or even the most likely one. Knowing nothing more than there was a sensed pressure drop inside the engine, combined with a visually obvious loss of airframe structure on the outside of the engine, it's not at all premature to think that it could have been worse.

"I understand. I guess I can't quite get over the fact that it happened at Max-Q. They test the engines enough that I'm betting it's somehow related to that."

It is definitely an intriguing coincidence. Whether one is the cause of the other...well, we'll know soon enough. If it is related to Max-Q, that would be pretty strong evidence of design or weakness, don't ya think? I mean, dynamic pressure is a pretty well understood phenomenon, with plenty of data for vehicle designers to work from.

"Point. But even then, I think you're being too harsh. We know virtually nothing beyond the fact that the engine shut down and the fairing disintegrated (I wouldn't be surprised if the order was the other way around, though)."

The engine didn't just "shut down". It experienced a pressure loss and then shut down. That's indicative of a hardware failure, whatever the ultimate cause.

Maybe it was poor construction. Maybe it was poor design. Maybe it was just an unavoidable accident of some type or the other. I'll go on the record right now that it's something that has to be fixed, either phsycally or procedurally. And I'll give you long odds that it was avoidable with more attention and time paid to either design development or quality control at the manufacturing end.

Byron said...

Tony:
Because we don't know what the failure was, it's a bit early to speculate what exactly the cause is and what the fix is.
That's what I said. We don't know if this was a one-off failure or a systemic problem, so going off on SpaceX is a bit premature.

It is definitely an intriguing coincidence. Whether one is the cause of the other...well, we'll know soon enough. If it is related to Max-Q, that would be pretty strong evidence of design or weakness, don't ya think? I mean, dynamic pressure is a pretty well understood phenomenon, with plenty of data for vehicle designers to work from.
It would indeed be a design or manufacturing problem if it was the pressure. On the other hand, Skylab had something of that sort, so it's not unheard of.

The engine didn't just "shut down". It experienced a pressure loss and then shut down. That's indicative of a hardware failure, whatever the ultimate cause.
Yes, it is. And hardware occasionally fails.

Maybe it was poor construction. Maybe it was poor design. Maybe it was just an unavoidable accident of some type or the other. I'll go on the record right now that it's something that has to be fixed, either phsycally or procedurally. And I'll give you long odds that it was avoidable with more attention and time paid to either design development or quality control at the manufacturing end.
To some extent, it almost certainly was avoidable with better QC or design. The same can be said of all engineering failures. The question is to what extent that is true. It could be that SpaceX was careful, and something just slipped through (Apollo 13 springs to mind as an example of this). Or it could be that it really should have been foreseen and avoided (Challenger). We don't know, and I'm going to give them the benefit of the doubt for now.

Tony said...

Byron:

"That's what I said. We don't know if this was a one-off failure or a systemic problem, so going off on SpaceX is a bit premature."

I think we're talking past each other a bit here. I'm not "going off" on SpaceX so much as I'm pointing out that this may be -- may be, not certainly is -- symptomatic of their budget rocket approach.

"It would indeed be a design or manufacturing problem if it was the pressure. On the other hand, Skylab had something of that sort, so it's not unheard of."

Not unheard of, but certainly much better understood forty years later. And one would think that of all places for it to manifest itself, it wouldn't be in the propulsion system end of things.

"And hardware occasionally fails."

Of course. But why it fails is a whole 'nuther can of beans. The why is the important thing to understand. Because that will tell you whether this was just an unfortunate confluence of events, or symptomatic of deeper issues.

"To some extent, it almost certainly was avoidable with better QC or design. The same can be said of all engineering failures. The question is to what extent that is true. It could be that SpaceX was careful, and something just slipped through (Apollo 13 springs to mind as an example of this). Or it could be that it really should have been foreseen and avoided (Challenger). We don't know, and I'm going to give them the benefit of the doubt for now."

I think the key thing to understand in this respect is that SpaceX didn't spend as much money on design development and quality control as other LV providers have in the past. Maybe that's okay. But maybe it's just not good enough for reliable space launch. It wouldn't surprise me to learn something about that, one way or the other, from this incident.

Byron said...

Tony:
Not unheard of, but certainly much better understood forty years later. And one would think that of all places for it to manifest itself, it wouldn't be in the propulsion system end of things.
Most of that sort of work was done long before Skylab. And my best guess (which isn't that well informed) is that something at the top of the fairing broke loose, and hit something on the engine. Then, a little while later, the fairing went.

I think the key thing to understand in this respect is that SpaceX didn't spend as much money on design development and quality control as other LV providers have in the past. Maybe that's okay. But maybe it's just not good enough for reliable space launch. It wouldn't surprise me to learn something about that, one way or the other, from this incident.
No, they decided to try to add some tolerance for failure instead of spending bucketloads of money to make sure it doesn't happen. Which seems to have worked fairly well for them in this case. Until we know more, I'm reserving judgement.

Rick said...

Welcome to a couple of new commenters!

I am with Byron in reserving judgment - for now - about whatever happened during the SpaceX launch. It was an anomaly that remained within the limits of the system to complete the mission. But prospective customers will want to know that it has been thoroughly evaluated.

In the broader discussion it won't surprise regulars here that I mostly agree with Tony. The careful, incrementalist approach evolved for good reasons, and efforts to save money via shortcuts are liable to end badly.

On going to Mars, see a previous discussion here of just that subject, A Mission To Mars.

Tony said...

Byron:

"Most of that sort of work was done long before Skylab."

They didn't have Computational Fluid Dynamics (CFD) for Skylab. They weren't exactly in the "by guess and by golly" mode, but even with wind tunnel testing they only have very rough estimates of how strongly the solar wings needed to be attached, and how to fair them. Things are a bit different today.

"No, they decided to try to add some tolerance for failure instead of spending bucketloads of money to make sure it doesn't happen. Which seems to have worked fairly well for them in this case. Until we know more, I'm reserving judgement."

There are two distinct issues here:

1. An engine-out capability is basic equipment for a multi-engine stage. It goes all the way back to Saturn. Having such a capability isn't a choice. It just means you studied prior art and applied lessons learned. Touting the capability -- in an email to a reporter, as reported by William Harwood on cnet.com (http://news.cnet.com/8301-11386_3-57528333-76/spacex-still-determining-cause-of-falcon-9-engine-failure/) -- as some kind of value-added is disingenuous (and typically Elon Muskish). It wouldn't surprise me if more than a few people in SpaceX engineering circles cringed at that bit of spin.

2. Just because the event was survivable and survived in this instance, it doesn't mean it was necessarily survivable in all instances. As Harwood also points out in his article: "Any problem serious enough to generate debris typically results in a loss of mission." As much as people like to say that they'd rather be lucky than good, luck is no substitute for competence.

And competence comes at a cost. Maybe SpaceX invested enough in competence and this was just bad luck. But maybe not. And that's a question that, after Sunday night, is legitimately still open.

Also, there's still the anomaly of the Orbcomm payload, which apparently didn't reach the specified orbit. Speculation is that the second stage either didn't restart as planned, or there wasn't enough propellant left in the upper stage after compensating for the engine lost on the booster. How that plays out is going to be interesting as well.

Byron said...

Tony:
They didn't have Computational Fluid Dynamics (CFD) for Skylab. They weren't exactly in the "by guess and by golly" mode, but even with wind tunnel testing they only have very rough estimates of how strongly the solar wings needed to be attached, and how to fair them. Things are a bit different today.
My guess as to a possible aerodynamic culprit is some quirk of aerodynamics that the CFD missed (or possibly a manufacturing error). Such things have finite precision, and wind tunnels don't scale perfectly. They designed it for 1.25x the CFD/tunnel predilections, and for whatever reason, it hit 1.5x in one spot. That broke, and it took the system with it. They reinforce and keep flying.
I'm not sure what in the engine could be the culprit.

1. An engine-out capability is basic equipment for a multi-engine stage. It goes all the way back to Saturn. Having such a capability isn't a choice. It just means you studied prior art and applied lessons learned. Touting the capability -- in an email to a reporter, as reported by William Harwood on cnet.com -- as some kind of value-added is disingenuous (and typically Elon Muskish). It wouldn't surprise me if more than a few people in SpaceX engineering circles cringed at that bit of spin.
Taking a quick look through the list of current boosters in the same class, they all have a single engine on the first stage. So while it is inherent to the design, that design is not shared by other rockets. If there were other boosters with large numbers of first-stage engines, I might take issue with it, but there aren't.

2. Just because the event was survivable and survived in this instance, it doesn't mean it was necessarily survivable in all instances. As Harwood also points out in his article: "Any problem serious enough to generate debris typically results in a loss of mission." As much as people like to say that they'd rather be lucky than good, luck is no substitute for competence.
You are correct that there are cases in which the loss of one engine would lead to the loss of the vehicle. But given that no other current booster can tolerate the loss of an engine at all, this does not strike me as terribly profound.

And competence comes at a cost. Maybe SpaceX invested enough in competence and this was just bad luck. But maybe not. And that's a question that, after Sunday night, is legitimately still open.
Yes, it is. But I'm extending the presumption of competence until I get some hard evidence to the contrary.

Also, there's still the anomaly of the Orbcomm payload, which apparently didn't reach the specified orbit. Speculation is that the second stage either didn't restart as planned, or there wasn't enough propellant left in the upper stage after compensating for the engine lost on the booster. How that plays out is going to be interesting as well.
That actually worries me more than the loss of the first-stage engine. I'd imagine that they put a lot more work into the second stage engine than the first-stage ones, so having it fail to re-light is a bit worrisome. That said, it could have been a consequence of the first-stage failure. Until we know more, I'm withholding judgement.

jollyreaper said...

Rick, comment I thought I'd make, I do appreciate when you post in the last post discussion that there's a new post up. Saves having to keep checking the website. I do all my reading in my mail client and only hit the site when I want to reply.

Waqualbus said...

From what I understand, they were unable to put the Orbcomm sat in its proper orbit because the longer burn time increased the risk of the second stage hitting the ISS to an unacceptable level.

Tony said...

Byron:

"My guess as to a possible aerodynamic culprit is some quirk of aerodynamics that the CFD missed (or possibly a manufacturing error). Such things have finite precision, and wind tunnels don't scale perfectly. They designed it for 1.25x the CFD/tunnel predilections, and for whatever reason, it hit 1.5x in one spot. That broke, and it took the system with it. They reinforce and keep flying.
I'm not sure what in the engine could be the culprit."


I think you need to go back and look at the video. The obvious pieces of fairing debris depart the vehicle at T+00:1:20, after the equally obvious pressure release at T+00:1:19.

WRT what in the engine could have caused that sequence of events? I think I already stated that a seal could have failed on the downstream side of the turbopumps, or maybe something in the turbopumps themselves failed. It pretty obviously wasn't a gradual failure, like a leaking pipe.

"Taking a quick look through the list of current boosters in the same class, they all have a single engine on the first stage. So while it is inherent to the design, that design is not shared by other rockets. If there were other boosters with large numbers of first-stage engines, I might take issue with it, but there aren't."

Ummm...the point is that the engine-out capability is inherrent in multi-engine design philosophy, whether you're building a rocket or an airliner. Touting it as a substantial safety feature is nothing more -- or less -- than sophistry.

"You are correct that there are cases in which the loss of one engine would lead to the loss of the vehicle. But given that no other current booster can tolerate the loss of an engine at all, this does not strike me as terribly profound."

I don't know if it's a profound observation, or just a counterintuitive one, but single engine vehicles, while obviously not capable of engine-out mission recovery, have proven to be very reliable in practice. IMO it's because there are fewer moving parts and critical interactions than a multiple engine vehicle. Yes, single engine vehicles require engines that are large and more energetic than smaller engines, but they also focus attention on quality of design, construction, and operations. With multiple engines, one runs into the quandary of how much is good enough, per engine, vs how many engines can we lose and still make mission?

SpaceX chose multiple small engines because those engines were quicker and cheaper to develop to a flight-ready status. Okay, I can see the business advantage. But they may (once again I said may) have cut even more corners, thinking that they could withstand an engine loss in flight. The problem with that, if it indeed is the case, is that engine losses aren't always survivable simply on the basis of having more engines.

"Yes, it is. But I'm extending the presumption of competence until I get some hard evidence to the contrary."

As a space launch services customer, I don't think you'd take that attitude. You'd want to know exactly what the failure was, how it happened, and reserve your judgment until SpaceX could adequately demonstrate that they were on top of things.

"That actually worries me more than the loss of the first-stage engine. I'd imagine that they put a lot more work into the second stage engine than the first-stage ones, so having it fail to re-light is a bit worrisome. That said, it could have been a consequence of the first-stage failure. Until we know more, I'm withholding judgement."

I'm pretty sure from what I've read in the last 24 hours that it was a knock-on effect from the booster stage event. Now let's look at that as a secondary payload customer. See where I'm going?

Byron said...

Tony:
WRT what in the engine could have caused that sequence of events? I think I already stated that a seal could have failed on the downstream side of the turbopumps, or maybe something in the turbopumps themselves failed. It pretty obviously wasn't a gradual failure, like a leaking pipe.
I'm aware of that. My theory is that something at the top of the fairing failed a few seconds earlier, flailed around, took out the engine, and then the fairing came off.

Ummm...the point is that the engine-out capability is inherrent in multi-engine design philosophy, whether you're building a rocket or an airliner. Touting it as a substantial safety feature is nothing more -- or less -- than sophistry.
I might call it sophistry if there was another rocket in the last forty years that had the same capability. And I bet that the engine-out capability came up in the design meetings. Not the deciding factor, but an important one.

SpaceX chose multiple small engines because those engines were quicker and cheaper to develop to a flight-ready status. Okay, I can see the business advantage. But they may (once again I said may) have cut even more corners, thinking that they could withstand an engine loss in flight. The problem with that, if it indeed is the case, is that engine losses aren't always survivable simply on the basis of having more engines.
Yes, but as was demonstrated Monday, there are cases where the engine can fail and the flight is not lost. In a Delta or an Atlas if an engine goes, (even to a sensor glitch) the flight is done. It depends on how the numbers stack up. If the RUD (rapid unscheduled disassembly) rate is below 1/9th the total failure rate of other engines, then there is an advantage.

As a space launch services customer, I don't think you'd take that attitude. You'd want to know exactly what the failure was, how it happened, and reserve your judgment until SpaceX could adequately demonstrate that they were on top of things.
No, if I were flying, I'd want a full report before I flew a payload with them. But I'm an observer, not a customer.

I'm pretty sure from what I've read in the last 24 hours that it was a knock-on effect from the booster stage event. Now let's look at that as a secondary payload customer. See where I'm going?
Yes, I see where you're going. On the other hand, that's just a risk factor. If SpaceX is cheaper, do I take the risk of something like this happening, or do I pay more to remove it? Depending on what you're doing, it might or might not be worth it.

Tony said...

Byron:

"I'm aware of that. My theory is that something at the top of the fairing failed a few seconds earlier, flailed around, took out the engine, and then the fairing came off."

While that's remotely possible, IME rapid, complete pressure losses are caused by spontaneous component failures. IOW, things like head gaskets (and rocket pressure seals) don't blow because you throw rocks at a running engine.

"I might call it sophistry if there was another rocket in the last forty years that had the same capability. And I bet that the engine-out capability came up in the design meetings. Not the deciding factor, but an important one."

Oh, I'm sure an engine-out capability came up in every phase of the process, from conceptual design to final technical review. How could it not have, considering that aerospace engineers and multiple engine propulsion systems go all the way back to before WW2? It was simply not a choice. That's why pointing out the existence of an engine-out capability is so retarded. To somebody that knows even the basics of powered flight, it's like, "Yeah, so what else is new?"

"Yes, but as was demonstrated Monday, there are cases where the engine can fail and the flight is not lost."

No disrespect intended, but that's beside the point. The question is whether that particular failure mode is always [relatively] benign, and whether there are other dragons lurking in either the design or operation of the Merlin engine.

"In a Delta or an Atlas if an engine goes, (even to a sensor glitch) the flight is done."

Which leads to very high reliability engineering standards. This goes all the way back to the first rockets, by the way, most of which only had one or two engines per stage. Single engines are also considered adequate for many aircraft that fly people around every day. (Yes, aircraft have wings, and thus fail more gracefully than space launch vehicles, but an emergency landing is still a mission loss, if not a vehicle or a crew loss.)

"It depends on how the numbers stack up. If the RUD (rapid unscheduled disassembly) rate is below 1/9th the total failure rate of other engines, then there is an advantage."

As we have seen, that's not quite the case. There was a partial mission loss on this flight. If the flight had been only an ISS cargo run, and the propellant had been calculated as finely as it normally is, there might not have been a vehicle loss, but there could have been a mission loss. The way things work with rockets, the launch vehicle has to perform pretty damn near nominally for an unqualified mission success, regardless of design. And if you're a launch services customer, particularly on multiple payload flights -- which are a big part of the market these days -- unqualified success is what you're paying for.

"No, if I were flying, I'd want a full report before I flew a payload with them. But I'm an observer, not a customer."

But it's the customers that decide the future of SpaceX.

"Yes, I see where you're going. On the other hand, that's just a risk factor. If SpaceX is cheaper, do I take the risk of something like this happening, or do I pay more to remove it? Depending on what you're doing, it might or might not be worth it."

See above. Given the undeniable expense of spacecraft to begin with, the cost of schedule delays, and the limited number of rides to orbit at any price, you're pretty much paying for an unqualified success, aren't you?

Byron said...

Tony:
While that's remotely possible, IME rapid, complete pressure losses are caused by spontaneous component failures. IOW, things like head gaskets (and rocket pressure seals) don't blow because you throw rocks at a running engine.
Maybe. Maybe a hose was attached to the part of the fairing that failed. I'm not sure, and this scenario is mostly to look at possibilites other than the Merlin being the cause of the failure.

Oh, I'm sure an engine-out capability came up in every phase of the process, from conceptual design to final technical review. How could it not have, considering that aerospace engineers and multiple engine propulsion systems go all the way back to before WW2? It was simply not a choice. That's why pointing out the existence of an engine-out capability is so retarded. To somebody that knows even the basics of powered flight, it's like, "Yeah, so what else is new?"
And what percentage of the population knows the basics of powered flight? The situation is like an aircraft manufacturer touting the fact that their plane has multiple engines in case of failure. If the plane is an airliner, then this just gets them weird looks. If it's the F/A-18 competing with the F-16, it's a valid point. What premium the customer puts on it is another story.

No disrespect intended, but that's beside the point. The question is whether that particular failure mode is always [relatively] benign, and whether there are other dragons lurking in either the design or operation of the Merlin engine.
We have no clue what the failure mode is, or about any other possible "dragons".

Which leads to very high reliability engineering standards. This goes all the way back to the first rockets, by the way, most of which only had one or two engines per stage. Single engines are also considered adequate for many aircraft that fly people around every day. (Yes, aircraft have wings, and thus fail more gracefully than space launch vehicles, but an emergency landing is still a mission loss, if not a vehicle or a crew loss.)
And this never struck you as a potential bad thing? The costs of the extra reliability are significant, and this way is probably cheaper.

But it's the customers that decide the future of SpaceX.
And they will get access to all of the relevant data. They've worked a lot more closely with SpaceX than we have, so they probably have a better feel for what the company is like. I doubt that any of them are as upset as you are at this. If it's a bug of some sort, they'll fix it and it won't be an issue again.

See above. Given the undeniable expense of spacecraft to begin with, the cost of schedule delays, and the limited number of rides to orbit at any price, you're pretty much paying for an unqualified success, aren't you?
This depends on what you want to do. If Waqualbus is right, they probably make some flight dynamics changes and it's not a problem. If you just want something in orbit, then it's even less of one. (If you're doing, say, microgravity experiments, for example.)

I guess my problem is that you seem to be rushing to call it as a potential fatal flaw because you distrust Spacex, but we just do not have enough data to know anything at all yet. Further, I'd take issue with the attitude that big space is the only possible way, which is, IMO, the driver behind this.

Tony said...

Byron:

"Maybe. Maybe a hose was attached to the part of the fairing that failed. I'm not sure, and this scenario is mostly to look at possibilites other than the Merlin being the cause of the failure."

If they attached a critical propellant or hydraulics hose to the fairing, that would come under the rubric of pretty poor (I'm tempted to say "piss poor") design. The fairing is there to protect the engine from aerodynamic forces. Attaching a critical engine part to it would subject that part to unnecessary vibrations and flexion, as the fairing reacts to dynamic pressures during ascent.

In general, if the anomaly was detected in the engine, particularly with the spectacular accompanying visiual evidence, look at the engine first.

"And what percentage of the population knows the basics of powered flight? The situation is like an aircraft manufacturer touting the fact that their plane has multiple engines in case of failure. If the plane is an airliner, then this just gets them weird looks. If it's the F/A-18 competing with the F-16, it's a valid point. What premium the customer puts on it is another story."

I personally know plenty of people who have been on airplanes that suffered an engine shutdown in flight. It doesn't happen as much anymore, but it does happen, and most people know it can happen. Back in the day, four engines in an airliner were considered as much a safety advance as a payload carrying advance, because it made long flights over water safer. Heck, my mother, who flew in DC-6s across the Pacific in her teens, was the person who taught me about multi-engine aircraft and engine-out capabilities. The flying public just knew all about it, and engine-out flying was a pretty common thing into the Eighties.

So I think many more people -- and certainly people of an age today to be in a decision making position -- are quite aware of multi-engine operations and modes.

"We have no clue what the failure mode is, or about any other possible 'dragons'."

Which is exactly why the failure mode cannot be considered to be benign (for certain values of "benign"). Given the stakes, it has to be considered at least potentially mission-fatal until proven otherwise.

WRT to other undetected failure modes, all I'm saying is that the mere fact that an engine suffered a pressure loss failure in flight leads one to wonder.

"And this never struck you as a potential bad thing? The costs of the extra reliability are significant, and this way is probably cheaper."

I'm thinking in terms of vehicle reliability, in the specific sense of unqualified mission accomplishment. Remember, the present mission is only a qualified success, with the secondary payload capable of a very restricted alternate mission. And what I'm thinking is that maybe sufficient reliability at that level is a pretty straightforward function of investment.

Tony said...

Byron:

"This depends on what you want to do. If Waqualbus is right, they probably make some flight dynamics changes and it's not a problem. If you just want something in orbit, then it's even less of one. (If you're doing, say, microgravity experiments, for example.)"

What flight dynamics changes are they going to make that are going to solve or even mitigate low propellant levels? Even in the Apollo 13 engine shutdown, they had serious concerns about if their was enough propellant left in the S-IVB to accomplish a safe translunar injection. If the mission had been to a higher lunar latitude, they would probably have lost it.

And I really don't think there's such a thing as just wanting something in orbit. A customer wants a specific orbit, to meet specific experimental or operational objectives. If the launch vehicle doesn't deliver the payload to that orbit, it's a failure.

"I guess my problem is that you seem to be rushing to call it as a potential fatal flaw because you distrust Spacex, but we just do not have enough data to know anything at all yet. Further, I'd take issue with the attitude that big space is the only possible way, which is, IMO, the driver behind this."

Who said "fatal"? Certainly not me. I've been talking about:

1. A specific in-flight hardware failure,

2. How that led to an apparent shortcoming in mission accomplishment, and

3. How it might have been caused by SpaceX's business approach.

I have not once said that this is the end of SpaceX. If you want to pin me down on a prediction, it's this: SpaceX is going to find out that they haven't invested as much time, effort, and resources in reliability as they should have. They will then have to do so, in order to remain competitive with "big space".

IOW, it may just be that launch vehicle reliability is about investment in reliability. Period. I understand why you perceive this as an "attitude". That doesn't make it any less of a possibility.

Byron said...

Tony:
I personally know plenty of people who have been on airplanes that suffered an engine shutdown in flight. It doesn't happen as much anymore, but it does happen, and most people know it can happen. Back in the day, four engines in an airliner were considered as much a safety advance as a payload carrying advance, because it made long flights over water safer. Heck, my mother, who flew in DC-6s across the Pacific in her teens, was the person who taught me about multi-engine aircraft and engine-out capabilities. The flying public just knew all about it, and engine-out flying was a pretty common thing into the Eighties.
Please remember that I'm still (barely) short of 20. That sort of thing has never happened around me. I've heard of it, but these days it's very rare. And this is a rocket, not an airplane, and as I've previously pointed out, no other rocket today can survive an engine loss.

Which is exactly why the failure mode cannot be considered to be benign (for certain values of "benign"). Given the stakes, it has to be considered at least potentially mission-fatal until proven otherwise.
Yes, it does have that potential, and whatever caused it needs to be addressed. But the simple fact is that no other launcher today can survive an engine failure, and we should be careful not to judge SpaceX more harshly than we would if this had happened to a Delta.

WRT to other undetected failure modes, all I'm saying is that the mere fact that an engine suffered a pressure loss failure in flight leads one to wonder.
Because adequate testing catches everything, and the existance of a problem points to a systemic flaw. Have you never heard of bugs? Again, I think that there is a double standard here. The Delta IV also had an engine problem on launch 4, which lead to loss of the vehicle. We hadn't met then, but did you have the same reactions to that? And if you didn't, why not?

I'm thinking in terms of vehicle reliability, in the specific sense of unqualified mission accomplishment. Remember, the present mission is only a qualified success, with the secondary payload capable of a very restricted alternate mission. And what I'm thinking is that maybe sufficient reliability at that level is a pretty straightforward function of investment.
Again, this is flight four. For anything except a spacecraft, we'd still be in early testing. I would guess (though I can't give numbers to show for it) that getting 95% total accomplishment and 4% partial accomplishment is a lot cheaper than 98% total and 1% partial. To the point at which you carry insurance for the potential problems, and fly again if you need to.

What flight dynamics changes are they going to make that are going to solve or even mitigate low propellant levels? Even in the Apollo 13 engine shutdown, they had serious concerns about if their was enough propellant left in the S-IVB to accomplish a safe translunar injection. If the mission had been to a higher lunar latitude, they would probably have lost it.
Did you read what he said? According to him, the risk of hitting the station was too high. Not a problem with the propellant.

Byron said...

Tony:
And I really don't think there's such a thing as just wanting something in orbit. A customer wants a specific orbit, to meet specific experimental or operational objectives. If the launch vehicle doesn't deliver the payload to that orbit, it's a failure.
And yet NASA managed to sell Getaway specials. Sometimes, microgravity is all you want.

I have not once said that this is the end of SpaceX. If you want to pin me down on a prediction, it's this: SpaceX is going to find out that they haven't invested as much time, effort, and resources in reliability as they should have. They will then have to do so, in order to remain competitive with "big space".
Maybe fatal was a poor choice of words. The test program does look pretty meager compared to, say, the RD-180. Even then, I'm going to state that it's probably not systemic.

IOW, it may just be that launch vehicle reliability is about investment in reliability. Period. I understand why you perceive this as an "attitude". That doesn't make it any less of a possibility.
And some of that reliability might be some level of fault-tolerance. There are other ways than big space.

Byron said...

Looking more at the engine on the Delta IV (the RS-68), I think it got about 4x the test time of the Merlin (I haven't been able to find a good figure for the testing up to the first few launches). OTOH, it is the only engine on that stage. This is probably the best comparison, as it was a recent, from-scratch engine designed with cost in mind.

Tony said...

Byron:

"Please remember that I'm still (barely) short of 20. That sort of thing has never happened around me. I've heard of it, but these days it's very rare. And this is a rocket, not an airplane, and as I've previously pointed out, no other rocket today can survive an engine loss."

Rocket or aircraft, the principle is the same -- don't lose the vehicle to a single engine failure. (But there's a bit of a problem with that philosophy where rockets are concerned -- stay tuned.)

I really don't want to go into age and experience again, but in-flight engine failures were still pretty common as recently as 15-20 years ago. And most people in decision making capacities today were young adults or older back then.

"Yes, it does have that potential, and whatever caused it needs to be addressed. But the simple fact is that no other launcher today can survive an engine failure, and we should be careful not to judge SpaceX more harshly than we would if this had happened to a Delta."

Surviving an engine failure doesn't mean what you appear to think it does. With launch vehicles, the vehicle is going to be expended anyway -- is in fact effectively expended the second it leaves the pad. So mere vehicle survival until the propellant runs out is not a decisive factor.

The real issue is how an engine-out condition affects orbital insertion, both in terms of altitude and shape. Because you're working against gravity with a launch vehicle, all modes of burning a given amount of propellant are not the same. In fact, an engine-out capability is only really helpfull towards the end of a burn, where the difference in lower thrust performance is within the contingency margin(s) of the next stage(s). As the fate of the Orbcomm payload demonstrates, that's not generally the case.

"Because adequate testing catches everything, and the existance of a problem points to a systemic flaw. Have you never heard of bugs?"

More testing catches more things -- or at least rules them out -- as a general rule. Try arguing against that proposition with your engineering profs and tell me how far you get.

And any problem could be systemic. Almost all problems in fact are, because a competent system is supposed to catch out of tollerance components that sneak in, just as much as it's supposed to be designed to do a certain set of tasks.

Most of the time, a system can be fixed. Sometimes it's the system design that's just plain wrong. I'm not delivering judgment right now on SpaceX's hardware, software, or procedural systems just yet. But they've got a problem within one or more of those systems that needs to be fixed.

"Again, I think that there is a double standard here. The Delta IV also had an engine problem on launch 4, which lead to loss of the vehicle. We hadn't met then, but did you have the same reactions to that? And if you didn't, why not?"

I don't think I thought much about it at the time. Boeing has a really good track record with rockets, and if I thought anything, I would have thought that they would spend the money to fix the problem. Also, the Delta IV anomaly was not the failure of an engine to contain pressure. It was an internal environment condition that caused eroneous sensor readings, leading to a premature shutdown of otherwise nominally operating engines. So it was a different and less troubling species of failure.

I think the point to be made is that Boeing had earned confidence, because Delta IV was far from their first rocket. SpaceX? Not so much -- at least not at this point.

Tony said...

Byron:

"Again, this is flight four. For anything except a spacecraft, we'd still be in early testing. I would guess (though I can't give numbers to show for it) that getting 95% total accomplishment and 4% partial accomplishment is a lot cheaper than 98% total and 1% partial. To the point at which you carry insurance for the potential problems, and fly again if you need to."

The problem here is that with launch vehicles, money is not an object when reliability is on the line. The insurance premium alone on a 3% delta in claims, when a claim run $100M or more, is probably several million dollars per launch. Then there are the uninsured losses, such as lost profits due to shcedule slip, and loss of confidence due to failure to perform (both for the launch services customer and launch services provider). I'm just guessing, but I bet a full cost analysis of even just a that 3% lower nominal performance rate would show that not much money is being saved at all.

"Did you read what he said? According to him, the risk of hitting the station was too high. Not a problem with the propellant.""

That's actually not what Orbcomm is saying:

http://www.redorbit.com/news/space/1112710518/spacex-falcon-orbcomm-orbit-101012/

They're saying that the satellite is in the wrong orbit, period. Why that is could indeed be a mission safety rule determination. But that rule would not have had to be invoked if the launch vehicle had performed nominally. That is the bottom line.

I think you'll learn throughout your engineering life, the bottom line is what drives customers, not intermediate line items.

"And yet NASA managed to sell Getaway specials. Sometimes, microgravity is all you want."

That sector is hardly a market driver.

Tony said...

Byron:

"Maybe fatal was a poor choice of words. The test program does look pretty meager compared to, say, the RD-180. Even then, I'm going to state that it's probably not systemic."

See my previous comments on failures in systems. If it was a design failure, a manufacturing failure, or a procedures failure, it can most often be traced to a problem with some kind of system. Very rarely is there something wrong that can't be caught with enough attention to detail.

"And some of that reliability might be some level of fault-tolerance. There are other ways than big space."

Once again, see previous comments. The launch vehicle is expended no matter how you cut the pizza pie. So, unlike manned aircraft, preserving the vehicle for a few continued minutes of operation simply isn't an objective. Getting the payload to the specified orbit is the only objective of any value. Yes, the Dragon mission was preserved, but only because there was margin in the flight plan for a secondary mission that didn't succeed. A monolithic payload of the same aggregate mass, with the same propellant loading, would have failed to reach the specified orbit, to some degree. Thinking in terms of a more fully loaded Dragon capsule to the ISS, the delivery mission might have failed altogether.

If fault tolerance is the anser for SpaceX, they'll have to de-rate their payload capacity, in order to carry enough margin for recovery from future faults. Or they'll have to eliminate fault tolerance from their thinking. Either is going to drive up their prices, on a per-pound basis.

And I will say "I told ya so" on that, because I in fact have. SpaceX operates in the same physical universe as "big space" (what a thought-terminating cliche that is...). That means they're subject to the same economic constraints. If they want to compete at the same level of reliability -- and they probably will want that, in the end, because launch services customers aren't stupid -- they'll have to pay the same price, because the established launch services providers are neither incompetent nor profiteers.

Looking more at the engine on the Delta IV (the RS-68), I think it got about 4x the test time of the Merlin (I haven't been able to find a good figure for the testing up to the first few launches). OTOH, it is the only engine on that stage. This is probably the best comparison, as it was a recent, from-scratch engine designed with cost in mind."

I think I've belabored the point enough about engine-out capability not being as big a deal as percieved.

So I'll jsut make a general comment here. SpaceX, in both business philosophy and marketing, reminds me of nothing so much as "Faster, Better, Cheaper". And I think we learned from that that you can't do space exploration on a budget. I think that we may (as always, the qualification is may) be finding out that you can't do space launch services on a budget either.

Byron said...

Tony:
I really don't want to go into age and experience again, but in-flight engine failures were still pretty common as recently as 15-20 years ago. And most people in decision making capacities today were young adults or older back then.
Yes. And I basically don't remember that time period.

The real issue is how an engine-out condition affects orbital insertion, both in terms of altitude and shape. Because you're working against gravity with a launch vehicle, all modes of burning a given amount of propellant are not the same. In fact, an engine-out capability is only really helpfull towards the end of a burn, where the difference in lower thrust performance is within the contingency margin(s) of the next stage(s). As the fate of the Orbcomm payload demonstrates, that's not generally the case.
I am also aware of this. SpaceX is going to have to build in a margin for this stuff, and that will raise costs.

More testing catches more things -- or at least rules them out -- as a general rule. Try arguing against that proposition with your engineering profs and tell me how far you get.
I never said it didn't. But if we go that way, then we as engineers will never do anything. Ever. Becuase it will always need more testing.

And any problem could be systemic. Almost all problems in fact are, because a competent system is supposed to catch out of tollerance components that sneak in, just as much as it's supposed to be designed to do a certain set of tasks.
Yes. But given that this is the real world, we have to get as close to perfect as possible, and hope. SpaceX may or may not have cut the testing margins too tight. I'm not going to guess one way or the other until I have more data.

Most of the time, a system can be fixed. Sometimes it's the system design that's just plain wrong. I'm not delivering judgment right now on SpaceX's hardware, software, or procedural systems just yet. But they've got a problem within one or more of those systems that needs to be fixed.
The same is true of any engineering failure. This is what I've been saying, but from a slightly less anti-SpaceX standpoint.

I think the point to be made is that Boeing had earned confidence, because Delta IV was far from their first rocket. SpaceX? Not so much -- at least not at this point.
Yet the biggest space debacles in history were both on a vehicle built by a company with a similar rocket pedigree to Boeing.

Why that is could indeed be a mission safety rule determination. But that rule would not have had to be invoked if the launch vehicle had performed nominally. That is the bottom line.
And if that is the case, they alter the launch window slightly to prevent it from happening again. That way the stage can safely send the secondary payload where it needs to go.

Byron said...

Tony:
That sector is hardly a market driver.
No, but it would make decent secondary payloads under the constraints of engine-out.

If fault tolerance is the anser for SpaceX, they'll have to de-rate their payload capacity, in order to carry enough margin for recovery from future faults. Or they'll have to eliminate fault tolerance from their thinking. Either is going to drive up their prices, on a per-pound basis.
I know that, but that still leaves them a fair bit cheaper, IIRC.

SpaceX operates in the same physical universe as "big space" (what a thought-terminating cliche that is...).
I decided to use "Big Space" as a shorthand for the existing launch companies, instead of typing that out. I'm not SpaceXs biggest fan. Elon Musk does not walk on water. I am defintiely bothered by what happened, but we should give them the benefit of the doubt until we know more.

So I'll jsut make a general comment here. SpaceX, in both business philosophy and marketing, reminds me of nothing so much as "Faster, Better, Cheaper". And I think we learned from that that you can't do space exploration on a budget. I think that we may (as always, the qualification is may) be finding out that you can't do space launch services on a budget either.
I really need to do some research on this one. I'm far from convinced that we can't bring down the cost of space launch significantly. SpaceX isn't magic, and they won't do as well as they hope to. I know that. But will there be some drop in costs? Probably.

Tony said...

Re: Byron

I think we're in violent agreement here, with a relatively minor disagreement on emphasis. So I'll just make a couple of comments:

1. Testing. Yes, there is a time where one has to say enough is enough. But there are also times when business gets in the way of engineering (with NASA as much as a commerical spacecraft designer -- see building the Shuttle both to budget and unreasonable technical specifications, then ignoring flight safety issues emerging during operations), and "enough" really isn't enough.

2. "[A]nti-SpaceX standpoint". I'm really not anti SpaceX. I'm pretty firmly anti Elon Musk, but I think SpaceX, if it is to survive -- which I fully expect it to do -- will have to start operating like a real launch services provider sooner rather than later. I'm actually hopeful that the even under discussion is in fact the sentinel event in that respect.

3. WRT Shuttle, IIRC Boeing was in fact the primary systems integrator. They in fact got 133/135 good flights out of the system. 98.5% mission success rate for such a long track record is right up there with the best.

Apropos engine-out capabilities, Shuttle had one too, though it was never exercised in flight. Also, it was understood that the capability only supported, at best, abort to orbit and an off-nominal alternate mission.

Byron said...

Tony:
I think we're in violent agreement here, with a relatively minor disagreement on emphasis.
This seems to happen a lot. I don't disagree at all on this point, nor is my agreement in any way violent.

1. Testing. Yes, there is a time where one has to say enough is enough. But there are also times when business gets in the way of engineering (with NASA as much as a commerical spacecraft designer -- see building the Shuttle both to budget and unreasonable technical specifications, then ignoring flight safety issues emerging during operations), and "enough" really isn't enough.
Exactly. My point was that the test program looks thin, but not Kerbal Space Program thin.

2. "[A]nti-SpaceX standpoint". I'm really not anti SpaceX. I'm pretty firmly anti Elon Musk, but I think SpaceX, if it is to survive -- which I fully expect it to do -- will have to start operating like a real launch services provider sooner rather than later. I'm actually hopeful that the even under discussion is in fact the sentinel event in that respect.
I suppose I confused the two. What exactly is your problem with him? (That's curiosity, not belligerence, BTW.)

3. WRT Shuttle, IIRC Boeing was in fact the primary systems integrator. They in fact got 133/135 good flights out of the system. 98.5% mission success rate for such a long track record is right up there with the best.
I was thinking North American/Rockwell. And given that it was a very babied reusable launch vehicle (which means that most hardware wasn't flying for the first time) that's not terribly impressive.

Apropos engine-out capabilities, Shuttle had one too, though it was never exercised in flight. Also, it was understood that the capability only supported, at best, abort to orbit and an off-nominal alternate mission.
Actually, they did use it on STS-51-F. They had an SSME sensor failure at 5:45, and did an abort to orbit. Another engine nearly went, which would have put them into a transatlantic abort, or possible a once-around (if I read the timing right). It was a Spacelab mission, so they were able to do most of what they wanted to do. Including the testing of special zero-G Coke and Pepsi cans.

Tony said...

Byron:

"This seems to happen a lot. I don't disagree at all on this point, nor is my agreement in any way violent."

It's just an expression. The "violent" means -- to me at least -- that we both mean business and mean to be taken seriously when we say something.

"Exactly. My point was that the test program looks thin, but not Kerbal Space Program thin."

No, not that thin. But I'm a big believer that one gets what he pays for, and no more.

"I suppose I confused the two. What exactly is your problem with him? (That's curiosity, not belligerence, BTW.)"

Too much of a promoter and not enough of an engineer. Which may be the Pot beating up on the Kettle, given that my own "engineering" is stricly software. But you asked, and it's what I think.

"I was thinking North American/Rockwell. And given that it was a very babied reusable launch vehicle (which means that most hardware wasn't flying for the first time) that's not terribly impressive."

I actually think it's more impressive, because expendables only have to perform once, for a few minutes per stage. The Shuttle had to keep on keeping-on for subsequent flights -- technologically impressive, even if economical lunacy.

BTW, Shuttle is the example I would give for an inherrently flawed system design. They couldn't recover the crew from a vehicle loss during powered ascent, except in the most unlikely circumstances (the Shuttle astronauts knew this and lived with it). And the failure modes in both vehicle losses were inherrent in the system design, and irremediable, within the context of the system.

"Actually, they did use it on STS-51-F. They had an SSME sensor failure at 5:45, and did an abort to orbit. Another engine nearly went, which would have put them into a transatlantic abort, or possible a once-around (if I read the timing right). It was a Spacelab mission, so they were able to do most of what they wanted to do. Including the testing of special zero-G Coke and Pepsi cans."

You know, I thought I renembered that, but I couldn't find the reference in the time I alloted to look (speaking of getting what you pay for...), so I went with the safe claim.

Ya know, I thought the soda in space was kind of groovy, by the Rule of Cool, even if it was a taste failure and ultimately frivolous.

Byron said...

Tony:
It's just an expression. The "violent" means -- to me at least -- that we both mean business and mean to be taken seriously when we say something.
I know what you meant. Sort of a joke.

Too much of a promoter and not enough of an engineer. Which may be the Pot beating up on the Kettle, given that my own "engineering" is stricly software. But you asked, and it's what I think.
I don't think he's any sort of engineer. Not to say he's not smart, but his engineering degree is honorary.

BTW, Shuttle is the example I would give for an inherrently flawed system design. They couldn't recover the crew from a vehicle loss during powered ascent, except in the most unlikely circumstances (the Shuttle astronauts knew this and lived with it). And the failure modes in both vehicle losses were inherrent in the system design, and irremediable, within the context of the system.
Totally in agreement here. The video lecture series has shed light on this. They could have done it a lot better if they'd been slightly less cost-constrained at the beginning.

Ya know, I thought the soda in space was kind of groovy, by the Rule of Cool, even if it was a taste failure and ultimately frivolous.
And we can put it on our Space Resume.
Achievements:
First Man on the Moon.
First Reusable Launch Vehicle.
First Soda in Space.

Tony said...

Byron:

"I don't think he's any sort of engineer. Not to say [Elon Musk]'s not smart, but his engineering degree is honorary."

Actually, he does have an earned bachelors degree in physics, so he probably does understand when his engineers are talking to him about theory. How well he does comprehending application? Who knows? But in the real world there aren't any spherical launch vehicles in a vacuum.

"[The Shuttle desingers] could have done it a lot better if they'd been slightly less cost-constrained at the beginning."

Hmmm...yeah, if they had used the money to put the Shuttle as a payload on top of an expendable booster. If they just made the original sin of parallel stacking more reliable through investment in, well, reliability, I think I'd still have to give them a failing grade on conceptual design, even if they returned fewer dead astronauts.

Tony said...

Oh, by the way, I wanted to congratualte Rick on his boyz advancing in the MLB playoffs.

With the Kung Fu Panda and a guy named Buster, how can you lose?

Byron said...

I just found out that speculation is centering on the fuel dome fracturing as the cause of the problem. I'm not sure exactly what this means (though I may hit the library tonight to see what I can find.) The dome either cracked because of a manufacturing error, insufficent testing or some sort of fluke. Their testing should have caught an error, but we don't know how their testing compares to, say, Rocketdyne, and probably never will. It's possible the engine is grossly flawed, but I'm betting construction error. To support you, I asked my roomate what he thought (I told him that the speculation was a fuel dome cracking. He said they should have caught it, and probably didn't test right.) He's studying metalurgy, so while it's not as good as going to a professor, he knows more than you or I.

Actually, he does have an earned bachelors degree in physics, so he probably does understand when his engineers are talking to him about theory. How well he does comprehending application? Who knows? But in the real world there aren't any spherical launch vehicles in a vacuum.
I know, but that's not the same thing. Could he have been an engineer? Yes. But taking a quick look at what a Physics Major at Missouri S&T would have to do, it's not even remotely the same as an engineering degree. Double-majoring would be no easier than doing so from Biology or Math. And his big success was as a software enterpernure. No offense, but that's a lot different from real (physical) engineering.

Hmmm...yeah, if they had used the money to put the Shuttle as a payload on top of an expendable booster. If they just made the original sin of parallel stacking more reliable through investment in, well, reliability, I think I'd still have to give them a failing grade on conceptual design, even if they returned fewer dead astronauts.
Best thing would've been the design with the shuttle and tank atop the S-IC. Modify it for flyback, and it gets reused. Though it probably wouldn've turned out fine sans insane payload and crossrange requirements. What's really funny is to consider that the Soviets built Buran to a nearly identical design, when they didn't even need the crossrange. Just launch out of, say, Vladivistok, and come down somewhere in the middle of Siberia. Of course we could have launched overland too, and done the same thing.

Byron said...

Oh, I shoud add that I haven't talked to my roomate about this beforehand. I'm still not 100% convinced they were negligent, but the meter is maybe at 50-50.

Tony said...

Byron:

"I just found out that speculation is centering on the fuel dome fracturing as the cause of the problem. I'm not sure exactly what this means (though I may hit the library tonight to see what I can find.) The dome either cracked because of a manufacturing error, insufficent testing or some sort of fluke. Their testing should have caught an error, but we don't know how their testing compares to, say, Rocketdyne, and probably never will. It's possible the engine is grossly flawed, but I'm betting construction error. To support you, I asked my roomate what he thought (I told him that the speculation was a fuel dome cracking. He said they should have caught it, and probably didn't test right.) He's studying metalurgy, so while it's not as good as going to a professor, he knows more than you or I."

It means the top of the engine came apart. See: http://heroicrelics.org/info/f-1/f-1-injector.html. (It's about the F-1 engine, but it gives you an idea what "dome" means in connection with liquid rocket engines.) It's still just speculation, but ouch. I don't know what the SpaceX customers are going to insist on if this is indeed the case, but in the past customers have wanted the manufacturer to prove his theory of the failure by recreating it on the test stand, then proofing a fix with more test stand work.

"I know, but that's not the same thing. Could he have been an engineer? Yes. But taking a quick look at what a Physics Major at Missouri S&T would have to do, it's not even remotely the same as an engineering degree. Double-majoring would be no easier than doing so from Biology or Math. And his big success was as a software enterpernure. No offense, but that's a lot different from real (physical) engineering."

No offense taken.

First of all, my associates is in physical sciences, but my upbringing was in the aerospace industry on my dad's side of the family, and hard rock mining on my mom's. The distinction between theory and application is almost literally mother's milk to me.

Second, I'm one of those programmers that thinks software is much more craft than science (even though we do OD on math from time to time). That's why I put "engineering" in quotes when I related it to software. I think the engineering approach to software is pure bogosity.

"Best thing would've been the design with the shuttle and tank atop the S-IC. Modify it for flyback, and it gets reused. Though it probably wouldn've turned out fine sans insane payload and crossrange requirements. What's really funny is to consider that the Soviets built Buran to a nearly identical design, when they didn't even need the crossrange. Just launch out of, say, Vladivistok, and come down somewhere in the middle of Siberia. Of course we could have launched overland too, and done the same thing."

According to Roald Sagdeyev, the Soviets built the Buran out of an excess of magical thinking WRT American technology. Even if (perfectly good) Soviet science told them that it was a dumb idea, they couldn't get past the fact that we were doing it -- and if Americans were doing it, there had to be something in it.

Byron said...

Tony:
It means the top of the engine came apart. See: http://heroicrelics.org/info/f-1/f-1-injector.html. (It's about the F-1 engine, but it gives you an idea what "dome" means in connection with liquid rocket engines.) It's still just speculation, but ouch. I don't know what the SpaceX customers are going to insist on if this is indeed the case, but in the past customers have wanted the manufacturer to prove his theory of the failure by recreating it on the test stand, then proofing a fix with more test stand work.
I ran across a similar diagram, and figured out basically the same thing, after I typed that up, and I forgot to go back and revise it. I concur with your conclusions WRT what SpaceX will have to do.

Second, I'm one of those programmers that thinks software is much more craft than science (even though we do OD on math from time to time). That's why I put "engineering" in quotes when I related it to software. I think the engineering approach to software is pure bogosity.
My approach to software (they make us take a programming class here) is "does it work?" If not, tinker until it does. This saves time for more important (and fun) things.

According to Roald Sagdeyev, the Soviets built the Buran out of an excess of magical thinking WRT American technology. Even if (perfectly good) Soviet science told them that it was a dumb idea, they couldn't get past the fact that we were doing it -- and if Americans were doing it, there had to be something in it.
That's my impression as well. I was pointing out that even if they decided they needed an RLV, the constraints we had imposed didn't apply to them, so they didn't have to go with a delta. One of these days I need to track down a decent history of the cold war, particularly the end of it.

Byron said...

Doing a little more research, apparently the typical design margin for a thrust chamber is only 1.2. This is from Sutton's Rocket Propulsion Elements which is the bible on the subject. Also, the chamber itself has a limited fatigue life. At a guess, there might have been a small flaw, which slipped through testing, then got fatigued into failing early. No real proof, but it seems plausible. And it was the number one engine. Could be just a coincidence, but I'd assume that, due to humans being humans, there's a good chance it got more tests than the other ones. More tests, more fatigue cycles, and so on.
Or it could be that it was undetectable and the engine had little time on it. Or SpaceX could be way unsafe. It just seems fairly plausible.

Anonymous said...

About Buran, while I agree somewhat with the "magical American thinking says it's true/ Americans are doing it" theory on why they started it in the first place, it diverged in a big way from the American ideas. While many things about Buran are cribbed from the American designs, the reusable engines were not. Buran was launched by an expendable Energia LV. Buran carried no Soviet counterpart to the SSME, and can't really be considered as integral to the stack like the US shuttle was. Essentially Buran is just a big payload for the Energia, one of what could have been many. While versions of a Shuttle-C/Shuttle-Derived have been proposed over the years the Soviets built their shuttle with heavy lifting parts separate from the shuttle ready for use. Energia actually flew first without Buran, lifting Polyus. Polyus failed to finish the job though, and reentered; Energia was designed to put ~100 tons of payload nearly to orbit, with the idea the payload would include something to provide the last bit of impulse to achieve orbit. Polyus was equipped to do that, but fired in the wrong direction, so it stayed suborbital and burned. Despite Polyus failing, Energia got good marks for that mission, and when it boosted Buran too. There were plans to put something like 32 tons in lunar orbit using Energia, with some sort of upper stage I haven't been able to find info on.

Instead of being part of a transport system like our Shuttle was, Buran was the payload of a rocket(Energia). It seems to me that the Soviet military would get a shuttle like the Air force got, Soviet politicians could say that their countries were as good(or better) as the US in engineering, and Soviet engineers and space planners would get a ~100 LV that was important for reasons of strategic parity. Buran more or less had to be like the American shuttle, similar wings, payload bay, and so on. But there wasn't a requirement to put the main engines on the shuttle, either from the military or engineers. I think something like what happened to produce the Almaz space stations happened to Buran and Energia: Soviet engineers would like a space station, and nobody gives them a damn. Americans propose MOL, the Soviet military wanted something similar, the Soviet government wants to be seen as better than America(as always), so a plan is hammered out by the groups. Salyuts(civilian space stations) are made to cover up the military Almazes, and a first in space is made on April 19, 1971. Nearly everybody gets what they came for. Except the military, as men tending to the cameras isn't as good as remotely tending to the cameras. But they did get the chance to try the idea out, unlike their American counterparts, which is a plus or a minus depending how you look at it.

So the Soviet military wanted a shuttle, the Soviet government didn't want to be left behind by American technology, and Soviet engineers had unfulfilled moon plans. It fell apart in the end because of the great economic implosion, but the result is more interesting than simply "Soviets copied the Americans and failed". I hope I did it justice, but feel free to call me out on anything you see as wrong.

OrbitingPluto

Byron said...

OrbitingPluto:
As far as I know, you nailed it. A couple of elaborations, though:
1. I think that, as much as anything, the reason the Soviets went with expendable engines was because they couldn't duplicate the SSME. It's a magnificent technological feat, though ultimately the wrong answer to the problem.
2. The aerodynamics involved are frankly funny. The shuttle's use of a delta wing was driven by the need for crossrange so they could land at Vandenberg from a single polar orbit. I think that the Soviets could have hit friendly territory with a lot less from Bikanour (although it might have been in Poland), and they could always have built the pad in Vladivostok. You could go two or three orbits before landing.
2. Polyus was supposed to be a prototype orbiting laser weapons platform. And it only had one laser. Take that, Tony! :-)

Byron said...

Oops. I miscounted my points there. It's probably time for bed.

Anonymous said...

@Byron

1. The RD-0120s the Soviets used on the Energia core are in the same ballpark as a SSME. Both were built to do similar jobs up to MECO. The fact the SSME gets used again doesn't detract anything from the RD-0120. A reusable RD-0120 could have been made, and there were plans for a reusable Energia core, so maybe there is a plan in Russian somewhere detailing the changes needed. But they didn't do it in real life, so even if they could it is only a paper engine, whereas the SSME is real hardware.

2.That is true to a point, but think about the prestige angle. Buran was built to have slightly better capabilities than the American shuttle. More crossrange, bigger payload lofted, bigger payload returned, and so on. They went in on Buran to show they were better. You can't say your shuttle is better without some better numbers. It's like a arms race; it doesn't matter what he has, ours will be bigger. No matter how silly.

3. I don't know what your getting at.

OrbitingPluto

Byron said...

OrbitingPluto:
1. The RD-0120s the Soviets used on the Energia core are in the same ballpark as a SSME. Both were built to do similar jobs up to MECO. The fact the SSME gets used again doesn't detract anything from the RD-0120. A reusable RD-0120 could have been made, and there were plans for a reusable Energia core, so maybe there is a plan in Russian somewhere detailing the changes needed. But they didn't do it in real life, so even if they could it is only a paper engine, whereas the SSME is real hardware.
The part about it being reusable is mostly what I meant. The Russians make good engines, don't get me wrong, but the SSME is about the best you can get with a conventional rocket.

2.That is true to a point, but think about the prestige angle. Buran was built to have slightly better capabilities than the American shuttle. More crossrange, bigger payload lofted, bigger payload returned, and so on. They went in on Buran to show they were better. You can't say your shuttle is better without some better numbers. It's like a arms race; it doesn't matter what he has, ours will be bigger. No matter how silly.
Indeed. But that was the problem of the Soviet system. They were playing to our media. It cost a bunch of money, and didn't really do anything.

3. I don't know what your getting at.
Sorry. Sort of a private joke. Look up 'laserstar' here if you're interested.

Tony said...

Re: Buran

1. If the Soviets wanted a heavy lifter, there was no reason the payload had to be a reusable winged vehicle, nor such a large one. Designing Energia for a side-mounted payload was driven by the existence of Buran.

2. I was just reporting what Roald Sagdeev (a big wheel in Soviet spaceflight) said. I happen to think it has a ring of truth to it, but that's beside the point.

Re: SpaceX Merlin engine failure

Believe it or not, even if their power head did come apart in flight, I'm going to reserve judgment on the ultimate cause till somebody reports out their findings.

Leaving judgment aside, and entering into the realm of speculation, if it was a material failure, that's really a big downcheck on SpaceX from a procedures standpoint. Why do I say that? When I was in college, I knew a lady who had been a quality control technician in the Air Force. She routinely did x-ray inspection on airframe and engine parts, because, well, you know -- f*ck a bunch of critical structures failing in flight. Now, apply that same lattitude to rocket engine parts, and one wonders how SpaceX could let a material fault past their quality control.

And that's the good news. If there's a design flaw in the Merlin engine, well...back to the drawing board is not what you want to be doing when you're already into revenue operations.

In any case, how SpaceX responds over the next few months is going to be an interesting saga, one way or the other.

s337101 said...
This comment has been removed by the author.
Locki said...

1. Shuttle Cross range requirement

This has always puzzled me. Exactly why did the USAF want a 1000nm cross range performance? It made the shuttle almost fatally fragile. Can anyone speculate what military purpose this may have served? Was the ability ever actually used?

2. Tony said

SpaceX operates in the same physical universe as "big space" (what a thought-terminating cliche that is...). That means they're subject to the same economic constraints. If they want to compete at the same level of reliability -- and they probably will want that, in the end, because launch services customers aren't stupid -- they'll have to pay the same price, because the established launch services providers are neither incompetent nor profiteers.

========

This is exactly how I feel about SpaceX. Most of my friends are in IT and their general assumption is the smartest dudes in the world work in IT. SpaceX is destined for victory because we finally have the smartest, most innovative dudes tackling this age-old engineering problem. Their usual argument runs something like this. "Look at how clever John Carmack /Romero (ID software) etc etc is. The government spent bajillions and decades designing this useless piece of outdated software and Carmack coded it in a single night of red bull powered brilliance. Surely he'll bring his genius to bear on the aerospace industry and we'll see the same cost savings. Well all be holidaying on Mars in 10 or so years. Hoorah!"

I spent literally weeks taking on every programmer in the pub about their misguided hubris. Software development lends itself to a hacking mentality. Other fields of human endeavour tend to be less well suited to this move fast and break things mentality. I believe aerospace engineering or really any sort of engineering to be a field ill-suited to the hacker mentality.

The smartest, most dedicated engineers from 5 different cultures have been working on this problem for over half a century. I don't think a couple of swaggering IT venture capitalists are going to suddenly find cost savings and efficiencies everyone has mysteriously overlooked.

On a slightly related note. I think the tesla's of the world are going to fall equally flat in bringing electric cars to market. The silicone valley capitalist model works great for software but cars and planes are a different story.


3. Manned Mars Mission tangent

It was a rather natural place to bring up a manned Mars mission. For the lay person like me the enormity of the task in sending several humans to the surface of Mars and bringing them back alive with about 90% certainty is hard to imagine. When I read through Curiosity's mission in detail and realised the complexity and expense needed to send a 1 tonne robotic probe on a one way mission to Mars it suddenly dawned on me that its highly unlikely we'll see a human on Mars in our lifetimes.


4. Romance of space travel

I think the romance of sending a live person to a dead rock is overstated. The world got pretty bored of the Moon pretty quickly after Apollo 11.

Ideas of romance and heroism change to suit the times. I liked reading about the dawn of the gunpowder age. Once upon a time romantic bravery was facing your foe eyeball to eyeball and beating him down personally. Gradually the military class (read knights) realised this was suicide and bravery came to be represented by the Officer class casually ignoring the shrapnel/musket rounds and calmly directing their troops.

As the difficulties of space travel become apparent to the general public maybe our romantic ideas of space travel will change in a similar way too. I personally think trying to send an astronaut to Mars with our current technology level (chemical rockets, 2 year round trip) is about as suicidal as charging a Redcoat Infantry Square. Maybe we'll evolve and learn to celebrate the heroic mission commander at Houston who calmly directs/programs his probes on their search for life around Europa/Enceladous.

Byron said...

Tony:
Believe it or not, even if their power head did come apart in flight, I'm going to reserve judgment on the ultimate cause till somebody reports out their findings.
I need to do the same (though a working theory might make that a bit tricky).

Leaving judgment aside, and entering into the realm of speculation, if it was a material failure, that's really a big downcheck on SpaceX from a procedures standpoint. Why do I say that? When I was in college, I knew a lady who had been a quality control technician in the Air Force. She routinely did x-ray inspection on airframe and engine parts, because, well, you know -- f*ck a bunch of critical structures failing in flight. Now, apply that same lattitude to rocket engine parts, and one wonders how SpaceX could let a material fault past their quality control.
You're correct. This sort of thing does not reflect well on SpaceX, and if it's the case, they'll have to do a lot to fix their reputation.

Locki:
This has always puzzled me. Exactly why did the USAF want a 1000nm cross range performance? It made the shuttle almost fatally fragile. Can anyone speculate what military purpose this may have served? Was the ability ever actually used?
Once-around out of Vandenberg. Take off, go into polar orbit, launch a satellite, land, all in one orbit. The crossrange lets them get back.

I really like the section on the "hacker mentality".

The smartest, most dedicated engineers from 5 different cultures have been working on this problem for over half a century. I don't think a couple of swaggering IT venture capitalists are going to suddenly find cost savings and efficiencies everyone has mysteriously overlooked.
I think there are a few savings, but not more than maybe 25% (which still leaves Proton in the lead).

Tony said...

Let's see...

1. Yeah, the cross range requirement was in support of a once-around return to launch site capability, nominally during wartime.

2. Hacker mentality. As a programmer who works in the nitty gritty of business information, not in making snazzy web pages or games, that mentality is just effing infuriating. All programmers work on the build it, break it, fix it model these days, because compute cycles are so cheap. Mechanical engineers do a lot of that too during development. But, in mechanical engineering, there's a time when the sh!t just has to work. And that takes a substantial investment in making sure it will work. (Happens in software too, but most programmers these days ain't working in that end of the biz.)

3. On the mechanical engineering of high performance rocket engines. Margin is a really ticky subject with rocket engines, because the more margin you put in, the heavier the thing gets. and if you make it heavier, you either have to build a bigger vehicle for a given payload, or carry less payload. Both have obvious costs. So engines are made as light as possible.

On the other hand, these engines do have to survive at least one factory test firing (and possibly more, if they have to be tweaked prior to installation on a vehicle), possibly a test firing on the vehicle out at the pad (SpaceX actually did this about a week before launch, on the vehicle in question), and a service firing during an actual flight. Back in the day, some restartable engines were run up to ten times service burn time with only moderate inspection and maintenance. I would think with non-restartables, one should be able to get at least 5x nominal service burn time out of an engine before you condemn it.

All of which may be irrelevant if the particular engine had not been run over historical test burn times prior to use on a flight. Then you have to start looking at faulty component and design issues.

If the engine was a long runner, or at least near the top of the fleet long run times, one has to ask what the deal was with that. Were they pushing the envelope on their safe burn time experience on flight hardware? That would be pretty, ummm...insane.

Byron said...

Tony:
Mechanical engineers do a lot of that too during development. But, in mechanical engineering, there's a time when the sh!t just has to work. And that takes a substantial investment in making sure it will work.
Absolutely. The only time that I might consider software to be truly engineering is when it absolutely has to work. Safety systems and military electronics spring to mind.

On the other hand, these engines do have to survive at least one factory test firing (and possibly more, if they have to be tweaked prior to installation on a vehicle), possibly a test firing on the vehicle out at the pad (SpaceX actually did this about a week before launch, on the vehicle in question), and a service firing during an actual flight. Back in the day, some restartable engines were run up to ten times service burn time with only moderate inspection and maintenance. I would think with non-restartables, one should be able to get at least 5x nominal service burn time out of an engine before you condemn it.
The F-1 was designed for 20 starts and 2250s of firing. RL-10s have been run for very long total times with minimal maintainence (not sure exactly how long), and are rated for 4000s continuous firing.

All of which may be irrelevant if the particular engine had not been run over historical test burn times prior to use on a flight. Then you have to start looking at faulty component and design issues.
I still think that fatigue was probably the deciding factor, and not the runtime itself. The dome (assuming it's in the same place as the one on the F-1) is in a fairly benign part of the engine. High pressure, and thus high stress, but that's easy to deal with. The fuel is not cryogenic, and there's no direct exposure to hot gas to cause erosion. The fatigue from the testing cycles (note that engines are rated both in time and in cycles) would have weakened any flaws, and at some point, it just blew. Admittedly, I haven't gotten to the classes on this stuff yet, but narrow safety margins and multiple cycles make it my best guess.

If the engine was a long runner, or at least near the top of the fleet long run times, one has to ask what the deal was with that. Were they pushing the envelope on their safe burn time experience on flight hardware? That would be pretty, ummm...insane.
Again, if there was some borderline crack (or something just over the line that they missed) the engine could have failed early. I was suggesting that it might have gotten a few more cycles than the other engines. (We need one to test. Bring me engine 1.) That, plus a small flaw, could cause the dome to crack. It might have been the leader among the batch, but almost certainly not fleet-wide. (If it was, I'd cancel their contract with NASA immediately.)

Tony said...

It's been called a "fuel dome", but if SpaceX was following Rocketdyne design practice, it's actually a LOX dome. So there is the issue of cryogenics.

Otherwise, I agree that there is a fatigue issue somewhere in the anomaly. Where the fatigue was is an interesting question. It could have been in the dome material itself, in a weld, or even in one of the bolts holding the dome to the top of the engine (one bolt fails and you get a rapid zipper effect across the whole interface plane, causing the seal to fail and a subsequent pressure loss).

WRT cycling and aggregate burn time, the engine levaes the factory with a log book (probably actually a computer log database, these days). So they should have known exactly how many times the engine cycled and how much aggregate firing time they had on it. One thinks they wouldn't install an overcycled or overtimed engine (compared to firing test articles) in a service vehicle. But if they somehow did...sheesh.

Byron said...

Tony:
It's been called a "fuel dome", but if SpaceX was following Rocketdyne design practice, it's actually a LOX dome. So there is the issue of cryogenics.
I'm aware of that. I was assuming that 'fuel dome' meant what was said, and that the people who said that had more knowledge of the Merlin than I did. It being the LOX supply would fit with what the engine seemed to be doing.

Otherwise, I agree that there is a fatigue issue somewhere in the anomaly. Where the fatigue was is an interesting question. It could have been in the dome material itself, in a weld, or even in one of the bolts holding the dome to the top of the engine (one bolt fails and you get a rapid zipper effect across the whole interface plane, causing the seal to fail and a subsequent pressure loss).
Good point. I'm not certain how the Merlin is constructed, so at this point we don't know. Everything I saw said 'cracked' which implies that it didn't come clean off. If that had happened, I think the injector would have backfired, letting all of the gasses up into the area above the engine. There might well have been a bunch of flames shooting out of the blowout panels. The video isn't great, but I'm guessing that cracked is the correct word.

WRT cycling and aggregate burn time, the engine levaes the factory with a log book (probably actually a computer log database, these days). So they should have known exactly how many times the engine cycled and how much aggregate firing time they had on it. One thinks they wouldn't install an overcycled or overtimed engine (compared to firing test articles) in a service vehicle. But if they somehow did...sheesh.
Again, I'm not suggesting that. If they did do so, then they really have no clue what they're doing. My theory also involves a manufacturing defect.

WRT to said defect, there are two ways it could have happened, either as a fluke (part snuck through QA) or because of a serious lack of QA on SpaceX's part. The question is how often they'd catch a similar defect (which would of course require knowing how bad the original was, which they might not) relative to how often, say, Rocketdyne would. Again, we may never know.

Tony said...

Here's a picture of a partially assembled Merlin on the factory floor:

http://upload.wikimedia.org/wikipedia/commons/4/41/SpaceX_factory_Merlin_engine.jpg

Looks like a pretty standard design, with the propellant injection system bolted onto the top of the combustion chamber, for what that's worth.

WRT catching defects, one would think that everything would be x-rayed before being installed. Maybe they relied on certification of x-ray results from suppliers, or maybe they just didn't include that step in the manufacturing process. Either would be double plus ungood, IMO.

Byron said...

Tony:
Looks like a pretty standard design, with the propellant injection system bolted onto the top of the combustion chamber, for what that's worth.
Sure, but how do we know which inlet goes to the fuel tank? I assumed that somebody who knew what they were talking about said 'fuel dome' and meant it. Not to say they couldn't be wrong, and even if they are, it wouldn't matter much.

WRT catching defects, one would think that everything would be x-rayed before being installed. Maybe they relied on certification of x-ray results from suppliers, or maybe they just didn't include that step in the manufacturing process. Either would be double plus ungood, IMO.
Or it was a marginal part, or somebody just wasn't paying attention right that instant. I'm certainly not saying that I know that there was a mistake, but that sort of stuff does occasionally happen. It might have been abetted by their testing procedures, but I find it inconceivable that they don't have some tests in place.

Tony said...

Byron:

"Sure, but how do we know which inlet goes to the fuel tank? I assumed that somebody who knew what they were talking about said 'fuel dome' and meant it. Not to say they couldn't be wrong, and even if they are, it wouldn't matter much."

AFAIK it's a standard engine design, meaning that the fuel is first run through the expansion nozzle and combustion chamber walls, both to cool the engine and preheat the fuel, and then into the injector plate from the side. The oxidizer is introduced into the injector plate through the dome.

"Or it was a marginal part, or somebody just wasn't paying attention right that instant. I'm certainly not saying that I know that there was a mistake, but that sort of stuff does occasionally happen. It might have been abetted by their testing procedures, but I find it inconceivable that they don't have some tests in place."

If it was a marginal part, then SpaceX's idea of tolerance is flawed, right? With rocket engines, you minimize margin, in deisgn, but you stay well within margin on execution.

WRT to testing procedures, I'm sure they tested their happy little butts off, to the degree that they thought they needed to test. Obviously they didn't test enough, or didn't test for the right things.

Byron said...

Tony:
Definite point about the regenerative cooling. As I said earlier, though, it doesn't really matter, except to make the fatigue a little more likely.

WRT to testing procedures, I'm sure they tested their happy little butts off, to the degree that they thought they needed to test. Obviously they didn't test enough, or didn't test for the right things.
Or an X-ray tech made a mistake and didn't see the problem. The best testing is only as good as the operator. There are ways of the part getting into the engine that involve no serious negligence on the part of the engineers or managers at SpaceX. I'm not saying that it's guaranteed, but we're running into the "be ye perfect" paradox of engineering.
I'm also not saying that there wasn't poor design. As mentioned, we don't know. Did the culture of SpaceX contribute to the accident? Almost certainly. But if we fed 100 similarly defective parts to them and Rocketdyne, would SpaceX take all 100 and Rocketdyne 0? Probably not. My guess is maybe 5 and 1 at the outside.

Tony said...

Hmmm...

Like I said, I'm reserving judgment.

Having siad that, I'm actually hoping its something fairly obvious in either hardware or procedures. If it's a lazy or distracted inspector, no one would likely ever figure out the problem to a significant degree of satisfaction. If they can't pin down a lieky cause, then all they can do is test more and more-redunantly, and hope for the best.

WRT where the "good enough" line lies, it looks like 97-98% is where the industry leaders lie. Where SpaceX would naturally fall if allowed to go on the way they have been is an interesting question, but customers don't want to be guinea pigs. So I think SpaceX, whatever their actual level of culpability and reliability might be, is going to be forced into preemptive investments in reliability, simply because that's the only way to make a sale, especially to multiple-mission customers.

Byron said...

Tony:
Having siad that, I'm actually hoping its something fairly obvious in either hardware or procedures. If it's a lazy or distracted inspector, no one would likely ever figure out the problem to a significant degree of satisfaction. If they can't pin down a lieky cause, then all they can do is test more and more-redunantly, and hope for the best.
That depends on how much of the inspection they recorded. If the X-ray machine copies its scans somewhere where they get stored until after the flight (which I wouldn't be surprised about) they can go back and find it. That said, I'm in agreement with you that it's best if it's obvious.

WRT where the "good enough" line lies, it looks like 97-98% is where the industry leaders lie. Where SpaceX would naturally fall if allowed to go on the way they have been is an interesting question, but customers don't want to be guinea pigs. So I think SpaceX, whatever their actual level of culpability and reliability might be, is going to be forced into preemptive investments in reliability, simply because that's the only way to make a sale, especially to multiple-mission customers.
Also very true. The price for later flights will undoubtedly rise after this, and the main question is how much.

Anonymous said...

Tony:

WRT catching defects, one would think that everything would be x-rayed before being installed. Maybe they relied on certification of x-ray results from suppliers, or maybe they just didn't include that step in the manufacturing process.

SpaceX is very big on vertical integration and builds almost every part of their rockets. They build their own Merlin engines. Looks like they may need to impove their testing.

The engines currently used on the Falcon 9 are the Merlin 1C. SpaceX will soon use their upgraded Merlin 1D engines for an improved Falcon 9. Hopefully, they will learn from this incident and resolve any design issues with the Merlin 1D.

Ron

Rick said...

Welcome to new commenters!

The bottom line for SpaceX - as noted a couple of comments up - is that they will have to do more testing, etc. And the meter is running, which means that SpaceX launches just got more expensive.

Note, by the way, that Virgin Galactic is a year behind their schedule, and their meter is running as well. A suborbital hop doesn't even have much to do with 'space flight' as most of us here mean it (since LEO capability is the prerequesite to going anywhere else). But it does point out that this stuff is hard to do, therefore expensive.


I will give a heads-up here when a new post goes up. Which hasn't happened yet - just as I was trying to get back on a more regular schedule here, Real Life reared its ugly head a bit.

Byron said...

Rick:
The bottom line for SpaceX - as noted a couple of comments up - is that they will have to do more testing, etc. And the meter is running, which means that SpaceX launches just got more expensive.
This is the law of accumulating reliability. The costs devoted to reliability (or safety, if somebody's life depends on the thing) never go down.

Note, by the way, that Virgin Galactic is a year behind their schedule, and their meter is running as well. A suborbital hop doesn't even have much to do with 'space flight' as most of us here mean it (since LEO capability is the prerequesite to going anywhere else). But it does point out that this stuff is hard to do, therefore expensive.
What they're doing isn't really that difficult. I'd guess that a large portion of the delay is regulatory or managerial, not technical.

I will give a heads-up here when a new post goes up. Which hasn't happened yet - just as I was trying to get back on a more regular schedule here, Real Life reared its ugly head a bit.
How dare it? :-)

Locki said...

1. Once round - return to launch capability? (shuttle cross range)

Why is this so useful in wartime anyway? Does anyone want to speculate why the USAF wanted this?

Is it so the shuttle can launch a satellite and return home before the Commies can get off a shot with their 23mm cannon on Mir?

Presumably if the space were so "hostile" you'd just use an expendable, unmanned rocket to get your satellite up there anyway.

Is it just the US Air Force generals getting a little too excited after watching the X-Wing attack run on the Death Star? Was the shuttle intended to drop munitions on a hypersonic flight over the USSR and then quickly return to base?

I really can't imagine any use for this ability.

2. The complexity of space flight and the hacker hubris.

I really, really wish some of you guys were around in the university pub 20-ish years ago when SpaceX was first announced. I think I almost punched out some computer science majors for their hubris.

Computer science was still pretty much a brand new field then and the IT guys and their newly minted billionaire tweenie idols would carry themselves with an almost intolerable confidence.

After all, everyone on campus knew the smartest guy and gals did engineering/law/medicine anyway. Only the leftovers took the computer science major route.

It was just an upheaval of the natural order of things :) (present company excluded Tony!)

Byron said...

Locki:
Why is this so useful in wartime anyway? Does anyone want to speculate why the USAF wanted this?

Is it so the shuttle can launch a satellite and return home before the Commies can get off a shot with their 23mm cannon on Mir?

That's more or less accurate. Do remember that the Soviets were developing ASAT weapons at the time. Using one on the first pass is very difficult, but it's easier if you know the orbit.

Presumably if the space were so "hostile" you'd just use an expendable, unmanned rocket to get your satellite up there anyway.
That would make sense, but the shuttle was supposed to replace all existing launch vehicles.

After all, everyone on campus knew the smartest guy and gals did engineering/law/medicine anyway. Only the leftovers took the computer science major route.
Actually, the smartest person I know (on campus or otherwise) is a CS. But otherwise, I can't disagree too much. At best, it's a dead heat.

Thucydides said...

The Space Shuttle's cross range and "once around" capabilities were to provide the USAF maximum flexibility when conducting a mission. Presumably they could have done a "once around", or chosen to stay in orbit to do "other" things (such as conduct orbital observation, launch or retrieve other satellites or take offensive action against enemy satellites).

The other advantage on a military mission is they could choose to deorbit at any time, since there were several pre designated landing sites around the world. The long cross range also would have put many major airports and airbases in range for landing as well.

Getting the military shuttle home again during wartime would be an exercise best left to the reader.

Locki said...

Byron:

I was being deliberately provocative. TWO of the very brightest guys I know are CS. I know they both lurk here so I was stirring.

Really the average intelligence of a doctor/lawyer/engineer is probably higher than CS but a higher percentage of the uncommonly gifted go into CS. I honestly haven't met anyone I would qualify as a genius in medicine. Whilst I've come across some truly astonishingly talented computer type people.

Maybe medicine attracts a conservative mindset thinker - maybe its the decades of training that breaks you down and makes you conform :).


2.

Thucydides said...

The Space Shuttle's cross range and "once around" capabilities were to provide the USAF maximum flexibility when conducting a mission. Presumably they could have done a "once around", or chosen to stay in orbit to do "other" things (such as conduct orbital observation, launch or retrieve other satellites or take offensive action against enemy satellites).



=======

What sort of offensive action do you speculate the USAF may have wanted the shuttle to take?
Almost any offensive action I can think of would be better done unmanned.

Was it supposed to be able to kick a couple of missiles out of the cargo bay.

The only thing I can possibly think of is if for some reason you wanted to snatch a soviet satellite straight out of orbit and stuff it into the shuttles cargo bay?


I guess the huge cross range gives you lots of options in case you are caught out be bad weather and have to change your landing area at the last minute.

Can the shuttle really land on a bog standard commercial airstrip though? Will it have enough length?

Byron said...

Locki:
Can the shuttle really land on a bog standard commercial airstrip though? Will it have enough length?
I think it will probably suffer significant damage if it does so. The crew will be safe, but the orbiter will likely be totaled.

What sort of offensive action do you speculate the USAF may have wanted the shuttle to take?
Almost any offensive action I can think of would be better done unmanned.

It was supposed to handle all of America's launch needs. So it had to do emergency polar launches too.

Tony said...

"SpaceX is very big on vertical integration and builds almost every part of their rockets. They build their own Merlin engines. Looks like they may need to impove their testing."

Yes, they build their own engines. That doesn't mean that a lot of parts, from simple fasteners to sophisticated subassemblies, aren't subcontracted. For example, I doubt they have their own foundry at the Norwalk plant that can cast a turbopump casing.

In any event, yes, they need to step up the quality control. A flaw that can cause an engine to lose pressure should be caught prior to use in flight.

"The engines currently used on the Falcon 9 are the Merlin 1C. SpaceX will soon use their upgraded Merlin 1D engines for an improved Falcon 9. Hopefully, they will learn from this incident and resolve any design issues with the Merlin 1D."

Merlin 1D uses a lot of the same design principles as the 1C, but it is fabricated in a totally different manner. The combustion chamber on the 1C was either machined and electroplated or totally electroplated (which would actually give you a very strong and uniform part, if done correctly, though it would take weeks to make a single example), while the same part on the 1D is explosively formed.

While one might think that means that any problems with the 1C power head won't migrate into the 1D, it ain't necessarily so. If the problem is simple poor quality control, that's not going to go away with a switch to a new model. If the problem is design, then it is possible that whatever was wrong will be recapitulated.

Too soon to tell...

Tony said...

Re: the intelligence of CS types

It kind of depends on what you mean by "smart". You have to be able to do calculus to take Discrete Math, which is a pretty standard requirement. If one wants to do scientific computing, or software for aerospace applications, one generally has to be able to do even more advanced math.

(Not that math is an absolute measure of intelligence, but it does seem to be the "men from the boys" standard for people in the physical sciences end of things.)

I think the real difference between computer science and engineering is mindset. Engineers are tool makers. Software people are tool users. Except unlike other craftsmen, programmers don't make another physical thing with their tools. They make a logical construct. You can't pick it up and hold it -- or at least stand next to it and admire it -- like a product of mechanical engineering.

Oddly enough, engineers and programmers are usually after the same thing at the most fundamental level -- elegance and efficiency. And they're both constrained by what can be done in the physical world. The most elegant algorithm is useless if the result can't be calculated using a real computer in a reasonable amount of real human time, or if the result set is so large it can't be stored in a physical storage device in the real world. And computer scientists that know what they're talking about (as opposed to gifted hackers who just wirte code) understand physical hardware limitations like bus skew, rotational latency on disks, memory capacity, etc.

Geoffrey S H said...

"The only thing I can possibly think of is if for some reason you wanted to snatch a soviet satellite straight out of orbit and stuff it into the shuttles cargo bay?"

That's been done in a couple of Bond films... ;)

Anonymous said...

"The only thing I can possibly think of is if for some reason you wanted to snatch a soviet satellite straight out of orbit and stuff it into the shuttles cargo bay?"

Don't forget boarding actions! A space station would make a great prize. :)

Ron

Tony said...

Ron:

"Don't forget boarding actions! A space station would make a great prize. :)"

It would actually be pretty useless. Aside from the fact that the US had no military uses for one, the US also didn't have the consumables matching Soviet life support systems.

Byron said...

Locki:
The only thing I can possibly think of is if for some reason you wanted to snatch a soviet satellite straight out of orbit and stuff it into the shuttles cargo bay?
That's a lot harder than it sounds. The launch window for a rendezvous that fast is going to be very small. Also, the satellite operator can do various things to make it more difficult to capture. They'd probably have to perform a spacewalk to capture it (see Intelsat for an example), and I'm pretty sure that the crew couldn't do that in the hour they'd have at the outside.

Tony:
It would actually be pretty useless. Aside from the fact that the US had no military uses for one, the US also didn't have the consumables matching Soviet life support systems.
I'm sure they could improvise something out of plastic bags and duct tape. Also, I think we may have had the specs on those from ASTP.

Anonymous said...

Don't forget boarding actions! A space station would make a great prize. :)"

"It would actually be pretty useless. Aside from the fact that the US had no military uses for one, the US also didn't have the consumables matching Soviet life support systems."

Boy, you guys don't know a joke when you see one. Lighten up.

The reason why the Air Force wanted to use the shuttle was because it was supposed to be far cheaper to operate than expendable rockets. All US space operations were to be handled by the shuttle. The engineers at the time were far too optimistic. They wanted to build a space truck, but they built a F1 race car, requiring a rebuild after every flight.

The shuttle was an amazing vehicle, but when you get right down to it, it was an operational failure.

One reason we don't have a shuttle replacement flying today is because the Clinton administration picked the Lockheed X-33 SSTO spaceplane to replace the shuttle. Once again, the engineers reached too far. This time they couldn't get past fueling tests and never had a launch.

Maybe Skylon will work, but I'm not betting on it. At least NASA is going back to good old expendable rockets for US human spaceflight.

Ron

Tony said...

Ron:

"Boy, you guys don't know a joke when you see one. Lighten up."

Stuff like that has been suggested in all seriousness in the past, both here and in many other places. So it has to be treated as a serious suggesxtion, no matte how outrageous one might personally think it to be.

Thucydides said...

Snatching a Soviet satellite was indeed a serious suggestion for the USAF Space Shuttle (and the Soviets did treat this as a serious issue for quite some time), but putting some sort of ASAT weapon or even an electronic warfare device to jam or scramble satellite signals in the cargo bay was probably a more realistic approach.

Of course using the Space Shuttle as a warcraft was about the same level as putting armour plate and crossbowmen on the tanker truck in "The Road Warrior", but it was probably cool as hell and something to dream about at the Air Force Academy.

Scott said...

Who else remembers the Sioux City crash of a DC-10? http://en.wikipedia.org/wiki/United_Airlines_Flight_232

Root cause was an inclusion in the fan disk, that acted as a crack source. Then the entire 8-foot-diameter fan section came apart.

While that particular casting flaw should have been detected during manufacturing QA (and also caught by maintenance), it's entirely possible that a similar flaw was not detected in the making of the engine that failed.

Tony said...

Scott:

"Who else remembers the Sioux City crash of a DC-10? http://en.wikipedia.org/wiki/United_Airlines_Flight_232

Root cause was an inclusion in the fan disk, that acted as a crack source. Then the entire 8-foot-diameter fan section came apart.

While that particular casting flaw should have been detected during manufacturing QA (and also caught by maintenance), it's entirely possible that a similar flaw was not detected in the making of the engine that failed."


It should be a very well remembered incident when talking about aerospace quality control and quality assurance, both in manufacturing and operations.

Considering how rarematerial failure engine anomalies are in launch vehicles these days, it seems almost certain that something was missed by somebody.

Locki said...

Byron said...

It was supposed to handle all of America's launch needs. So it had to do emergency polar launches too.


=======================

I keep reading this and yet only 4 shuttles were built. Which is a bit confusing. What was the reason for this. If occurs to me there’s two completely contradictory explanations for the low number. Which is it?

1. If they had operated as promised (eg monthly space launches) would 4 always have been more than enough?

2. If they had operated as promised (eg cheap space launch) then they would have built a lot more to take the load.



Thucydides said...

Snatching a Soviet satellite was indeed a serious suggestion for the USAF Space Shuttle (and the Soviets did treat this as a serious issue for quite some time), but putting some sort of ASAT weapon or even an electronic warfare device to jam or scramble satellite signals in the cargo bay was probably a more realistic approach.

Of course using the Space Shuttle as a warcraft was about the same level as putting armour plate and crossbowmen on the tanker truck in "The Road Warrior", but it was probably cool as hell and something to dream about at the Air Force Academy.


==========================

I think many of us were envisaging a Moonraker scenario with the cargo bay hold 20 astronauts in EVA and laser rifles duking it out around Drax’s space station

It occurred to me that maybe the primary rationale for the huge cross range was a defensive requirement of the USAF. Eg it enables the shuttle to get the hell out of dodge if the bad guys come looking.

Which rather begs the question why you aren’t using expendable one shot launchers in wartime anyway ….
On a fantastical “Clancy” type scenario.

Docking with a foreign station that is not compliant is going to be impossible I’d presume. For starters how are you even going to get in without depressurising the station?

Any clever lateral thinkers want to suggest how the Chinese may be able to get onboard the ISS if they were unwelcome?

Byron said...

Ron:
Technically, the X-33 was a suborbital prototype of the VentureStar SSTO.

Locki:
1. If they had operated as promised (eg monthly space launches) would 4 always have been more than enough?
That's pretty much it. The turnaround was supposed to be about 2 weeks. That's 2 launches a week, which is (IIRC) well above the normal rate. Probably one would have been at Vandenberg full-time.

It occurred to me that maybe the primary rationale for the huge cross range was a defensive requirement of the USAF. Eg it enables the shuttle to get the hell out of dodge if the bad guys come looking.
Why? It does increase your chances of getting to a suitable field, but you can get down whenever. And the video series Tony linked to above explicitly tied it to once-around from Vandenberg.

Which rather begs the question why you aren’t using expendable one shot launchers in wartime anyway ….
That would make sense. The preliminary design of the shuttle was notably lacking in that area.

Docking with a foreign station that is not compliant is going to be impossible I’d presume. For starters how are you even going to get in without depressurising the station?

Any clever lateral thinkers want to suggest how the Chinese may be able to get onboard the ISS if they were unwelcome?

Oddly enough, I've read through all of the old threads here recently on this sort of thing (for the paper.) First off, there's an extra (probably inflatable) airlock. Just cut your way in. Or mess with things outside the station. Cut the radiators, and they either have to come out or boil to death. And they could legally demand entry, under the Outer Space Treaty.

jollyreaper said...

The SpaceX commentary here isn't striking me so much as prudently conservative (which I am completely for) but old man stick in the mud.

My personal experience with getting bamboozled by sharks in suits relates to working in and living through the dot.com boom. I also grew up in the christian church so I know BS artists when I see them, I know hype, I know pump-and-dump scams and get-rich-quick schemes that rely on starry-eyed suckers who believe in the dream.

Now some people might not be setting out to scam others but are still full of it. These are impractical idealists who believe that vague handwaves can substitute for real solutions, whose boundless optimism refuses to address serious engineering difficulties.

The real kicker here is that none of us personally have skin in this game aside from tax dollars we have no control over contracting for SpaceX services. This isn't like all of us in a room deciding on going in for a time-share and none of us are going to personally gain or lose regardless of what happens. At most we're like fans watching the team make a go for the championship and the thrills are vicarious.

They call it rocket science because it's tough. There are going to be mistakes. The rocket survived a pretty serious failure. I'm amazed that SpaceX has never lost a vehicle. That's a pretty rare record in the history of space travel.

SpaceX has set extraordinary goals but they've already done stuff along the way I never thought I'd see outside of fiction. Seriously. Private space flight? I'd given up on that around the same time I realized we'd never be seeing O'Neill colonies or a serious human presence in space in the PMF. The future we wanted was never going to happen. So this is a lot more than I'd ever expected. I'm grabbing the popcorn and watching with interest.

Locki said...

jollyreaper said...

The SpaceX commentary here isn't striking me so much as prudently conservative (which I am completely for) but old man stick in the mud.

My personal experience with getting bamboozled by sharks in suits relates to working in and living through the dot.com boom. I also grew up in the christian church so I know BS artists when I see them, I know hype, I know pump-and-dump scams and get-rich-quick schemes that rely on starry-eyed suckers who believe in the dream.

Now some people might not be setting out to scam others but are still full of it. These are impractical idealists who believe that vague handwaves can substitute for real solutions, whose boundless optimism refuses to address serious engineering difficulties.

The real kicker here is that none of us personally have skin in this game aside from tax dollars we have no control over contracting for SpaceX services. This isn't like all of us in a room deciding on going in for a time-share and none of us are going to personally gain or lose regardless of what happens. At most we're like fans watching the team make a go for the championship and the thrills are vicarious.

They call it rocket science because it's tough. There are going to be mistakes. The rocket survived a pretty serious failure. I'm amazed that SpaceX has never lost a vehicle. That's a pretty rare record in the history of space travel.

SpaceX has set extraordinary goals but they've already done stuff along the way I never thought I'd see outside of fiction. Seriously. Private space flight? I'd given up on that around the same time I realized we'd never be seeing O'Neill colonies or a serious human presence in space in the PMF. The future we wanted was never going to happen. So this is a lot more than I'd ever expected. I'm grabbing the popcorn and watching with interest.


I've read back over the posts, including my won.

I think you've made a fair point.

Its almost sounds like I was willing SpaceX to fail so I could tell them all "I told you so."

An uncharitable attitude at best.

Fair point.

Are Ferrel/SA Phil et al ready to come back and start posting to maintain the balance again?

«Oldest ‹Older   1 – 200 of 249   Newer› Newest»